Jump to content

Wikipedia:Reference desk/Science

From Wikipedia, the free encyclopedia

This is an old revision of this page, as edited by Maelin (talk | contribs) at 16:36, 16 May 2009 (→‎is there any evidence that the NSA uses Papal infallibility?: No.). The present address (URL) is a permanent link to this revision, which may differ significantly from the current revision.

Welcome to the science section
of the Wikipedia reference desk.
Select a section:
Want a faster answer?

Main page: Help searching Wikipedia

   

How can I get my question answered?

  • Select the section of the desk that best fits the general topic of your question (see the navigation column to the right).
  • Post your question to only one section, providing a short header that gives the topic of your question.
  • Type '~~~~' (that is, four tilde characters) at the end – this signs and dates your contribution so we know who wrote what and when.
  • Don't post personal contact information – it will be removed. Any answers will be provided here.
  • Please be as specific as possible, and include all relevant context – the usefulness of answers may depend on the context.
  • Note:
    • We don't answer (and may remove) questions that require medical diagnosis or legal advice.
    • We don't answer requests for opinions, predictions or debate.
    • We don't do your homework for you, though we'll help you past the stuck point.
    • We don't conduct original research or provide a free source of ideas, but we'll help you find information you need.



How do I answer a question?

Main page: Wikipedia:Reference desk/Guidelines

  • The best answers address the question directly, and back up facts with wikilinks and links to sources. Do not edit others' comments and do not give any medical or legal advice.
See also:



May 8

Skin color in Hindu art

I've looked on Wikipedia but haven't found an answer to my question. In various types of Hindu and/ or Indian visual arts, various divine figures and some non-divine figures (as far as I can tell, not having much knowledge of Hinduism), are often depicted with skin tones of bluish tint, or other colors which appear to be non-naturally occuring. Is there any scientific explanation for why such skin tones are used? What is the symbolic/religious meaning? Thank you. --71.111.205.22 (talk) 01:04, 8 May 2009 (UTC)[reply]

Well from a scientific point of view there's not much i can say about this. But from a religious point of view, many hindu gods, as you, said, are depicted blue. Vishnu and many of his avatars are portrayed blue, which is what you might have been. This might have something to do with the fact that the sea, highly regarded as a symbol of power by the hindus, is blue. But that's not saying much... some questions really don't have a comprehensive answer...Rkr1991 (talk) 08:24, 8 May 2009 (UTC)[reply]

Krishna is often portrayed in blue. Hinduism uses a lot of symbolism to depict its deities and blue, amongst other things, denotes transcendentalism. Remember, the blue depiction of god is in our material universe and on our planet, the sky is blue. The gods are in the spiritual abode and the entire material universe is a cloud in the sky of the spiritual abode. So, do you get the symbolism now? Sandman30s (talk) 08:41, 8 May 2009 (UTC)[reply]
That helps a lot. Thanks for all your assistance! --71.111.205.22 (talk) 15:14, 8 May 2009 (UTC)[reply]

Among the Hindu Trimurti (see File:Brahma Vishnu Mahesh.jpg):

  • Vishnu and his avatars Rama, Krishna etc. are typically depicted blue-skinned (often with yellow dhoti); in fact the word Krishna literally means "black; dark; or dark blue"
  • Shiva is depicted white-skinned (with a blue tongue)
  • Brahma is depicted either red-skinned and/or wearing a red dhoti.

Other deities have different associated colors and symbols; see this very incomplete table for some more information. It is possible to interpret all these associations symbolically but one should be aware that the symbolic meaning is likely to vary with time, place and philosophical leanings of different sects. Abecedare (talk) 18:27, 8 May 2009 (UTC)[reply]

Shiva's blue throat is a result of the poison he holds there. Jay (talk) 11:38, 11 May 2009 (UTC)[reply]
FWIW Osiris, the ancient Egyptian god of the underworld, was commonly depicted as a green (the color of rebirth) or black (alluding to the fertility of the Nile floodplain) complexioned pharaoh in mummiform. Cuddlyable3 (talk) 07:24, 13 May 2009 (UTC)[reply]

Gravitational potential of an 'eccentric' object

I'm trying to find the gravitational potential of the 3-dimensional object described by a sphere of constant density and radius 1 centered at the origin (0,0,0) - 'A', but hollow in the sphere of radius 1/2 centred at (1/2,0,0) - 'B' - and hollow in the sphere of radius 1/4 centred at (-1/4,0,0) - 'C' - for some point x outside of A- effectively a full sphere of density plus 2 spheres of density as described previously. Can we just effectively 'add' the potentials for the respective spheres so that, if M is the mass of the large sphere 'A', we have , where b and c are the respective sphere centers? Is this at all valid?

If so, is there any point in C where a particle could theoretically remain at rest? I think not due to the asymmetry along the x-axis but I'm not sure how to show it...

Thanks very much! Mathmos6 (talk) 04:27, 8 May 2009 (UTC)[reply]

You can just add the potentials since Newtonian gravity is linear, but the answer you wrote down isn't correct because GM/r is the potential of a point particle, not a sphere of constant density. (Also it has the wrong sign.) Your potential is fine (aside from the sign error) if you're only interested in the |x| > 1 region. There is an equilibrium point somewhere in C because every potential function, no matter how misshapen, has to have a minimum somewhere. The minimum in this case is a ring around the x axis. There's also a saddle point (unstable equilibrium) on the x axis. -- BenRG (talk) 11:42, 8 May 2009 (UTC)[reply]
Sorry, I just noticed I didn't read your question very carefully. I think the saddle point is in C (the hole of radius 1/4). It shouldn't be hard to prove that, but you'll need to write down an inside-C, inside-A, outside-B potential. -- BenRG (talk) 12:23, 8 May 2009 (UTC)[reply]
Oh, also, the potential is a scalar, so you want , not . Funny I didn't notice that before. -- BenRG (talk) 12:31, 8 May 2009 (UTC)[reply]

Fair enough, I was only interested in the region outside the largest sphere yes, although I can't say I'd know how to calculate the potential inside the big mess in A - also, I see your point about the scalar, but don't we need to take the fact that the potential isn't radially symmetrical into account somewhere though? Or do the 'x-b'/'x-c's do that anyway? Thanks a lot! Mathmos6 (talk) 03:51, 10 May 2009 (UTC)[reply]

The |x-b| and |x-c| takes care of that. There are four regions (inside B, inside C, inside A but outside B and C, and outside A) and you'll have a different formula for each one. Or you could write down a single formula with max/min or some such, but you'd end up having to split into four cases to prove anything interesting anyway. If you can write down the potential inside and outside a single spherical object then you can write down the potential in all four regions, since it's just a sum of the inside/outside potentials of A, B, and C (with the B and C potentials negated). -- BenRG (talk) 22:42, 10 May 2009 (UTC)[reply]

How to ensure 100% RDA with supplements?

Dietary supplements usually contain 100% RDA of some micronutrients, but some ridiculously high or low value of many others, if they contain them at all. Given that it is prohibitively complicated to find out how much of each micronutrient we actually get in our food each day, and given that it probably won't hurt to get a little bit more than 100%, wouldn't it be nice if there were a supplement on the market that had simply 100% of every micronutrient, just to be safe? Why is there no such "100% supplement" available? Or is there? Mary Moor (talk) 06:02, 8 May 2009 (UTC)[reply]

You don't say where you are, but in the UK there are such supplements available: Centrum makes one [1] --TammyMoet (talk) 08:37, 8 May 2009 (UTC)[reply]
Yep, I've got a bottle of '100% everything' in my own cupboard. I got mine from Boots, I think - though I've seen similar in most pharmacies and health food shops that I've been into... --Kurt Shaped Box (talk) 10:31, 8 May 2009 (UTC)[reply]
Thank you; maybe I have to look some more. I'm in the US, in the state of Washington. Mary Moor (talk) 15:46, 8 May 2009 (UTC)[reply]
A word of advice. Don't pay extra for products that claim to contain 500% (or more) of your RDA. You don't need *that* many micronutrients and vitamins. I don't think that your body can even do anything useful with the excess.--Kurt Shaped Box (talk) 00:21, 9 May 2009 (UTC)[reply]
Not only do you not need that level, high levels of some micronutrients can make you quite ill. Franamax (talk) 09:11, 9 May 2009 (UTC)[reply]
Unless you've got a very unhealthy diet, vitamin pills will not do you any good. Overdoses of vitamins in fact do you harm - more is not better. I used vitamin pills for many years until I started reading scientific research about them, which showed that overdoses reduced longevity, increased cancer rates, kidney damage, etc. Now I never use them (with the exception of Vitamin D in the winter as I am in northern latitudes). Better to put your time and money into eating a healthy varied diet - plenty of vegetables and fruit etc. Its very likely that there are many as yet undiscovered micro-nutrients that have long term effects, and these are only found in healthy foods, not pills. If you want to get particular vitamins or minerals then its best to find out which foods are rich in them and eat those. For example B12 - sardines. Selenium - brazil nuts. And many others. Sardines are also rich in calcium and Omega3, which illustrates how natural foods give you extra nutrients for free. 78.146.190.197 (talk) 13:00, 16 May 2009 (UTC)[reply]

neutrons, protons and electrons

During my one year of chemistry class at school, I had other things on my mind. But now I try to learn on my own...
My very elementary question is: are all neutrons, protons and electrons the same?? That is, can you see for instance, this is a typical oxygen electron and that is a typical sodium electron? Or is there no such difference? Lova Falk (talk) 06:17, 8 May 2009 (UTC)[reply]

Yes, as far as we know, all protons are identical to each other. None of them have nicks, bumps, or customized parts hanging off of them. Same with neutrons, electrons, etc. An electron is an electron - there's no difference between oxygen electrons and sodium electrons. We have an article (of course), Identical particles, but that's rather advanced. Clarityfiend (talk) 06:59, 8 May 2009 (UTC)[reply]
Thank you! This is actually a bit shocking. In the articles on neutrons, protons and electrons, the article Identical particles is not mentioned in "See also". Do you think I should put a link there? Lova Falk (talk) 07:17, 8 May 2009 (UTC)[reply]
Probably not, because this is a generic property of all quantum particles (and even atoms and small molecules). Maybe subatomic particle should link to it. -- BenRG (talk) 21:53, 8 May 2009 (UTC)[reply]
See no-hair theorem for the black hole version of this identity issue. --Sean 14:06, 8 May 2009 (UTC)[reply]
They're not quite the same. For example: The two electrons in a helium atom have opposite spin. It's easy to change these differences, however. — DanielLC 15:45, 8 May 2009 (UTC)[reply]
No, Daniel, you are wrong. Single-electron states may have opposite spin, but the electrons occupying these states are still identical. If they were not identical (i.e., distinguishable), then the helium atom would have had many more quantum states than it actually has. Both atomic physics and statistical mechanics would have looked very differently if at least some of the electrons were distinguishable. --Dr Dima (talk) 18:25, 8 May 2009 (UTC)[reply]
It's more than that. "One electron up and one down" is the same quantum state as "one left and one right" or any other pair of opposite spin directions, so the electrons lose their identities in a more profound way than mere swappability. You don't have two electrons with opposite spins, you just have a compound state with zero spin (a singlet state) that doesn't divide into electrons any more uniquely than a two-liter bottle of soda divides into liters. Still, I think what DanielLC said was reasonable—you can have one electron over here and one over there and none in between, and though they are exchangeable in the wave function they aren't going to swap places with each other by crossing the intervening space. That's a kind of distinguishability, though it's not the technical sense in which the word is used in quantum mechanics. -- BenRG (talk) 21:53, 8 May 2009 (UTC)[reply]

Surface tension

What will be the Condition of spreading of one liquid over another and its expression ?Supriyochowdhury (talk) 06:51, 8 May 2009 (UTC) —Preceding unsigned comment added by Supriyochowdhury (talkcontribs) 06:50, 8 May 2009 (UTC)[reply]

I'm sorry, but I had trouble understanding your question. Could you try again with a little more detail? Thanks. --Sean 14:08, 8 May 2009 (UTC)[reply]
I think Supriyochowdhury is asking for a word for situations like having a layer of oil on top of water, and for the conditions that permit this to happen. I don't know the answers, though. Looie496 (talk) 15:58, 8 May 2009 (UTC)[reply]

== I want to about spreding coefficient $ba (where liquid b spread over liquid a)of a liquid and how can I derived.Supriyochowdhury (talk) 17:54, 8 May 2009 (UTC)[reply]

See spreading coefficient. --Heron (talk) 19:27, 8 May 2009 (UTC)[reply]

British bird identification/song question

There is a bird outside one of the places I reside frequently which makes a "doo-deladoo-delaadoo" (sorry for the crappy transcription) song over and over again, with the pitch going up and then it making a thrush sort of strangled noise, for long periods of time, most of the day. It will occassionally switch a bit, but this is what it does at least 90% of the time. It's been going on for a few months, at least, even into night sometimes. The place is a housing estate, but it backs out onto some wild scrubland/wooded area. First guess would be a blackbird but if so, why does it seemingly have such a restricted singing voice? Thanks for any answers 82.26.198.108 (talk) 07:38, 8 May 2009 (UTC)[reply]

Sounds a bit like a wood pigeon to me, check out the audio file in the article, it certainly has a very repetitive call. Mikenorton (talk) 07:58, 8 May 2009 (UTC)[reply]
Or try [2]. Bazza (talk) 13:29, 8 May 2009 (UTC)[reply]
Common Blackbird#Behaviour mentions the fact that some of them imitate noises and other things they heard in their song. (OR: There used to be one near my aunt's home that did a bit of a Mozart tune followed by a motor saw.)71.236.24.129 (talk) 13:25, 9 May 2009 (UTC)[reply]
It probably is a blackbird. Maybe it's just not very good at music. Nothing else sticks out of the songs I tried on the RSPB site and it's definitely not a wood pigeon. Thanks for the help everyone! 82.26.198.174 (talk) 19:15, 9 May 2009 (UTC)[reply]
Now that I read your description of the song as if sung by a blackbird, then it does sound like a blackbird. They have different calls - this is not their alarm call, but sounds like the call I recall hearing when all is peaceful. 78.146.17.231 (talk) 17:03, 16 May 2009 (UTC)[reply]

Identifying a tree

Can someone identify the species of the large tree growing directly in front of the building in this image? Thanks! LANTZYTALK 08:04, 8 May 2009 (UTC)[reply]

Podocarpus perhaps? Where was this photograph taken?CalamusFortis 15:32, 8 May 2009 (UTC)[reply]

Detective work says at Gujurat University in Ahmedabad, India. Looie496 (talk) 16:05, 8 May 2009 (UTC)[reply]
Podocarpus seems very plausible. Thanks! LANTZYTALK 17:16, 9 May 2009 (UTC)[reply]

Magnatite and hydrochloric acid

All of my searches to date indicate that magnatite dissolves slowly in hydrochloric acid, but no time frame. Does anyone now the time to dissolve, cold and warm —Preceding unsigned comment added by 86.137.11.144 (talk) 08:59, 8 May 2009 (UTC)[reply]

Speed of dissolving will depend on much more than the identity of the substances involved. For example, ground magnetite will dissolve much faster than a solid chunk, and a stirred solution will dissolve much faster than one left to sit idle. It will also depend on the concentration of the HCl, and as you thought, on the temperature. There are just far too many factors to give a definitive answer, variations of any one of these could change the speed of the reaction by orders of magnitude. --Jayron32.talk.contribs 00:05, 9 May 2009 (UTC)[reply]
From testing in lab: 3M HCL will dissolve magnetite powder almost instantly at RT, useful for cleaning. But 1M HCl has to be left at least a few hours, and more dilution solutions can take weeks, to the extent that it is no problem for washing to remove basic residues.YobMod 10:17, 14 May 2009 (UTC)[reply]

Random number

Can humans generate perfectly random numbers between any two arbitrary limits? - DSachan (talk) 09:43, 8 May 2009 (UTC)[reply]

Our page Random_number_generator_attack#Human_generation_of_random_quantities may help. 194.221.133.226 (talk) 10:19, 8 May 2009 (UTC)[reply]

It all comes down to what you define "random number" to be. --98.217.14.211 (talk) 10:23, 8 May 2009 (UTC)[reply]
(ec):No humans can't. It is well known that when humans are asked to pick numbers at random, they are strongly biased to particualr numbers, a fact often made use of by stage magicians. Computer algorithms for generating pseudo-random numbers are not truly random, but appear to be so for all practical purposes because the algorithm is seeded by a continuously changing number such as the time and date. One of the best hardware methods for producing random numbers is to amplify thermal noise. However, even this is not truly random since the signal must necessarily be band-limited which slightly biases against large differences in successive numbers over short intervals. There is also the question of what is meant by random. If the requirement is to build a machine that can output integers within a certain range with equal probability then a machine can be built to approximate to this. If the requirement is to output any real number randomly between two limits then no finite machine is capable of doing this. The reason is that there are infinitely many numbers between any two points. Any real (finite) machine will have a finite resolution and cannot possibly represent all of them, only a finite number of them. In fact, a digital machine is only able to represent rational numbers and these have a zero probability of occuring in a true random sequence of numbers even if the machine was capable of producing any of the infinite rational number between the stated limits. Why? because there are infinitely more irrational numbers in any interval than there are rational numbers, even though the number of both is infinite. SpinningSpark 10:30, 8 May 2009 (UTC)[reply]
When discussing "true random", an example I give that tends get the point across is that a true random number generator programmed to output true random numbers between 0 and 9 should have the possibility of outputting 1 1 1 1 1 1 1 1 1 1 1 ... The point is that it is truly random. Even if every number previously has been 1, the chance that the following number will be 1 is just as high as every other number. So, a generator that ensures the distribution of numbers is equal is not truly random. -- kainaw 13:24, 8 May 2009 (UTC)[reply]
That's true, but actual (computer) pseudorandom number generators don't try to even things out—they're just as likely to produce long sequences of ones as real random number generators are.
Computers aren't limited to rational numbers. They can work with numbers of the form where a and b are rational, for example. More interestingly they can represent the whole collection of computable reals. Computable real arithmetic systems work by successive approximation, and there's no reason you couldn't add genuine uncomputable random reals to such a system if you have a random oracle available in hardware. /dev/random is supposed to give you genuine quantum randomness, so it should work. -- BenRG (talk) 14:20, 8 May 2009 (UTC)[reply]
True, but the point stands - computable numbers are still only countable (the same size as the rationals), so you can still be almost certain that a number chosen at random from a bounded interval with a uniform distribution will not be computable. I don't see how you can add uncomputable numbers to a computer system, a computer can only work to finite precision. You might be able to fiddle around and extend the set of computable numbers a bit, but you won't stop it being countable. --Tango (talk) 14:27, 8 May 2009 (UTC)[reply]
Well, there's no real number that you can prove won't occur (exactly) in an exact-real-arithmetic system running on a machine with /dev/random. It may be philosophically dubious, but I don't think randomness gets any less dubious than that. Sure, a Turing machine with a given oracle can only be in countably many states and will only reach finitely many of those, but by the same token there are only countably many theorems about real numbers in ZFC and only finitely many of those will ever be written down. Only finitely many digits of pi will ever be known to the human race, every formal system has a countable model, etc. Of course, mathematicians never actually generate random numbers, they just reason about what would happen if they did, and computers can do that too. -- BenRG (talk) 20:33, 8 May 2009 (UTC)[reply]
Only because I wouldn't be able to describe such a number, but that doesn't mean much - almost all real numbers are undefinable. (There are describable numbers that aren't definable real numbers, but I believe the set of all describable numbers is still countable, by even the broadest definitions - I could be wrong, though.) --Tango (talk) 21:41, 8 May 2009 (UTC)[reply]
Getting off-topic here a little bit, but I kind of think this is important to repeat: There is no single, mathematically precise, notion of what it means for a real number to be definable. You have to say definable how. This has been the problem from the beginning at the definable real number article, and I have never been able to come up with a truly satisfactory way of making that article both correct and compliant with WP standards. I'm not sure it can be done. The option of campaigning for its deletion has certainly crossed my mind, but that won't make the issues go away; they'll just spread out through other articles, so I don't really think that's a good idea either. --Trovatore (talk) 03:52, 9 May 2009 (UTC)[reply]
Indeed, but I think any reasonable definition gives you only countably many numbers, so it doesn't really matter in this case. --Tango (talk) 13:21, 9 May 2009 (UTC)[reply]
It also depends on what you mean by "generate", as a human certainly can build a Geiger counter which s/he can then use to generate perfectly random numbers. A human can also shuffle a deck of cards pretty well, which can also generate good quality random numbers. --Sean 14:11, 8 May 2009 (UTC)[reply]
Geiger counters are not perfectly random in the mathematical sense and humans probably cannot shuffle as well as you think they can. SpinningSpark 19:42, 8 May 2009 (UTC)[reply]

When two waves intercept at a right angle

Do they interfere with each other? If yes, what types of waves interfere with each other?--Mr.K. (talk) 11:21, 8 May 2009 (UTC)[reply]

Yes, you will get additive interference just as any other case which combines waves. If you know the wavefunction for each wave, you can do a point-by-point combination of the phase for each wave, to determine the resulting total wave amplitude. The interference pattern will depend on whether you have two plane waves, wave-packets, or something else; in the case of two plane waves at right-angles, your interference pattern will be sort of "checkerboard-like" with constructive and destructive interference. Nimur (talk) 12:24, 8 May 2009 (UTC)[reply]
It depends what you mean by "interfere". There will be interference in the sense of transient superposition, as described by Nimur, but not necessarily interference in the sense of a permanent distortion. Gandalf61 (talk) 12:29, 8 May 2009 (UTC)[reply]
OK, if I put two lasers each one intercepting the other at 90 degrees. Will they affect each other in any way? (even if for practical purposes, it's irrelevant).--Mr.K. (talk) 17:11, 8 May 2009 (UTC)[reply]
Did you see our article on interference? They will affect one another, in the region of intersection; but if you were to measure/observe one of the beams outside the region of intersection, you wouldn't be able to tell that it had "crossed" with another beam. If you do the mathematics of adding the sine-waves and solve the wave equation, you will find this result is physically consistent. Nimur (talk) 19:42, 8 May 2009 (UTC)[reply]
In quantum electrodynamics there is an interaction between the beams, mediated by virtual electrons (the simplest Feynman diagram is this one). It's a small effect but it's large enough to be detectable. In principle there's a very tiny gravitational attraction too. Roughly speaking, though, they just pass through each other. -- BenRG (talk) 20:15, 8 May 2009 (UTC)[reply]
If they are in a non linear material you may get non linear optics effects with modulation harmonics or mixing occurring. Graeme Bartlett (talk) 09:02, 17 May 2009 (UTC)[reply]

youtube

How does this work

http://www.youtube.com/watch?v=e3kyNGVK-hI

Thanks--Mudupie (talk) 11:21, 8 May 2009 (UTC)[reply]

I've only watched the first minute or so, but it looks genuine to me. I imagine if one were to beat box into a flute (with sufficient skill) that is what it would sound like. --Tango (talk) 11:30, 8 May 2009 (UTC)[reply]
I agree, it's probably genuine. As for how it works, well the flute works by acoustic resonance, and the beat-boxing is sort of augmented by the percussive effect of the microphone amplifier clipping. Nimur (talk) 12:27, 8 May 2009 (UTC)[reply]

mWS in a mesurement of pressure drop

What´s means "mWS" in a mesumement of pressure drop or in a indication of pressure drop of a equipment? —Preceding unsigned comment added by Gigobqto (talkcontribs) 15:31, 8 May 2009 (UTC)[reply]

It is metres of water (approx 104 Pa, see Conversion of units#Pressure or mechanical stress) but I do not know what the "S" stands for. In the unit mW(g), the g stands for guage pressure, meaning the pressure is metres of water relative to atmospheric pressure, so I would surmise that mWS is measured relative to something else. Perhaps vacuum or negative guage pressure? SpinningSpark 16:19, 8 May 2009 (UTC)[reply]
Ah found it on German Wikipedia, it's metres Wassersäule which if my useless technical German is not letting me down means column of water. It seems to be used exclusively for pressure differences, such as the pressure drop along a supply pipe, and is distinguished from both absolute pressure and guage pressure. SpinningSpark 17:33, 8 May 2009 (UTC)[reply]

buffer solution

Hi I have been asked to write a symbol balanced equation for equilibrium of a buffer solution of 0.6 mol of propanoic acid and 0.8 mol of sodium propanoate. Would the answer be: CH3CH2COOH (reversable arrows) CH3CH2COO- + H+? I'm guessing its not CH3CH2COO-,Na+ (reversable arrows) CH3CH2COO-+Na+ because the next question is asking for a balanced symbol equation for the dissociation of sodium propanoate. Please help! —Preceding unsigned comment added by 92.18.81.46 (talk) 15:39, 8 May 2009 (UTC)[reply]

Looks good to me! Not that you mentioned it, but you may find Buffer solution and Henderson–Hasselbalch equation to be relevent reading. --Jayron32.talk.contribs 20:37, 8 May 2009 (UTC)[reply]
So. Balanced symbol equation. What's that then? Even google thinks we might not know. --Tagishsimon (talk) 00:10, 9 May 2009 (UTC)[reply]

Speed of magnetic and electric fields

What are the speeds of the magnetic field and electric field? Can we treat their speeds like that of an electromagnetic wave? --Email4mobile (talk) 17:27, 8 May 2009 (UTC)[reply]

Generally speaking electric and magnetic fields are not going anywhere at all unless the source of the field is itself moving. What does move is a disturbance in the field and such disturbances are called electromagnetic waves. The speed of these disturbances is c, the speed of light. Also note that there are not really separate electric and magnetic fields, but only the electromagnetic field, and the two are united through the Lorentz transformation. SpinningSpark 17:56, 8 May 2009 (UTC)[reply]

Thanks S Spinningspark for this explanation, but how can I describe the attraction and repple forces without accepting that these fields must move like radiation. For example If I've a giant magnet and another giant magnet was passing by very fast causing the nearest distance between them to be let's say 300,000 km. Will the maximum attraction|repple force take place at that position or after the magnet has passed by another distance proportional to its velocity? —Preceding unsigned comment added by Email4mobile (talkcontribs) 18:27, 8 May 2009 (UTC)[reply]

First of all, it is wrong to think of the magnetic field being solidly attached to the magnet and moving with it through space. Rather, the magnet is causing causing a disturbance in the field as it moves through it. That disturbance is propagated through the field and finally arrives at the second magnet. As for where the moving magnet is when the maximum effect is felt at the stationary magnet: ask yourself this, if instead of a moving magnet we have a moving star, where is the star when the maximum light from it is seen at the fixed position? SpinningSpark 19:10, 8 May 2009 (UTC)[reply]
It works surprisingly well to think of the field as being solidly attached to the magnet. When you push on a solid object the rest of it doesn't move instantly, there's a ripple effect through the object at the speed of sound. The magnetic field acts in a similar way, with the radiation being the ripple. When you stop pushing, the object eventually returns to its original shape, and the same happens to the field. I think possibly you (the original poster) are wondering how much the magnet's field lags behind the magnet when the magnet is moving. The (somewhat surprising) answer is that it doesn't lag at all, as long as the speed is constant. The maximum force between the magnets will be at the point of closest approach (which is the same from the rest frame of either magnet). The star is different—the maximum intensity of light will be somewhat before the time of closest approach. -- BenRG (talk) 21:19, 8 May 2009 (UTC)[reply]

Thank you BenRG and Spinningspark. Indeed the reason, I raised that question was that I was wondering why a conductor has to cut the magnetic lines in order for the e.m.f to be produced. If we think about massive bodies and the fields we will find that massive bodies have a relative motion (example rotating Earth and its movement around the Sun and the Galaxy, meanwhile the fields must have an absolute motion?. If so then why should the motion or disturbance be created in the conductor for example manually to get this emf?--Email4mobile (talk) 23:24, 8 May 2009 (UTC)[reply]

Carbon Dioxide and Dry Ice

Is carbon dioxide inert and is Carbon Dioxide in its solid form, Dry Ice inert? THX —Preceding unsigned comment added by Jzeilhofer (talkcontribs) 19:03, 8 May 2009 (UTC)[reply]

"Inert" is always relative:) Carbon dioxide is a common material used to extinguish fires (the fire extinguishers with a black plastic cone-shaped nozzle, for example), and they produce both the gas and solid forms. But burning magnesium reacts (quite spectacularly sometimes) with it. It dissolves in water (carbonated beverages) and dissolves in and/or binds to many biological components (carried by blood). And plants consume and chemically react CO2 (photosynthesis). Solid vs liquid is still the same chemical, just colder and "more of it" present in the same measured volume. So dry ice reacts more slowly (many reactions go slower at lower temperature) but for reactions that do occur. DMacks (talk) 19:10, 8 May 2009 (UTC)[reply]
Definitely not inert by a simple chemistry perspective, in which Oxygen is reactive and Nitrogen is inert (even if many reactions of N2 are known, they are relatively few). User:DMacks mentions Mg, but CO2 should never be used on any metal fires, due to its reactivity. It also reacts with water, forming carbonic acid, and is involved in many organic chemistry reactions (carboxylations). It will also act as a ligand in metal complexes, and can be reduced. In most lab conditions, it sublimes quickly, so while the solid form is easier to store, in terms of (non-cryonic) chemistry it is equally reactive. YobMod 11:45, 14 May 2009 (UTC)[reply]

Aluminium in tap water, tea, vegetables, and foods?

How much of the aluminium/aluminum in tea is due to the aluminium in the tap water from which it is made? And are there any vegetables which have significantly more aluminium in them than other vegetables? What about other foods? 89.240.209.79 (talk) 21:21, 8 May 2009 (UTC)[reply]

I'm not sure of tea but this site (http://www.eatwell.gov.uk/healthissues/factsbehindissues/aluminium/) has some info - it seems to suggest that it is from the water and soil. On the basis of water-based i'd guess that more water-y veg (e.g. Cucumber) would have more aluminium than 'drier' veg. ny156uk (talk) 21:30, 8 May 2009 (UTC)[reply]


May 9

Cockatiel Blues

What do you guys think of this? Is this Cocktatiel really performing a spontaneous blues improvisation? --Kurt Shaped Box (talk) 00:29, 9 May 2009 (UTC)[reply]

No doubt the bird has heard that tune played 546 times and learned to imitate it. Along the same lines, have you seen Snowball (Cockatoo)? Looie496 (talk) 02:17, 9 May 2009 (UTC)[reply]
Yep, I'm certainly aware of Snowball and others like him. I actually started that article... ;) It seems that parrots really do have some appreciation of human music... --Kurt Shaped Box (talk) 16:20, 10 May 2009 (UTC)[reply]

Thought Experiment: Perpetual Energy

Of all the systems in a car, I believe the brakes are the most amazing. They will reliaby work just the same with no gas and a dead battery. With a minor press on a pedal, 2000 lbs of automobile will come careening to a dead stop from 100 mph in just a few dozen feet. I had no idea my foot was that strong!

I understand that liquids can't be compressed and that is the "magic" of hydraulics. My question is: what would happen if that same effortless press on a pedal was used to power up turbines instead of stopping a giant machine barrelling at ungodly speeds? Surely the energy produced would be able to overtake the effortless pedal press with more to spare! Sappysap (talk) 01:55, 9 May 2009 (UTC)[reply]

You seem to be confused about either the first or the second law of thermodynamics. The first says that energy is conserved. The kinetic energy of the car doesn't just disappear, neither does it go into your foot. The brakes convert the kinetic energy into heat (for example by pressing a disk against the wheel and letting friction slow it down). See File:Ceramic brakes.jpg for a striking visual manifestation of this heat.
The second law says that entropy always increases, or in less precise and more practical terms, energy can't be converted back from heat. This is why it's impossible to make a brake go in reverse and spin up a turbine rather than slowing something down. Braking a car is an inherently irreversible process, because the entropy increases so much. —Keenan Pepper 02:55, 9 May 2009 (UTC)[reply]
And just so we're clear, hydraulic brakes aren't fundamentally different from any other brake system. Ultimately, they also convert kinetic energy into heat by means of friction. The hydraulics simply transfer the kinetic energy before it is turned into heat. —Keenan Pepper 03:00, 9 May 2009 (UTC)[reply]
(Edit conflict) You seem to have confused two separate sets of energy. Your "effortless press" uses a small amount of energy generated in the cells of your leg muscles. This merely moves the brake pedal, which through the medium of the hydraulics (perhaps supplemented by servo-motors powered by your car's engine or battery) moves parts of the mechanisms of the Disk brake on each braked wheel, pressing the brake pads against the brake disks. The energy required is small, and is independent of whether your car is moving or not.
Your moving car, as you are aware, possesses a large quantity of kinetic energy which has been converted from the stored chemical energy of its fuel by its engine. By your lightly pressing the brake pads against the wheels, this kinetic energy is converted by friction into heat: no additional energy is generated by your operation of the brakes. This is not the same as somehow exerting a new large force opposite to the car's motion in order to stop it, which would indeed require an amount of new energy comparable to the car's kinetic energy (though somewhat less as road friction, internal mechanical friction and air drag are already helping to oppose the car's motion). Ask yourself, since energy can neither be created nor destroyed, only converted from one form to another, where would this new energy have come from?
This form of braking is called Dynamic braking, and the dissipated heat energy (which started as chemical energy in the fuel you paid for) is lost/wasted. In some recent vehicles, some of the energy converted by the brakes in order to slow the car is not dissipated as heat, but converted into a storable form, usually mechanical (for example by spinning up a flywheel) or electrical (in the existing or a supplementary battery), for re-use. This type of brake is called a Regenerative brake, but it too is not somehow creating additional energy, merely saving some of the energy that would otherwise have been lost.
My explanation has been broadly conceptual, but I expect another responder will set out the relevant physical equations in mathematical form, if that will help. 87.81.230.195 (talk) 03:12, 9 May 2009 (UTC)[reply]
The reason you can produce so much force in the brake pads with a fairly light touch of your foot is because your foot moves downwards several inches - but the pads only have to move inwards by a tiny fraction of an inch. That provides the 'mechanical advantage' you need to exert that kind of force...kinda like leverage...but with a liquid. Of course, many cars also have power-assisted brakes - and in that case, power (sometimes from the vacuum system of the engine, sometimes via a hydraulic pump attached driven from the serpentine belt - but increasingly, electrical) is applied to help your foot do the work.
As others have pointed out, the energy from the cars motion gets converted into heat and is dissipated by the disk (or drum) in the braking system. After a lot of heavy braking, the disks can get so hot they they'll actually glow - and if they get too hot, the hydraulic brake fluid might boil - this is called "brake fade" and it's exceedingly dangerous because the gasses produced by the boiling brake fluid is easily compressible (unlike the liquid which is impossible to compress) - so if the brakes get too hot, when you push down on the pedal, all that happens is that the gasses compress, the brake pads stop moving and quite suddenly you have no brakes!
That's why on long hills, you should slow down using the engine (down-shifting the gearbox) rather than using the brakes for large amounts of time. If you do have brake fade, then in an emergency, you can gently apply the parking brake - which is typically connected using a cable system that won't suffer from the heating problems. But you have to be pretty careful because you can easily lock up the wheels and put the car into a skid. SteveBaker (talk) 03:30, 9 May 2009 (UTC)[reply]

will ... what you are'nt understanding is that work is constant , so that brakes just applied the work you've done in avery

effective way on the wheels , its just like hydrolic machines , belozers , were the engine do more than 3000 rpm will its juut

make the vhicle to left the bocket up less than a meter . its just transfere like 40000 rpm to left the bocket up ward like

a meter. thats it .

you can explain it by imagining the multi speed bicycle , in some speeds you have to cycle you legs two times to revolve the

wheel once , which is easy ... in other cases you need to cycle one time to revolve the wheel 5 times which is too hard ,its

need mush bower than your legs can produce .--Mjaafreh2008 (talk) 10:52, 9 May 2009 (UTC)[reply]

MSUD in Adults??

Can an adult have MSUD. Can it be contracted in teen or adult years. What else might be causing this Maple Syrup smell in my sons room and bathroom?? —Preceding unsigned comment added by 68.195.147.98 (talk) 03:03, 9 May 2009 (UTC)[reply]

Sorry, the Reference Desk doesn't give out medical advice. Please consult a doctor. Tempshill (talk) 03:12, 9 May 2009 (UTC)[reply]
The second half of the question is not a request for medical advice, though. There are many things that could be causing maple syrup smell in the room - maybe a stash of hidden pancakes? Nimur (talk) 04:32, 9 May 2009 (UTC)[reply]
I'm with Nimur on this one. The substance that is responsible for maple syrup odor - both in pancakes and in MSUD - is sotolone, and, naturally, we have an article about it. --Dr Dima (talk) 07:48, 9 May 2009 (UTC)[reply]

Earthquakes

Is it possible to find the centre of an earthquake? —Preceding unsigned comment added by 174.6.144.211 (talk) 03:14, 9 May 2009 (UTC)[reply]

Yes - they can measure the arrival time of the seismic waves at remote seismology stations and by comparing those times, figure out where the earthquake was happening. Those shock waves move pretty fast but with accurate clocks, it can be done to a reasonable precision. SteveBaker (talk) 03:17, 9 May 2009 (UTC)[reply]
Well, to the extent that there is a well-defined "center". Earthquakes, especially big ones, don't happen at a single point. There's a section of the fault that ruptures. So it's more like it happens along a line (in the non-mathematician's use of the word line — for mathematicians that would be a one-dimensional manifold, possibly with boundary :-). --Trovatore (talk) 03:41, 9 May 2009 (UTC)[reply]
Actually, earthquakes happen (mostly) along a surface, (a two-dimensional manifold, possibly with boundaries.) And the motions vary with time, so we need to ass a third dimension. A classic example would be an earthquake that progresses along a fault. -Arch dude (talk) 07:34, 9 May 2009 (UTC)[reply]
Also see Epicenter. 71.236.24.129 (talk) 08:59, 9 May 2009 (UTC)[reply]
The epicentre, as mentioned above which is where the rupture begins. It then propagates within the fault surface until it runs out of stored elastic strain energy. In many cases the epicentre is distinctly offcentre for the rupture e.g. both the 2004 Indian Ocean earthquake and the 2008 Sichuan earthquake initiated right at one end of the fault segment that moved. We know this because of the distribution of aftershocks, which define pretty exactly the extent of the rupture. In terms of the felt intensity the area with greatest damage is often near the epicentre and for historic earthquakes, that happened before instrumental recordings, this is how the epicentre is estimated. In some earthquakes. however, such as the 2002 Denali earthquake the maximum intensity was felt at the other end of the fault surface that ruptured from the epicentre. That earthquake also illustrates another possible complexity, it began on a thrust fault jumped to the strike-slip Denali Fault before jumping again onto the Totschunda fault, another strike-slip structure. Finally, most earthquakes hypocentres (the actual point of initiation) occur within a small range of depths, normally 10-15 km, known as the seismogenic layer. This is the strongest part of the crust, as increased confining pressure makes fracturing progressively more difficult until the temperature rises sufficiently for ductile processes to become important. Most of the rupture propagation, therefore, occurs either laterally or upwards, so if you looked at the area that slipped, the epicentre is often nowhere near the centre of the rupture at all. Mikenorton (talk) 12:29, 9 May 2009 (UTC)[reply]

Are convergent, divergent, collisional, and transform boundaries fault lines? —Preceding unsigned comment added by 174.6.144.211 (talk) 04:20, 10 May 2009 (UTC)[reply]

All types of plate boundaries have faults associated with them, but only for some of them is there a single fault surface along the boundary itself. Subduction zones pretty much act as simple fault surfaces, as do transforms in oceanic crust. Some continental transforms such as the Alpine Fault in New Zealand, form single fault surfaces along the boundary but others, such as the San Andreas Fault form part of a deformed zone within which there are many active faults. Divergent boundaries in continental crust form rifts with many active normal faults, such as the East African Rift. Collisional zones such as the Himalayas have huge zones of deformation, sometimes 100s of km across, within which there are many seismically active fault zones. Mikenorton (talk) 11:51, 10 May 2009 (UTC)[reply]

I know that it is possible to calculate the depth of a hypocenter by using p waves, but is it possible to calculate the location of a hypocenter? —Preceding unsigned comment added by 174.6.144.211 (talk) 23:31, 10 May 2009 (UTC)[reply]

Electric circuit for highpower sawtooth wave output.

I need a sawtooth wave generator and i've searched the internet for it; but all the circuit i found have very low current output.The output i need is shown in fig.

File:Output Sawtooth.JPG

Where can i find my desired circuit diagram? —Preceding unsigned comment added by Shamiul (talkcontribs) 04:06, 9 May 2009 (UTC)[reply]

You're going to need a pretty hefty amplifier. Have you considered a PA system connected to a signal generator? You could take the output off the speaker-lines and connect it to whatever your load is. My back-of-the-envelope calculations are suggesting that this is going to need an average power output of around 600 watts, which is awfully big for an audio system. Are you really sure you need 25 amps at 40 volts? Nimur (talk) 04:30, 9 May 2009 (UTC)[reply]

Thank you for your answer, but i don't mean amplifier. The main circuit should deliver the output required that can be directly delivered to the load. I want to know the procedure that you used to calculate the power output, in case if i had made mistakes. —Preceding unsigned comment added by 123.49.45.67 (talk) 04:57, 9 May 2009 (UTC)[reply]

Nimur appears to have arrived at the power requirement by a standard RMS integration of the voltage waveform multiplied by 25 amps. However, in his audio amp solution the 5 volt offset would have to be added back after the amp because audio amps do not usually pass dc, the required power rating of the amp is therefore a little less. You do not say why this is not an acceptable solution, and without details of your application it is difficult to give you precise help. Possibly bandwidth is a concern, 10kHz would certainly go through a high quality audio amp but the harmonics are likely to get mangled, badly distorting the waveform from linear. There are certainly plenty of audio amps out there in the right power range at affordable prices. In any case, any solution is liable to be a low power waveform generator stage plus a power amplifier stage as Nimur says, although the power stage might be just a power transistor working off a 40 volt rail if your waveform generator has an open collector output. I think you will struggle to find a ready made circuit diagram to do this - high voltage ramp circuits were common in the days of crt televisions, but I can't think offhand of many high current applications. So to give you some specific suggestions: for the waveform generator there is the 555 timer which has been around since the stone age and the internet is littered with circuit diagrams and application notes. Here is a page describing some alternative waveform generator ICs. If you need great precision of the waveform then you might want to consider a synthesised arbitrary waveform generator which are available as modules/pcbs from some manufactures. For the output transistor, there are many available with the required current and voltage rating. SpinningSpark 10:37, 9 May 2009 (UTC)[reply]
Yes, that's how I got it... I think the original questioner should really provide context. When you say you don't want an amplifier, I'm a little worried - because any real circuit which will generate such high voltages and currents IS an amplifier. If you're a novice electronics enthusiast, you should not be playing with kilowatt-scale, high-amperage systems, because they can kill you. (Let's be clear here - 40 volts and 25 amps is deadly). What are you trying to do? With some context we can give you a better and more realistic answer. Nimur (talk) 15:05, 9 May 2009 (UTC)[reply]
By the way, when you asked the same question last year your image was deleted for the same reason this one's going to be if you don't put a licence tag on it soon. SpinningSpark 20:35, 9 May 2009 (UTC)[reply]
As a retro alternative circuit, how about a 5 volt DC generator in series with a DC generator, the field current of which is regulated by a motor driven rheostat such that it sweeps through the required output voltage? A 600 watt generator is not all that large or expensive. Edison (talk) 01:40, 10 May 2009 (UTC)[reply]
That wouldn't be very energy-efficient! Half the power would be burned over the rheostat! You really shouldn't be encouraging resistive-loss as an effective method for generating a desired voltage. This is especially bad if you're really going to pump the output signal at 25 amps through it! That sucker's going to need a heat sink! Nimur (talk) 16:15, 10 May 2009 (UTC)[reply]
Is your first name Heath? SpinningSpark 02:15, 10 May 2009 (UTC)[reply]

Downshifting vs. braking

Inspired by a SteveBaker answer above: Is it really better to downshift than brake when going down a hill? Won't this cause excess wear on the transmission or gearbox or some other part of the car that, unlike brakes, was not designed to be worn so much? Tempshill (talk) 04:10, 9 May 2009 (UTC)[reply]

Hmm? I don't see where there's any extra stress on the drivetrain- it's the same as what's going on most of the time while you're driving. Brakes slow you down by friction- the pads wear out as you use them. And they're emphatically not meant to be used constantly for long periods of time, because they'll heat up and fail (see brake fade). The drivetrain is meant to be turning pretty much all the time. Friday (talk) 04:15, 9 May 2009 (UTC)[reply]
When I lived in Colorado and Montana I did a lot of mountain driving. I could always tell who the flat-landers were by the excessive amount of brake lights they displayed on the down grade. --jwalling (talk) 04:41, 9 May 2009 (UTC)[reply]
In Colorado and Montana you'd also likely have encountered quite a few people driving cars with automatic transmissions. They are a lot more common here than in Europe. 71.236.24.129 (talk) 08:51, 9 May 2009 (UTC)[reply]
Not exactly the same, but maybe you're interested in jake brake. Shadowjams (talk) 09:02, 9 May 2009 (UTC)[reply]

i think gears were'nt designed to slow the vhicle , because at high speeds its not right to shift back from the fourth to the

first , and in some cases it could break the sustem . i think its better to use brakes instead except when its dangrous to do

so at high speeds or at mountain roads were such action could cause the car to go off the road .--Mjaafreh2008 (talk) 09:49, 9 May 2009 (UTC)[reply]

I live in a mountainous country, where we are taught to always downshift when driving downhill. The article Måbødalen disaster may be of interest. It was in part caused by the bus driver's lack of experience in mountain driving. --NorwegianBlue talk 11:40, 9 May 2009 (UTC)[reply]
But you aren't trying to slow the vehicle, you're trying to prevent the vehicle speeding up. That's a very different thing. If you want to slow down, use the brakes, if you want to maintain a constant speed, put the car in the right gear for that speed (which will be a lower gear when going downhill than when on the level). --Tango (talk) 13:27, 9 May 2009 (UTC)[reply]
Let's be PERFECTLY clear about this. For routine slowing down - brief dabs on the brakes separated by minutes of time, typical in-town driving - then you most certainly should use the brakes. That's what they are there for. However, if you are going to have your foot on and off the brake - or lightly pressing the brake for (let's say) a minute or more - then you're driving extremely dangerously and sooner or later you'll boil your brake fluid and you could very easily DIE as a result of your failure to understand the principles of controlling your vehicle. This most often happens on long downhill sections (but it can happen if you're driving agressively on the freeway at high speeds). In these circumstances, you MUST use the engine to slow yourself down. On a stick-shift, you need to shift down progressively (ignoring the horrific racing noises from the engine!) until your speed is under control - where you want it to be. The engine and transmission most certainly are designed to be able to do this - you aren't damaging it any more than when you stamp on the gas pedal to accelerate away from a stop. On an automatic, taking your foot right off of the gas pedal may be enough - but if you're still accelerating then you need to use the brakes one time to get your speed down - and then put the transmission into '2' or even '1' for the remainder of the hill. People who worry that they are ruining their transmission or engine need to understand that they are ruining their brakes by NOT doing this - heating their brake disks/drums until they are literally glowing red hot is going to cause them to warp and to wear the brake pads prematurely. If you do experience brake fade, the seals on brake slave cylinder can be ruined and you can get all sorts of crud in your brake lines. I can't overstress the importance of this. It's one of the commonest mistakes people make when they aren't used to driving in hilly terrain. Failure to understand how brake fade comes about kills a good number of people every year - you should have paid more attention during driver's ed. classes! SteveBaker (talk) 14:13, 9 May 2009 (UTC)[reply]
Well, this is a bit exaggerated. It's pretty hard to lose your brakes in a modern, ordinary-sized passenger car, unless they are in bad shape to start with. The danger is only serious for heavy vehicles like trucks or buses, or if you do something really stupid like keeping your foot on the brake while pressing the accelerator. It's definitely better to use engine-braking as much as possible, but you don't have to be paranoid about it. I've numerous times descended steep switchbacky mountain roads where I had to brake the whole way down for thousands of feet -- being in low gear helped of course but only to a limited degree. Looie496 (talk) 17:21, 9 May 2009 (UTC)[reply]
[citation needed]? Please quote sources, and re-read Steve's response and the article I linked to. --NorwegianBlue talk 19:11, 9 May 2009 (UTC)[reply]
If you were in first and still going too fast then, of course, you have to use the brake, you have no choice, but you were still risking your brakes overheating. --Tango (talk) 20:19, 9 May 2009 (UTC)[reply]
You sure have a choice. Try shifting to reverse. --NorwegianBlue talk 20:36, 9 May 2009 (UTC)[reply]
Ok, you don't have a good choice! --Tango (talk) 21:50, 9 May 2009 (UTC) [reply]
I think we are confusing two different issues: The use of either brakes or the transmission to slow down the car versus the use of either brakes or the transmission to maintain speed on a hill. The brakes should always be used to slow the car down, but the transmission should be used to maintain the proper speed when going down hill. That's because brakes are not designed to be applied for long periods of time, but in short bursts. --Jayron32.talk.contribs 04:36, 10 May 2009 (UTC)[reply]
Why do you advise against taking advantage of the engine's ability of slowing down the vehicle, say, when leaving a motorway? Used with proper technique, you can drive very smoothly that way. Combined, of course, with using the brakes. --NorwegianBlue talk 10:44, 10 May 2009 (UTC)[reply]
Generally - because brake pads are cheaper to replace than clutches, you should use the brake for short decelerations. But if you aren't concerned about wear and tear, then either approach works and is safe for brief periods. I confess that I use engine braking more than I probably ought. The big issue is only with prolonged use of the brakes - which is just plain dangerous. SteveBaker (talk) 14:20, 10 May 2009 (UTC)[reply]
Steve, do you have a reliable source for this advice? Thanks. Axl ¤ [Talk] 06:28, 13 May 2009 (UTC)[reply]

Climate change vs. Global warming

Are there simple guidelines to determine which term should be used in a discussion. In my mind, Global warming is a major cause for the effects of Climate change on regional scales. It's my impression that 'climate change' is used often when 'global warming' would be more accurate. --jwalling (talk) 04:27, 9 May 2009 (UTC)[reply]

Global warming is one form of climate change (i.e a subset). It's also possible to have global cooling and other forms of climate change. We are currently experiencing massive human induced climate change in the form of global warming which is expect to get worse but obviously when you refer to global warming it doesn't have to refer to this particular instance nor does climate change have to mean global warming. Nil Einne (talk) 04:37, 9 May 2009 (UTC)[reply]
The ambiguity of 'climate change' (warming or cooling) supports my view that 'global warming' is more accurate when discussing the current global climate. 'Climate change' is appropriate for an unspecified region or uncertain period of time, but if you do specify a region and a time period which is showing warming, such as the Arctic for the next 10 years, then 'Climate warming' is more accurate. -- jwalling (talk) 05:35, 9 May 2009 (UTC)[reply]
Well firstly I'm presuming you've already specified on the planet earth and over the past 100 or so years and the next 100 or so years. Otherwise it's still confusing (are we referring to global warming on Venus? 1 billion years ago? 1 billion years from now). But even then arguably anthropogenic global warming (a favourite of denialists) or human-induced global warming is more accurate (or more correctly more precise since global warming is by definition a form of climate change so it's very rare you can be wrong if you specify climate change instead of global warming, it's just that you are not being precise as to what form of climate change you are talking about) since it is what we are observing (not say global warming due to sunspots). If you want to go further, perhaps global warming due primarily to an increase in global greenhouse gas levels as a result of human activities. Even further global warming due primarily to an increase in global greenhouse gas levels as a result of human activities including extensive use of fossil fuels, large scale changes in land use...... I think to some extent both are precise enough. There is only one form of massive global climate change we are currently observing and expect as far as I'm aware and that is global warming. So if you want to talk about climate change from a global sense in the next 100 years then of course that will primarily be about global warming not global cooling or anything else. It also depends on what you're talking about. For example the scientific opinion on climate change is about all climate change we expect in coming years and as I've mentioned this is global warming. Similarly the Intergovernmental Panel on Climate Change is supposed to study all climate change and of course global warming is of primary interest to them. In other words, IMHO it's more a matter of semantics and trying to argue which one is more accurate or perhaps IMHO since as I've already stated more accurate is a bit of a misnomer, which term is better applied to a given situation is mostly a pointless waste of time outside of times where it matters e.g. the naming of wikipedia articles, it's better to concentrate on more important things liking convincing those who are fooled by the denialists that there isn't one big conspiracy and climate change in the form of global warming is happening and is going to cause major problems and yes it is caused primarily by human activity. Nil Einne (talk) 08:46, 9 May 2009 (UTC)[reply]
The problem with the terminology is that the increase in greenhouse gasses is causing global warming - but also other kinds of climate change (increased hurricane strength and/or frequency, rising sea levels, changes in drought patterns, etc). So the term "Climate Change" better suits the whole range of problems that are being caused. However, some of these effects are secondary effects of global warming - without global warming, most (if not all) of the other effects wouldn't be happening. However, there is at least ONE other effect that we can describe as "Climate Change" which is totally unrelated to global warming or the greenhouse effect - and that is the destruction of the ozone layer. That was not caused by greenhouse gasses like CO2 and Methane - but instead by Chloroflourocarbons released into the atmosphere. Destruction of the ozone layer causes increases in ultraviolet radiation - which is harmful in all sorts of ways. Worrying about the ozone layer has become less trendy than concern over the greenhouse effect - but it's also a major problem. I think it's reasonable to say then that the term "Climate change" is more encompassing than "Global warming" which is in turn more encompassing than "The Greenhouse effect". So, although I confess to tending to use the terms interchangeably - if we are being super-careful:
  • If you are talking about CO2, Methane and other gasses that we're dumping into the upper atmosphere - then you need to talk about "The Greenhouse Effect".
  • If you are talking about organohalons, chloroflourocarbons, nitric oxide and related gasses causing destruction of ozone - then "Ozone layer depletion" is the correct term.
  • If you are talking about gasses dumped into the lower atmosphere that cause plant destruction and soil pollution as they are washed out of the atmosphere by rain - then you need to talk about "Acid Rain".
  • Any or all of the previous three terms could be described as "Atmospheric pollution".
  • If you are talking just about the rise in globally and seasonally averaged air temperatures - then you should say "Global Warming" - because there are other causes (and mitigating factors) for that beyond the greenhouse effect - things such as the decrease in the planets' albedo due to melting ice caps and glaciers and the increase in albedo due to the contrails from high flying jet aircraft.
  • If you are talking about all of the consequences of all of the things we're doing to the planet's atmosphere and hydrosphere - then "Climate Change" is a more appropriate term - because there are other causes of that - such as changes in water usage patterns and evaporation rates, destruction of the Ozone layer, the effect of large cities creating local hot-spots, etc.
  • If your concerns range beyond that, then "Oh Shit!" may be the term of last resort!
SteveBaker (talk) 13:45, 9 May 2009 (UTC)[reply]

marine diesel engine

working of air distributar Bose09 (talk) 07:10, 9 May 2009 (UTC)[reply]

Diesel engine has some information under medium-speed engines and low-speed engines but it is a bit scanty I'm afraid. SpinningSpark 11:08, 9 May 2009 (UTC)[reply]

Kc calculation

"The equilibrium constant, Kc, for the reaction to form ethyl ethanoate from ethanol and ethanoic acid, C2H5OH + CH3CO2H=CH3CO2C2H5 + H2O, at 60 degree Celsius is 4.00. When 1.00 mol each of ethanol and ethanoic acid are allowed to reach equilibrium at 60 degree Celsius, what is the number of moles of ethyl ethanoate formed?" This is the question I have to solve. I know you guys dont do homework questions and I dont want the answer to the question either. I just want to know how I am supposed to solve this question, i.e the working. Thanks. —Preceding unsigned comment added by 116.71.59.203 (talk) 08:38, 9 May 2009 (UTC)[reply]

Use the first formula given here. yandman 11:50, 9 May 2009 (UTC)[reply]

But the question doesn't give the amount of water produced. I cant use that formula. —Preceding unsigned comment added by 116.71.59.203 (talk) 11:57, 9 May 2009 (UTC)[reply]

You don't need to be given that...you need to figure it out (at least algebraically) from the info that is given. You're not told "how much ethanol is present at equilibrium" but "how much ethanol is starting before equilibrium has occurred". The amount of reactants are decreased by the identical (considering a stoichiometry of a balanced reaction) that the amount of products increases. I am sure you did examples like this in class or in your textbook. DMacks (talk) 17:37, 9 May 2009 (UTC)[reply]

So by "figuring it out", Id say that the amount of water is also 1 mole? I haven't really understood this part. —Preceding unsigned comment added by 116.71.33.115 (talk) 05:10, 10 May 2009 (UTC)[reply]

Maybe and maybe not. You start with 1 mole of starting materials and 0 mole of products (per question as written). As you can see in the reaction, every bit of water that is formed comes from ethanol, so if the full 1 mole of starting material is consumed, you get 1 mole of product, and that would leave leave you with 0 mole of starting material. The whole idea of equlibrium and Kc is that the reaction doesn't run all the way (which would consume all of the starting material) but only runs partway. "Some" starting material gets converted to product, "the rest" remains as starting material. And Kc defines exactly the ratio of those two quantities. You know the total (perhaps SM+P?) and the ratio (formula for Kc and its given value), now solve for the variables. DMacks (talk) 08:07, 10 May 2009 (UTC)[reply]

teeth

I have a very weak teeth , i visit alot of dentist by they were'nt alot of help

now after all that advance in evry field , i find it wierd that they had'nt find

asolution for cavity , its brrety simple .

some kind of amouth wash that will produce a thin film at the surface of the tooth

so it wont be in contact with food left , so no cavity will form .

realy i dont think its that hard , we built a space ship , rockets , submarines , airplanes

i think its aprrety easy task .

i said that because i visit the dentist so much , and its frightning each ti.

--Mjaafreh2008 (talk) 09:37, 9 May 2009 (UTC)[reply]

Okay. Do you have a question? -- Captain Disdain (talk) 10:26, 9 May 2009 (UTC)[reply]


Answers removed, after discussion on the talk page.

Mjaafreh2008, as Captain Disdain pointed out, you aren't really asking a question. Please rephrase your question, keeping in mind that we cannot give medical advice. --NorwegianBlue talk 16:55, 11 May 2009 (UTC)[reply]

Class of drug

Is Clopidogrel an Anticoagulant? If not what's the difference? (No medical advice.)71.236.24.129 (talk) 11:02, 9 May 2009 (UTC)[reply]

In the ATC classification system, clopidogrel has the ATC number B01A C04, and is classified as an antithrombotic agent. This ATC group has its own Wikipedia page: ATC code B01. If you check it out, you'll find that a traditional anticoagulant such as Warfarin is in the same group. The difference is that clopidogrel inibits platelet aggregation, whereas anticoagulants inhibit the proteins necessary for coagulation. --NorwegianBlue talk 11:25, 9 May 2009 (UTC)[reply]
(After EC) The usual definition of an anticoagulant is a substance that stops the blood from clotting. Anticoagulants like heparin or warfarin directly affect the clotting factors and prevent the entire process from getting started. Clopidogrel is an antiplatelet drug, which prevents platelet activation, the process by which platelets form a kind of plug to stop bleeding. The drug therefore prevents aggregation of platelets but does not interfere with the rest of the clotting cascade. It may seem a subtle distinction since the end result of both classes of drugs is to interfere with the formation of blood clots, but there are differences in mechanism and what the various drugs are used for, so it's a meaningful separation. --- Medical geneticist (talk) 11:38, 9 May 2009 (UTC)[reply]
A difference that also may be of interest is in their indications: clopidogrel is typically used to prevent arterial thrombosis after medical procedures such as the insertion of a coronary stent, anticoagulants are typically used to prevent venous thrombosis. --NorwegianBlue talk 12:00, 9 May 2009 (UTC)[reply]
Thanks. It's a bit clearer now. 71.236.24.129 (talk) 14:01, 9 May 2009 (UTC)[reply]

How a factory works

Is there any book/package/work/whatever that will teach you exactly how a factory works? Or if you want to create a factory you must pick up the pieces from here and there? --Mr.K. (talk) 11:53, 9 May 2009 (UTC)[reply]

Please clarify: are you asking about a factory or a Factory pattern? --NorwegianBlue talk 12:04, 9 May 2009 (UTC)[reply]
If it's the latter: A piece of software that "constructs class instances" (all three of those words having very specific meanings) according to some set of parameters - then you REALLY need to ask this question on the computing section of the ref desk.
If you're talking about an actual physical factory...then it's still not got much to do with science and you might get a better answer on the miscellaneous desk where there are more people to respond to the question.
SteveBaker (talk) 13:18, 9 May 2009 (UTC)[reply]
Every factory is different. Some are highly mechanized, and some are just big empty rooms where people work, sewing or bending sheet metal or some other process. If you have a question about a specific type of factory, we can answer that better. Maybe you would find the machining, assembly line, and manufacturing articles helpful. Nimur (talk) 13:39, 9 May 2009 (UTC)[reply]
Most factories are based on a Workflow analysis. So if you are thinking of a brick and mortar version that would be a good starting point. Apart from that we'd really have to know what type of factory you're interested in. There are significant differences between building a meat processing plant, a saw-mill or a coffee machine manufacturing plant (OR). Even restrictions on site selection (where you could/should build it) would be quite different. 71.236.24.129 (talk) 13:58, 9 May 2009 (UTC)[reply]

Yes, every factory is different. I just wanted to know if any of you have ever seen a book/work/or whatever that explained exactly how you could build your own factory of whatever. I don´t want you to describe how a factory works, I just need the bibliographic reference. --Mr.K. (talk) 17:49, 10 May 2009 (UTC)[reply]

When you say 'build', do you mean like construction of the building? Or like design of the inner mechanical workings? Or like making all the machine parts? Or building the workforce and management structure? Or what? Are you thinking start with a plot of greenfield land and end up with a working factory?
A warehouse with workers stood at tables could be a factory, and all you'd need to build that would be the construction skills to make a one-room building (and the workers and tables and supplies and know-how and...) Something to make pharmaceuticals would be very different, which in turn would be completely different from a factory for smallscale cake production which would, in turn, require quite different skills to building a facility to process volatile petrochemicals.
So, are you just looking for a book that describes how to build a simple building? Like, bricklaying skills and suchlike? O_o 80.41.104.79 (talk) 21:12, 10 May 2009 (UTC)[reply]

Net charge of earth

Is the net charge of the earth is +ve ? Why?? —Preceding unsigned comment added by 117.98.99.182 (talk) 13:24, 9 May 2009 (UTC)[reply]

I don't think so. I'm pretty sure that the Earth as a whole entity has neutral charge. There are locally charged regions (a lot of times, charge builds up in the atmosphere); and while we have a radiation belt full of charged particles, there are both electrons and positive ions to balance out. Take a look at this Discussion on the Earth's Net Electric Charge. In this published paper, a scientist from the Navy Research Lab states "As a consequence of this discussion, any statement made up to now on a net electric charge of the earth remains in doubt. It may be that the net electric charge of the earth (as globe or as planet) is zero or near zero - always or in the average - but we do not know a way to prove or disprove it." This paper is a pretty good introduction - it defines the problem well, mentions several experimental and theoretical approaches, and describes the various difficulties in all these methods. Nimur (talk) 14:04, 9 May 2009 (UTC)[reply]
Hi Nimur, I saw that paper but cannot read all of it. Does it debunk the common statement that the surface of the earth has a high negative charge (balanced by positive charge in the upper atmosphere/ionosphere)?. Or is it merely saying that past attempts to enumerate this charge precisely are misguided? SpinningSpark 14:25, 9 May 2009 (UTC)[reply]
The author refines the question by asking whether the question is "net charge on the earth, volumetrically", "net charge on the earth's surface, integrated over the entire surface," and "net charge on the surface in a local sense". Without doubt there is a net charge on the earth's surface in a local and transient sense, and this is generally believed to be nulled out by atmospheric electric charge, sometimes re-equalizing with lightning. The author discusses the "spherical capacitor" model, with a charged surface (lithosphere) and a charged ionosphere boundary, but investigates whether that can really apply to a surface that is 70% oceanic. Further, there are discussions about net current, vs. net charge, and whether one can exist without the other. There is discussion about Earth's space environment (being a conductive plasma, any net charge on Earth would be able to flow off into space and equalize to zero). In each of these scenarios, the author provides the essential basic-physics approximation, and attempts to derive some consequences, before presenting conflicting experimental evidence to suggest that the basic-approximations do not hold well. His concluding remarks are that there are no practical methods to measure a net-charge on Earth, because of the various confounding factors; and that the theoretical explanations give various answers ranging from "large positive charge", "neutral," "large negative charge", "dynamic and fluctuating with zero-mean," yet none of these theories are practical to verify experimentally. Nimur (talk) 14:43, 9 May 2009 (UTC)[reply]
In specific answer to your question, SpinningSpark, he does not "debunk" the common statement about charge-splitting between lithosphere and troposphere/ionosphere. (As I mentioned above, this explanation is widely understood as the basis for a lot of tropospheric weather, and has been clearly observed in a variety of experimental setups). However, he does make two counterpoints regarding the extension of this model to the entire globe. First, characterizing the magnitude is much more complicated than it might seem (my experience in this area strongly supports such a stance). Second, the capacitor-model of charge-splitting may not be applicable over the entire earth's surface, (certainly not "instantaneously", and probably not even in the time-average-sense). Nimur (talk) 14:46, 9 May 2009 (UTC)[reply]
A 'Science Advisor' on physicsforums.com says that Earth is "neutral to about one part in 10^26", but I can't see where he gets that figure from. As mentioned above, the Earth is bathed in a plasma that would conduct away any excess charge. Perhaps the next question should be "What is the net charge of the Solar System?" --Heron (talk) 17:55, 9 May 2009 (UTC)[reply]

Polynomial in several variables

Somehow I stumbled on the polynomial . Now I wonder what that is, that's to say, does this polynomial occur naturally in some context of math or other science? 93.132.150.3 (talk) 13:50, 9 May 2009 (UTC)[reply]

It doesn't look particularly useful to me, and I don't think it's a polynomial it's represented in a way that doesn't look very polynomial-ish. It appears to be some kind of norm function (though I don't think it satisfies some of the properties). It looks like it takes two vectors of length n, and combines them to give a single scalar value. Did you notice, though, that if any value of x is equal to any value of y, that the result is zero? Maybe that's a feature, or maybe it's a bug... Nimur (talk) 14:00, 9 May 2009 (UTC)[reply]
You don't just stumble on something like that as if you found it lying in the street, tell us what page it is on or otherwise give context. SpinningSpark 14:36, 9 May 2009 (UTC)[reply]
(After ec) I first mistook for some kind of volume, which it is clearly not, at least not the common kind. In the context I found it, the variables are free symbolic variables, so I didn't think of it evaluating to zero ... but yes, it's a feature, not a bug. I deliberately did not mention any details on the domain of the variables so I wouldn't miss any suggestions for other kind of variables. 93.132.150.3 (talk) 14:38, 9 May 2009 (UTC)[reply]
Take a look at Cauchy determinant SpinningSpark 14:48, 9 May 2009 (UTC)[reply]
Great, that's the polynomial! The way I came on it was that I had a look at signature of permutations where the second proof uses a similar polynomial and I was wondering if the somehow arbitrary restriction to i<j could be avoided. Unfortunately, if you use the above polynomial for that you get the square of the signature, which makes it not very useful for that purpose. 93.132.150.3 (talk) 15:07, 9 May 2009 (UTC)[reply]
It is also similar to the Resultant. 81.98.38.48 (talk) 11:11, 13 May 2009 (UTC)[reply]

cloud formation

I have searched google and wikipedia and could not find the answer to this question. What attracts the water vapor/ice particles together to form clouds? Ionic or static charges? The wikipedia page for cloud formation simply says "When surrounded by billions of other droplets or crystals they become visible as clouds".

Is the attractive force between these billions of particles simply a natural phenomenon and no one knows why, or is there a commonly accepted answer?

thanks for your help,

HarveyHarveyalton (talk) 15:08, 9 May 2009 (UTC)[reply]

For clouds to form, two things must happen, the air must be below the dew point and there must be seeds (eg dust) around which water vapour can nucleate. This can happen with rising warm air, as the air rises in the atmosphere the pressure falls which causes the dew point to fall. If the pressure falls far enough, the air becomes saturated (ie relative humidity is 100%) and water droplets start to form (if there are cloud condensation nuclei present). Other processes can cause air to rise and trigger cloud formation. Amongst these are the collision of warm and cold regions of air in frontal systems and orographic lift. SpinningSpark 15:24, 9 May 2009 (UTC)[reply]
Just a few corrections: the air need not be below the dew point, but only equal to it, for a cloud to form. This is because the dew point, by definition, is the temperature at which the air is saturated with water vapor. Also, it is not the pressure fall which causes rising air to become saturated, but the temperature change due to adiabatic cooling. But these are minor quibbles.
I think the OP misunderstands the concept of a cloud: The cloud particles are not physically bound to each other, they merely reside in the same area. Cloud droplets appear together (as a cloud) because air temperature can not change rapidly in a short area without equalizing itself quickly. Therefore, you won't have just small areas here and there where the temperature is at the dew point, but rather a (relatively) large area on the order of a few hundred or thousand feet. And since the atmosphere has no shortage of cloud condensation nuclei, almost any area at the dew point will also have cloud droplets, and thus a cloud.-RunningOnBrains 16:14, 9 May 2009 (UTC)[reply]

Pur water filter.

We have one of those Pur water filter gadgets that bolts onto the kitchen tap. It has an exchangeable charcoal filter cannister and a little battery-operated gizmo with a red/green LED.

The LED lights green when you run water through it - and the light turns red when the cannister needs changing.

Does anyone know what the sensor is actually measuring? I kinda suspect it's just measuring flow rate and integrating that to figure out the amount of water that's run through the filter...but I'd like to know for sure.

SteveBaker (talk) 16:07, 9 May 2009 (UTC)[reply]

I used to work in a laboratory that used de-ionised water in industrial quantities. We checked for when the filter chemicals needed changing by measuring the resistivity of the water. This can only be done when the water is actually running - as in your device apparently. So the facts fit but its still only a guess. SpinningSpark 16:27, 9 May 2009 (UTC)[reply]
If you have one of these the promo video on that page (click the cinemachine) says the LED indication is based on amount of water used. SpinningSpark 16:36, 9 May 2009 (UTC)[reply]
The one I'm thinking of is this one (actually, the one on the left of that picture). SteveBaker (talk) 17:38, 9 May 2009 (UTC)[reply]
I'mm not sure resistivity is a usable measure for simple charcoal-filtration. I would expect that lots of ions aren't filterable that way, so the detector would have to be calibrated for the expected presence and amount (fluoridation, hardness, etc.) for your particular supply. Ain't gonna get 10MΏ without reverse-osmosis or at least some well-designed ion-exchange resins. DMacks (talk) 18:55, 9 May 2009 (UTC)[reply]
I agree, there is a big difference between filtered drinking water and laboratory quality de-ionised water. I expect it is simply measuring throughput and knows the life expectancy of the filter cartridge. --Tango (talk) 20:13, 9 May 2009 (UTC)[reply]
That's my guess too - but I have a deeper concern. What cheap flow rate sensor might they be using? These gizmo's are pretty cheap. Is it possible that they just measure the amount of time for which water is present? (I'm kinda skeptical that these things are doing a good job). SteveBaker (talk) 22:39, 9 May 2009 (UTC)[reply]
I'm curious - why do have one of these in the first place? Do you live somewhere with water that doesn't taste very nice? As far as I know, that's the only genuine use to these things. Have you done double blind taste test to check if it is really making a difference? --Tango (talk) 00:31, 10 May 2009 (UTC)[reply]
The water quality here in Round Rock, Texas is generally pretty good - but there are certain times of year when the turbidity of the water goes off the charts and despite assurances that it's safe, it looks disgusting - and the filter takes care of that. I have not done double-blind taste tests - I'm more concerned about the actual, published data from the water utility. But overall, I'd rather we had this than have my wife attempt to destroy the planet by buying bottle water - which would be the sure and certain result of not using it. SteveBaker (talk) 14:03, 10 May 2009 (UTC)[reply]
Steve, I can do the standardized test for drinking water turbidity. Can you collect some water on a day when it is very turbid and send it to me? It would be interesting to see if it exceeds what you water utility is reporting (or it exceeds the established limits for these things. ike9898 (talk) 15:58, 12 May 2009 (UTC)[reply]
My Miele vacuum cleaner shows when its dirt bag is full by a mechanical indicator of the air pressure difference across the bag. I suspect the charcoal water filter is monitored the same way, by the water pressure difference. If so, a red light could change to green when the tap is turned partly off. Cuddlyable3 (talk) 00:13, 10 May 2009 (UTC)[reply]
Oohh, Miele keeps my clothes clean and my whites white! Also the best vacuum cleaner in the world. I'm always suspicious of automatic indicators for replaceable items though - they can be equally easy to calibrate to meet corporate revenue targets. Steve, I'd suggest a call to the customer support desk (and persistence) or appropriate use of a screwdriver. I'd agree though with the comments above that it uses pressure differential, which really has nothing to do with whether or not the charcoal is active anymore - it just tells you when the filter is clogged. If they work on the water mains and send down a blob of black goo, you might still be getting good water, just not very much of it Franamax (talk) 00:56, 10 May 2009 (UTC)[reply]
Since they sell a similar gizmo with an all mechanical indicator I'd be rather surprised if they didn't just use the battery operated LED for the indicator. The rest of the setup is probably the same as for the mechanical thing. I don't want to take ours apart. So, I can't tell you what exactly the mechanism is. My guess would be nothing more fancy than an impeller like thing moved by water flowing through the unit. (No pressure increase like in our article.) It likely turns some wheel and lever setup till it hits the battery contacts. Here's an experiment you can do when your filter is full. Just remove it and put it back in. I guess your LED will return to green. (Our mechanical indicator gets reset when I do this.) 71.236.24.129 (talk) 04:23, 10 May 2009 (UTC)[reply]
This patent describes a monitoring system for such a filter (not necessarily your filter though). It consists of a switch activated by the flow of water and a timing circuit that counts how long water has been flowing across the total lifetime of the filter. Replacement is indicated when the timer reaches a preset value. The switch is a simple on-off, and so doesn't measure actual flow but rather assumes some typical flow rate. Dragons flight (talk) 04:25, 10 May 2009 (UTC)[reply]
Possibly also of interest this patent describes a system for measuring actual flow rate based on pressure difference (and not just on-off). Dragons flight (talk) 04:36, 10 May 2009 (UTC)[reply]

Light bulb

What does it mean to have a 60W, 120V light bulb? I would assume it means that the light bulb transfers energy at a rate of 60W and that the potential lost across the light bulb is 120V. The reason I ask this is because I had the following homework question: Two 60W, 120V light bulbs are connected in series with each other and are in parallel with a 1600W, 120V air conditioning unit. The battery has an emf of 120V. What's the current intensity?

To me, there seems to be a contradiction. Both light bulbs have to lose 120V in total, so they can't be both 120V light bulbs, right? Normally I would have said that the total power of the system = 1720W, which equals E*I. Solving for I, we get 14.3A. The correct solution, however, is 13.59A, which I was able to get by the applying the formula P=V^2/R to each light bulb and the AC unit. But working backwards from this, we get that the potential across the light bulbs is 60V (to be expected, but it's still not 120V) and the power is 15W. Can someone help? Thanks. —Preceding unsigned comment added by 65.92.6.148 (talk) 18:43, 9 May 2009 (UTC)[reply]

You're on the right track and see an interesting problem! The voltage-rating of the bulb is just a maximum allowable, not the definite drop. A passive component such as a resistor would have a voltage-drop of "whatever potential is applied across it" by definition:) But if you put "too much" potential across it, you'll cause more current to flow than it can handle or more power dissipation (i.e., heat) to it can withstand or something like that. DMacks (talk) 18:48, 9 May 2009 (UTC)[reply]
So if I understand you correctly, 60W is also the maximum allowable power, right? But since neither of those (voltage or power) describe this situation, how can we use them to answer the problem? —Preceding unsigned comment added by 65.92.6.148 (talk) 19:15, 9 May 2009 (UTC)[reply]
For your problem think of a bulb (and even the AC!) as just a resistor, and the watt and voltage ratings as an indirect way of defining its resistance. Does that help ? Abecedare (talk) 19:27, 9 May 2009 (UTC)[reply]
(ec)Because knowing the maximum power and voltage allows you to calculate the resistance. Once you have the resistance you can calculate the voltage and power in a new situation. But beware, this only works in homework questions, in real life the resistance changes quite dramatically at different voltages because the temperature of the filament is different. SpinningSpark 19:30, 9 May 2009 (UTC)[reply]
The OP sees a contradiction where there is only this error: A light bulb that is called a "120V light bulb" never has to lose 120V. It just "needs" that voltage to shine at its full strength which is 60W. When two such bulbs are connected in series the applied 120V is split between them so each bulb gets 60V. They shine dimly but are still "120V light bulbs", that's just their maximum rating.
Clearly the homework question was designed to be a simplistic exercise using Ohm's Law for DC circuits and ignores some practical realities Cuddlyable3 (talk) 00:01, 10 May 2009 (UTC)[reply]
An air conditioning unit draws a lot of power. I never heard of one that works from a battery. Even the cooling unit on a refrigerated truck works from a small gasoline en;gine. For a homework question maybe we can accept the imaginary situation, But I think it would be better to stay with real-life situations unless there is a definite reason for going into the reaalm of imagination. Otherwise, the imaginary aspect may confuse the student. —Preceding unsigned comment added by 174.130.249.69 (talk) 21:34, 9 May 2009 (UTC)[reply]
Real life problems require many years of experience and training to solve and developing the solution can take months or even years. Homework on the other hand needs to be solved in 30 minutes max with no more than a calculator. Of necessity the questions are artificially contrived and it is pointless and unnecessary to complain about it. SpinningSpark 00:00, 10 May 2009 (UTC)[reply]
Indeed. Any air-conditioner I know of uses an AC motor, so I'd rather expect the current intensity to be the maximum discharge rate of the battery, and if the battery had 120V output, my answer would be "fire extinguisher". However, that's not the spirit of the question and it doesn't even ask whether the ACU is turned on. :) Franamax (talk) 01:07, 10 May 2009 (UTC)[reply]
The question shows (apparently) that the teacher is unaware that light bulbs are far from Ohmic resistances. Their resistance varies dramatically with the voltage across them. The light bulb filament's resistance increases dramatically as voltage increases. A hot tungsten filament has perhaps 13 times the resistance of a room temperature tungsten filament, with more recent estimates being 15 times the resistance. Answer the question in a simple minded way and you will likely get the teacher's answer. Or use the curve from the second ref and get an accurate answer. There is absolutely no reason that light bulbs and air conditioner motors could not be operated from a 120 volt battery. I have worked with 120 volt batteries and with light bulbs operated from them. I have worked with powerful motors operated from 120 volt DC batteries. Edison (talk) 01:29, 10 May 2009 (UTC)[reply]
The small-text indent was supposed to show a light-hearted comment, sorry if that didn't come across. The teacher is obviously making an assumption about steady state operation and expecting straight series-parallel math. Do you have a source for how a lamp filament operating at 60V has appreciably less temperature (and thus lowher resistance) than one at 120V? Not that it matters to the OP, but since you brought it up, I'd be happy to learn. Oh yeah, what was the exact model of air conitioner you ran off a 120V DC battery? Franamax (talk) 02:13, 10 May 2009 (UTC)[reply]
Note the portions of my answer in blue. If you click on them, they magically take you to books. As for DC air conditioners, ones operating on 24 volts DC are readily available at up to 3000 watts input. No reason the motors could not have been wound for 120 volts DC input. (I would exercise caution in connecting 5 of them in series across 120 volts DC (locked rotor and all that)). Certainly an invertor or motor-generator could be used to run an AC air conditioner from 120 volts DC, which to the battery would look like a 120 volt DC air conditioner. Edison (talk) 02:57, 10 May 2009 (UTC)[reply]
How appropriate, to have Edison explaining about light bulbs! Looie496 (talk) 03:15, 10 May 2009 (UTC)[reply]

Balanced symbol equation

A chemistry question, above, mentioned in passing a balanced symbol equation, also calling it a symbol balanced equation. What would one of those be? Even google thinks we might not know. Anyone fancy putting an article together? --Tagishsimon (talk) 19:24, 9 May 2009 (UTC)[reply]

The "reversible/equilibrium reaction" symbol is the ⇌ character (or the wikipedia template {{eqm}}). DMacks (talk) 19:35, 9 May 2009 (UTC)[reply]

I'm not sure that we need a new article. Assuming that "balanced symbol equation" simply means a chemical equation that is balanced, then the concept is already covered by Chemical_equation#Balancing_chemical_equations. --Heron (talk) 20:37, 9 May 2009 (UTC)[reply]
Reversible reaction uses "<math> \rightleftharpoons </math>" to get . hydnjo (talk) 21:15, 9 May 2009 (UTC)[reply]
Unfortunately, unicode &#8652; doesn't have an HTML entity name that I can find. DMacks (talk) 21:39, 9 May 2009 (UTC)[reply]
And yet another image. hydnjo (talk) 22:25, 9 May 2009 (UTC)[reply]


May 10

Microwave oven question

So I'm watching the Food Network and this lady is trying to soften up a big block of chocolate in a microwave oven so she can make chocolate curls and she goes overboard and melts the inside of the block, which happens to me all the time when I try to soften butter - those extra four seconds make all the difference.

My question here is about pre-loading. I believe you're always supposed to put some load into the oven, i.e. if you run the oven empty, you can ruin the magnetron. So if you put a cup of water in with something else, does the presence of the extra EM-absorber influence the other thing?

Put another way - if she'd put a cup of water in the oven at the same time, would the block of chocolate turn out more uniformly heated or would it have melted internally the same way and just taken longer to get ruined? And would it make any difference if it was a static or rotating-tray oven? If I can figure out those questions and world peace, I'll be a happy man! :) Franamax (talk) 01:30, 10 May 2009 (UTC)[reply]

Putting a cup of water would have been a bad idea with chocolate. The steam from the heating water would have caused the chocolate to "seize". Basically, if small amounts of water get into chocolate as its melting, all the fat clumps together in rather annoying ways. Chocolate should be kept scrupulously dry during melting. The water may slow down the melting of the chocolate, but is also is likely to mess up the process itself... --Jayron32.talk.contribs 04:29, 10 May 2009 (UTC)[reply]
For melting Chocolate you should not set your microwave to full power. Most (all?) microwaves regulate the "power" setting by switching the unit on and off at certain intervals. That gives the heat time to dissipate during the "off" cycle. Water takes a lot of energy to heat in a microwave. It's unlikely to produce any significant amount of steam at the settings needed to melt chocolate. (20% to 30% should do for most microwaves.) What you are going to get at full speed is some components of the chocolate starting to boil and separating out of the mixture. (Even if you don't try to boil water at the same time.) It's like trying to melt it in a hot frying pan instead of in a bowl suspended in a pot of hot water. A cup of water in the microwave would have created an uneven heat distribution in the oven. That would not really change with a rotating tray, because the relative position of water mug and chocolate would not be affected. You'd just get less of an effect on the side not directly facing the cup. So just lower your setting or heat the chocolate in short intervals and you should be fine. 71.236.24.129 (talk) 04:52, 10 May 2009 (UTC)[reply]


the real answer to the question is don't use the microwave to melt chocoloate. It's lazy and unprofessional. And gets bad results, as you've discovered. Melt it over water you put on the stove. —Preceding unsigned comment added by 94.27.208.52 (talk) 10:27, 10 May 2009 (UTC)[reply]

There is one reason to melt chocolate in the microwave. SteveBaker (talk) 13:55, 10 May 2009 (UTC)[reply]

has there ever been a double-blind study of any of the claims of the bible?

Have there ever been a controlled, double-blind study of any of the claims of the bible?

I'm thinking things like breaking a group of sinners into control and test groups and having only one group repent and seeing the results, etc. Obviously we can't tell which subjects would go to Heaven, but are there any claims in the bible that we can tell, and which have been tested in a controlled double-blind experiment? 94.27.208.52 (talk) 10:19, 10 May 2009 (UTC)[reply]

Not exactly what you ask for, but the article Efficacy of prayer may be of interest. --NorwegianBlue talk 10:32, 10 May 2009 (UTC)[reply]

There are no claims of the efficency of prayer in the Bible. There are no claims in the Bible that could be verified in the manner you suggest. Generally, the Bible doesn't contain many claims except of the kind "this guy said such-and-such", or historical accounts of the kind of "this tribe slaughtered another tribe, and then the Assyrians enslaved everybody". People unfamiliar with the Bible, or religion, generally tend to confuse claims made by specific churches or clergies with "the Bible". It is very simple to dispel such confusion, all you need to do is actually read the book. Reading the Bible doesn't mean you are automatcally a pious Christian, it simply means you want to check for yourself what these people keep talking about. --dab (𒁳) 10:49, 10 May 2009 (UTC)[reply]

It might be worth to add that some of the historical claims of the bible have been verified, but likewise that many are incompatible with our current knowledge of history. And many again are neither independently verified nor refuted. History does not use double-blind studies, but other methods, of course. --Stephan Schulz (talk) 11:21, 10 May 2009 (UTC)[reply]
I happen to agree with dab that actually reading the bible is the quickest way to convince one of its nonsense. However, I wonder if he has been taking his own advice, I think the bible is littered with references to prayers being answered. A quick random trawl through just one book (1 Kings) came up with numerous examples, for instance;
  • Yet have thou respect unto the prayer of thy servant, and to his supplication, O LORD my God, to hearken unto the cry and to the prayer, which thy servant prayeth before thee to day (8:28)
  • Then hear thou in heaven their prayer and their supplication, and maintain their cause. (8:45)
  • And the LORD said unto him, I have heard thy prayer and thy supplication, that thou hast made before me: I have hallowed this house, which thou hast built, to put my name there for ever; and mine eyes and mine heart shall be there perpetually. (9:3)
By the way, is double-blind referring to Saul and Onan? SpinningSpark 12:00, 10 May 2009 (UTC)[reply]

Unless you're taking the fundamentalist approach to interpreting the bible, the bible makes *no* claims. The bible is a written expression of people's faith, much like a poem or a song can be an expression of a person's love. If you're reading the poem looking for verifiable claims, you've missed the point. Wikiant (talk) 13:20, 10 May 2009 (UTC)[reply]

Of course, if everyone who was "into religion" took that view - the world would be a very different place. Sadly, I'd guess that at least 99% of religious people are either:
  1. People who DO take the fundamentalist view that every word in that crazy book is true...OR...
  2. People who haven't actually read the book from cover to cover and who are taking their view of the subject from what they are told by the people in group (1).
The book itself is mostly self-contradictory gibberish. SteveBaker (talk) 13:47, 10 May 2009 (UTC)[reply]
The trouble with this thought experiment (and one of the major reasons that science says that religion is unfalsifiable) is that if such an experiment were to disprove some hypothesis that's stated in the Bible, the religious folks out there would undoubtedly say something like "But God wants to test our faith and therefore made your experiment come out that way." - there is no way on earth they'd say "Huh! Wow! Well, I guess you atheists were right all along - I won't bother repenting anymore."
So there is no experiment you can do that would falsify what's stated in the bible because it says that god is literally omnipotent - and all rational thought on the subject ends right there. When some hypothesis is unfalsifiable, it's beyond the realms of experimentation and generally regarded as (at best) irrelevant and (typically) something that Occam's Razor says we should simply ignore.
Moreover, the experiment you suggest (where one group of sinners repents and the others do not) cannot be perfomed blind - yet alone double-blind. Again; if the repentant group were not to gain any benefits, the religious crowd would merely claim (with some justification perhaps) that they weren't truly repentant - or that the "unrepentant" group were secretly repentant after all. Of course if the experiment did show some benefit for the repentant group - I would claim (with a good deal of justification) that the results were merely consequences of the placebo effect. Without a true double-blind study (which seems impossible in this case), none of this would tell you anything useful at all.
I suppose the best you could do would be to raise a bunch of children from birth, half randomly assigned to "conventional" religious teachings and with the other half being taught some entirely fake religion. As they grow older, find the sinners amongst them (and one group has the traditional 10 commandments to sin against while the other group has things like "thou shalt not run with scissors" and "thou shalt not exceed the speed limit, even in a 55mph zone") - sort both groups into those who repent and those who do not and follow their progress. If repenters of both religions do better - then it's placebo effect - if only repenters of the "real" religion do better then maybe we're on to something here. However, the religious people would doubtless come up with a whole bunch of weak excuses to explain any results that disproved their position - so it's still unfalsifiable. Of course it wouldn't be enough to have just the children not knowing whether their given religion is real or faked - the whole society in which they grew up would have to be similarly double-blinded...so this isn't in any way a practical experiment.
SteveBaker (talk) 13:47, 10 May 2009 (UTC)[reply]
Statements such as for I the LORD thy God am a jealous God, visiting the iniquity of the fathers upon the children unto the third and fourth generation of them that hate me seem to me to be very much making a claim. Unless as you say, you are reading it like a poem or a song can be an expression of a person's love (although there is precious little love in that quote). But in that case the Bible cannot be held to be the "word of God" since it is now, according to your view, "an expression of people's faith" (not God's word) and I cannot see how it can be held to be one's moral authority with that outlook. You have a nice book of poems but not an authoritive source of belief. SpinningSpark 14:09, 10 May 2009 (UTC)[reply]
I wouldn't say a "nice" book of poems...most of it is exceedingly nasty when compared to modern standards. SteveBaker (talk) 15:01, 10 May 2009 (UTC)[reply]


In reply to Spinningspark, I did not claim the Bible contained any "nonsense". It is a compendium of Iron Age to Roman Era texts, no more nor less "nonsensical" than any other text of the same category or genre.

I did also not dispute that the Bible relates incidents of God answering prayers. It does not contain any claim that God answers prayer with any frequency or predictability. He answers them whenever he bloody pleases. Hence the verses you quoted, which portray God's reaction to prayer as a remarkable exception, not the statistical rule. You will also note that then hear thou in heaven their prayer and their supplication is the subjunctive mood, i.e. the expression of the wish that he may hear, not the statement that he does hear. Obviously, the Bible implies that God is in control. But it does not make any prediction as to when or how he isgoing toexertthat control next time you pray, it is only presenting a collection ofanecdotes of how he (allegedly) did react in the past. I would be interested in your proposal for a double-blind study for that. This would be similar to offering a prediction of what my next edit is going to be based on my edit-history, but with the requirement that this prediction is made where I can see it. I would then be completely at liberty to either confirm your prediction or to prove it wrong. In reply to SteveBaker, if you think the Bible is "self-contradictory gibberish", I trust you have never tried to read a Buddhist sutra or a Hindu Purana. If you're going to read a 2000 year old text, you'll need to be prepared to make some philological effort. You cannot read ancient texts like you do the NYT. The problem with the Bible seems to be that many people assume that they can, while nobody would dream of approaching a Pali sutra or an Ugaritic hymn like that. --dab (𒁳) 12:00, 11 May 2009 (UTC)[reply]

Yeah - I've tried to read some of those other religious texts and I have to agree that they are vastly worse than the Bible. I use the bible as an example because it's the only book of that genre that I've ever managed to read all the way through...and even then, it was a struggle! Talk about turgid! Once you get away from the well known stories, there is an awful lot of pointless waffle about who begat who and junk like that. Also, the Bible isn't a 2000 year old text - it's been hacked about by countless generations of people who were not acting to preserve the meaning of the text - many parts of it are less than 1000 years old. However, it's very clear that there is no effort made to preserve consistency or direction - you are told to do some thing on pain of eternal damnation on one page - then 10 pages later we're told the exact opposite. If this book (in it's present state) were regarded as a mere historical curiosity - then that would be very different from a book that well over a billion people claim to be the core of their belief system. I'm pretty sure though that less than one in a thousand of them has actually read the book from cover to cover (which is REALLY surprising - I mean - if you're going to define your entire life according to some book - you'd really want to read it at some point!)...right now, nobody I know (including one priest, two rabid fundamentalists and one missionary) has ever actually sat down and read it from start to finish. SteveBaker (talk) 13:17, 11 May 2009 (UTC)[reply]

tangent on atheism

And herein is the problem. If you want the bible to stand as "an authoritative set of beliefs," then you must live with the circular argument: The bible is authority because the bible says that it is the word of God and the word of God is authority. Thus, you are left with a choice. Either (1) The bible is, fundamentally, a circular argument and so is relevant only to those who, a priori, believe it to be relevant, or (2) The bible is an expression of the God-human relationship and so is relevant to everyone. Option (1) makes the bible, in effect, completely meaningless. Wikiant (talk) 17:34, 10 May 2009 (UTC)[reply]
I fail to see either the logic of that statement or its relevance to the question we are supposed to be answering. If the Bible is an expression of the God-human relationship (option (2)), it is only relevant to me if it conforms, a priori, to my beliefs in what the God-human relationship consists of. That sounds awfully similar to option (1) in my book. Supposing I accept option (2), it is not relevant to me (and I would guess a lot of other people) because I have no intention of having anything to do with a God-human relationship that has God sending a bear to rip apart children who have merely called his prophet "baldy" or who strikes a man blind for refusing to fuck his dead brother's wife. (edit) Thus your claim that option (2) is relevant to everyone is false by at least one. SpinningSpark 19:15, 10 May 2009 (UTC)[reply]
You're suggesting that, under option (2), the bible is not relevant to atheists. But, wrt assumptions, atheists are simply the flip side of fundamentalists. The former assume that everything in the bible is true (ergo double-blind studies cannot satisfy them). The latter assume that everything in the bible is false (ergo, again, double-blind studies cannot satisfy them). WRT the original question, applying double-blind studies to biblical claims is like applying double-blind studies to poetry -- one misses the point from the very start. Wikiant (talk) 22:42, 10 May 2009 (UTC)[reply]
Atheists don't believe everything in the Bible is false. We believe those parts that are supported by evidence, disbelieve those parts that contradict evidence and are generally indifferent to the rest (most of us use Occam's razor when we need to decide something on those issues, which doesn't happen very often). --Tango (talk) 23:41, 10 May 2009 (UTC)[reply]
(@Wikiant) You assume too much, I made no statement about whether or not I was an atheist, and my reasoning on (2) does not apply only to atheists; it is perfectly possible to believe in God and at the same time discard the Bible. You are making straw man arguments. It is certainly not true that a double-blind experiment could not possibly satisfy a scientifically minded atheist. Regarding us missing the point, the question asked was not what is the point of the Bible (not a suitable question for the Science Desk anyway), but whether it can be tested, which is most definitely answerable. Arguing that the Bible is poetry and so is untestable is really just trying to avoid the issue. Even if that is accepted, the Bible is still being used as a moral reference and it is valid to ask if its moral claims make sense. SpinningSpark 06:59, 11 May 2009 (UTC)[reply]
You aren't describing an atheist, but an agnostic. The atheist believes that God does not exist. If you can be swayed by a double-blind experiment, then you aren't an atheist. Wikiant (talk) 11:21, 11 May 2009 (UTC)[reply]
No, that's nonsense. I believe that when I tumble down the stairs I will fall, so I'm a gravitist. But that does not mean that I would keep that position if I managed to miss the floor and waft away. Of course positions can and do change in response to evidence. --Stephan Schulz (talk) 11:39, 11 May 2009 (UTC)[reply]
See Weak and strong atheism. What you describe is strong atheism, which I consider just another religion (generally an unorganised one, but still characterised by the same rejection of the scientific method). Most atheists are weak atheists (a category which includes (at least some) agnostics, but is larger). I consider myself a weak atheist and not an agnostic. I am certain that, given the evidence I have available to me, I can confidently reject (at any commonly used confidence level) the hypothesis that there is a god. I am, however, open to the possibility that new evidence will turn up to support that hypothesis and I will need to change my beliefs accordingly. I think most people that describe themselves are atheists hold similar views. --Tango (talk) 11:54, 11 May 2009 (UTC)[reply]
Who is "you"? Anyways, that's still not quite right. Strong atheism is a believe that there is no god, weak atheism is the lack of a believe in god, agnosticism is the belief that either we don't know, or that we cannot know, depending on whom you listen too. But even my belief in gravity or no god can be changed by proper evidence. There is a difference between belief and dogma. --Stephan Schulz (talk) 12:16, 11 May 2009 (UTC)[reply]
"You" is Wikiant, the person that wrote the message above mine and one indentation less. "Lack of belief in god" would describe agnostics, weak atheists and strong atheists - by definition, it describes anyone that isn't a theist. The difference between those categories is the level of certainly that there isn't a god. A strong atheist is absolutely certain that there isn't a god and nothing will change their mind. A weak atheists think it is possible, but very unlikely that there is a god. Agnostics aren't sure either way (they may err to one side or the other, but will be somewhere near the 50/50 mark). (Precise definitions vary depending on who you ask, by some definitions the categories overlap (see agnostic atheism, for example [by my definitions, that would probably fall under weak atheism]).) --Tango (talk) 13:08, 11 May 2009 (UTC)[reply]
Well, we have to be a little careful here. My personal position is that the "God hypothesis" is unfalsifiable. That means that I know for an absolute fact that proof of the non-existance of God (as defined by almost all religions) is impossible. That's not quite the same thing as total disbelief...but it's very, very close. I know for a fact that there are an infinite number of unfalsifiable hypotheses (Russell's teapot for example) - and that the probability that any particular one of those is true is arbitarily close to zero...but not zero. Technically, that makes me an agnostic. However, that word is generally understood by the religious fraternity as someone who has genuine conflicted doubt about the existance of God...which could not be futher from the truth. I am as certain that there is no god as that there is no tooth fairy and that there is no Santa Claus. I am as certain that there is no god as that you have not just transformed into a pink bunny rabbit while reading this post. That's a very high degree of certainty indeed. However, it's not 100% - nor can it be because we have an unfalsifiable hypothesis here. What matters is that I act and behave in every way as if I'm certain that there is no god. The almost infinitely small amount of doubt is of interest to mathematicians only! 1/infinity is pretty damned close to zero!
So I am the strongest kind of atheist there can possibly be...because logically, nobody can be 100% certain that an unfalsifiable hypothesis is actually false - although they can be arbitarily close to 100% certain. If the word "atheist" has any meaning at all - then a lot of people are atheists. If it has no meaning then there are dramatically different kinds of agnostics. There are the kind who (like me) put the probability of god(s) existing at the 1/infinity level - those who think it's a 50/50 thing - and the totally rabid fundamentalists who have a 1/infinity level of doubt in their chosen god. If you choose to apply those terms then absolutely everyone is an agnostic...which makes the word useless.
Considering atheism to be "just another religion" is the entire topic of this thread. If you still believe that to be true - then you have not read a single word of my first two posts to this thread. There is a huge difference here. To be a strong atheist (or even a weak one, I think), you have to want logic, proof, verifiability and falsifiability in all things. To be in any kind of religion at all - you have to be prepared to believe without question what someone else has told you - with no possibility of there ever being proof. That's a really dramatic difference. Calling atheism a "religion" is profoundly upsetting to me - and a contradiction of the very meaning of the word: a-theism...no theism...no religion. Religious people would dearly love for the atheists to be labelled as "religious but just in another way" - but that's totally untrue.
SteveBaker (talk) 13:17, 11 May 2009 (UTC)[reply]
Steve, you seem to have misunderstood either me, or the concept of strong atheism. I said that strong atheism is just another religion, and I stand by that. Strong atheism is the belief that there is absolutely no chance of a god existing (not a very very small chance, literally zero chance). That isn't based on logic or the scientific method, it's an absolute assertion which is not (and cannot be) supported by empirical evidence - sounds pretty religious to me (I don't think it is a belief actually held by many people, though). What you describe as your own beliefs is weak atheism, which is not religious. As a mathematician, I must object to your abuse of "infinity". Infinity is not just a very large number, it is larger than any number can be. 1/infinity is precisely zero (for the appropriate concept of infinity, it is undefined for other concepts of infinity). If I interpret you correctly (ignoring your bad maths!), you believe the probability of the existence of a god is a finite, positive real number. A very small one, but still finite and positive. (There are no infinitesimals in the real numbers - that's why 0.999...=1.) --Tango (talk) 16:08, 11 May 2009 (UTC)[reply]
By Wikipedia's definition of a strong atheist (ie someone for whom the probability that gods exist is zero), I don't think anyone is a strong atheist. The older version of the article had a more useful distinction:
Strong atheism - belief that there are no gods
Weak atheism - no belief that there are gods
There might be utility in Wikipedia's current definition to contrast with the people for whom the probability that (their) god exists is one. There are people with that stance. Zain Ebrahim (talk) 14:39, 11 May 2009 (UTC)[reply]
As I've said above, defining weak atheism as the lack of belief in a god is useless. Someone that believes there are no gods also lacks a belief in a god (tautologously). Weak atheism is the view that the existence of a god is extremely unlikely (to the point where you can just assume there isn't one), but with the acceptance that one can never be 100% certain. --Tango (talk) 16:08, 11 May 2009 (UTC)[reply]
Sorry, but that is not the standard definition. There is a difference between "I believe there is no teapot in orbit around Mars" and "I don't believe there is a teapot in orbit around Mars", although it requires a decent supply of nits. The one is active disbelief, but the other is just a lack of positive belief. As an example, someone may not have any opinion on the topic, be it for lack of information or lack of interest. --Stephan Schulz (talk) 16:50, 11 May 2009 (UTC)[reply]
I'm pretty sure it is the standard definition. Either way, your definition is certainly wrong. I know there is a difference between those two statements, but they are not mutually exclusive (a belief in the lack of a god is a lack of belief in a god, as I've said 3 times now [the converse obviously doesn't hold]). If you wish to categorise people by their beliefs in this fashion, those beliefs need to be mutually exclusive. --Tango (talk) 16:57, 11 May 2009 (UTC)[reply]
Ah. There is the beef. No, they do not need to be mutually exclusive. Of course a strong atheist also is a weak atheist, just like a communist is left-wing, or an obese person is overweight. --Stephan Schulz (talk) 19:20, 11 May 2009 (UTC)[reply]
I don't think that is the standard definition. It certainly isn't the definition use in our article, which starts: "Strong atheism is a term generally used to describe atheists who accept as true the proposition "gods do not exist". Weak atheism refers to any other type of non-theism." (emphasis mine) --Tango (talk) 19:29, 11 May 2009 (UTC)[reply]
The odd thing about these definitions is that they describe only three qualitatively dissimilar groups: theists (i.e., probability that god exists = 1), strong atheists (i.e., probability that god exists = 0), and weak-atheists-and-agnostics for whom the probability that god exists is positive but strictly less than 1. Within this group, one might draw a continuum from weak atheist to agnostic to (weak?) theist, but the difference among these is quantitative, not qualitative. Wikiant (talk) 20:13, 11 May 2009 (UTC)[reply]
I don't think that to be a "strong atheist" you have to think the existence of God is actually impossible. You just have to believe that it's possible to derive reliable conclusions from God's nonexistence.
So for example a strong atheist who believes "if the Sharks someday win the Stanley Cup, then there is a God" will also believe "the Sharks will never win the Cup".
On the other hand, a weak atheist who believes "if there is a God, then the Sharks will someday win the Stanley Cup" will simply decline to conclude that the Sharks will someday win.
As for me, I believe in God, but I don't actively hold to very many specific propositions about God. So God is there, but our beloved los tiburones are still going to have to find a way to suck it up in March. --Trovatore (talk) 20:44, 11 May 2009 (UTC)[reply]
I guess that depends on what you mean by "reliable". If you mean "certain", then those two definitions are equivalent. If you allow a little uncertainty then I would disagree. I conclude from the lack of a god that there is no harm in me not going to a church every Sunday, a mosque every Friday and a synagogue every Saturday (isn't is nice of the Abrahamic religions [I regret, I don't know when the various non-Abrahamic religions like to meet up] to have their main prayer meetings on different days? It makes everything so much simpler for those that like to hedge their bets!). I consider that conclusion reliable enough to follow it. I don't consider myself a strong atheist and I don't think I count as one under any common definition of the term. --Tango (talk) 22:53, 11 May 2009 (UTC)[reply]
Yes, it's all pretty qualitative. You might be able to add a dividing line at the 50/50 mark quite easily. I describe a weak atheist as someone that believes the chance of there being a god is very very low, but I don't attempt to quantify "very very low", I don't see anything to be gained by arbitrary dividing lines. --Tango (talk) 22:53, 11 May 2009 (UTC)[reply]
Personally, I would only use the word "atheist" of someone who was fundamentalist in their disbelief in God. Otherwise agnostic is fine and can happily include non-fundamentalist Christians too since we are all open minded. I think very few people can actually define what they mean by the statement "God exists" anyway (I tried at [3]). But the initial point in this thread is important. There can be no notional of authority or infallibility without first cracking the notion of "intent". And from an information technology point of view no finite set of words can stand as truth (description is only convergent to the real world because of the size of the data sets). Anyway for goodness sake if you learn nothing else from Richard Dawkin learn that people who explain religion to you are a selection biased set and instead make up your own mind and arguments. Most of purported religious "logic" is rubbish even to hairy old Christians like me. But a lot of claimed "science" is rubbish to so as long as I don't buy a magnet for my car fuel inlet why should these other claims bother me? --BozMo talk 11:28, 13 May 2009 (UTC)[reply]
This was a very interesting discussion about probabilities, but I think most people don't look at theism and atheism as a mathematical formula. For example, most religious people call those people strong atheists, who actively fight against theists, seeking arguments against them or just bullying them, while weak atheists or just nontheists are those who don't care. --131.188.3.21 (talk) 08:19, 13 May 2009 (UTC)[reply]
I've never heard the terms used like that. --Tango (talk) 10:54, 13 May 2009 (UTC)[reply]
Nor have I. I like to use the term "evangelical atheist" to describe the former. But according to our disputed article on Antitheism, it's a pejorative term used by Christian apologists and one or two atheists. A Quest For Knowledge (talk) 20:26, 13 May 2009 (UTC)[reply]
@Tango: Looking through google, I see that both definitions of strong atheism are fairly common. Imo, the definition I outlined above is better than WP's because it distinguishes between actual atheists. If we accept that atheists are all rational reasonable people, then there won't be any strong atheists by the WP definition (to claim any absolute knowledge about unfalsifiable hypotheses is irrational) but there are people who actively believe that there aren't any gods.

Regarding the mutual exclusivity, I don't understand your concern. Why is the distinction useful iff it's mutually exclusive? Sure, according to that definition, strong atheists are also weak atheists but I don't see this as being a problem in practice: if someone tells me they're a weak atheist, I'll assume they're not a strong atheist. Of course, if you wanted to, you could tweak the definition to make it mutually exclusive. Zain Ebrahim (talk) 09:38, 14 May 2009 (UTC)[reply]

Quark strangeness and charm

No, not the Hawkwind album. Strangeness and charm quantum numbers are preserved in strong interactions, as are the (less commonly used) bottomness and topness quantum numbers. The natural extension of these properties to first generation quarks would be "upness" for up quarks and antiquarks and "downness" for down quark and antiquarks. However, "upness" and "downness" do not seem to be identified as separate quantum numbers; instead, they are merged into the isospin quantum number. Is this purely a historical anomaly (isospin was introduced a long time before the discovery of quarks), or is there a more fundamental reason - e.g. are "upness" and "downness" not actually preserved in all strong interactions ? Gandalf61 (talk) 10:50, 10 May 2009 (UTC)[reply]

Upness and downness are as good flavor quantum numbers as strangeness, charm, bottomness and topness and are indeed preserved by the strong interactions. But it is conventional (and convenient) to replace them by two other flavor quantum numbers, namely isospin and baryon number. It is not uncommon to also replace strangeness by flavor hypercharge. The reasons for doing so are twofold.
Firstly, as you guessed, historically baryon number and isospin were identified as important quantum numbers much earlier then the existence of quarks and their flavors became accepted. Secondly, there are practical reasons for using baryon number and flavor isospin. For instance, unlike upness and downness, baryon number is preserved by all known interactions including week interactions.
Also, the energy scale of the strong interactions as given for instance in terms of the pion decay constant is much bigger than the up and down quark masses and roughly the same order of magnitude of the strange quark mass, but much smaller than any of the other quark masses. That means that the symmetry operation that replacess the up quark with a down quark and vice-versa is a good symmetry and the charge associated with that SU(2) symmetry (which happens to be flavor isospin) is a valueable theoretical tool. For instance, there are two different kinds of nucleons (protons and neutrons) and three different kinds pions, which means there are six different ways pions can interact with nucleons. Flavor isospin symmetry tells us that if we can understand and describe one of those interactions, we have automatically understood and described the other five. It pays to use the symmetries of the theory to organize the description of the interactions.
The mass of the strange quark is not entirely negligible but is still small enough that the symmetry operation that replaces the up quark, the down quark and the strange quark among themselves is a fair symmetry and the charges associated with that SU(3) symmetry (which happen to be the flavor isospin we already talked about plus the flavor hypercharge) are still somewhat valueable tools. That's why sometimes it is usefull to replace strangeness with flavor hypercharge. All the other quarks are too heavy and enlarged symmetries that also include those heavier flavors are bad and their charges are essentially useless. That's why charm, bottomness and topness are used instead.
Notice that I have carefully described those quantum numbers as flavor isospin and flavor hypercharge in order to avoid confusion with weak isospin and weak hypecharge which are entirely different quantum numbers. The latter ones are gauged symmetries and are preserved by all known interactions. Beware that folks in the Nuclear Physics crowd use unqualified isospin and hypercharge to refer to the former ones while folks in the High Energy (Particle Physics) crowd use those unqualified terms to refer to the latter, which unfortunately adds a lot to the confusion. Dauto (talk) 19:41, 10 May 2009 (UTC)[reply]
Thank you for that very clear and comprehensive answer. Gandalf61 (talk) 06:15, 11 May 2009 (UTC)[reply]

Submarine hull breach

In numerous films and books, both fictional and factual, it's said that a pressure hull breach in a submersible at great depth would happen so fast the occupants wouldn't know it had happened, and that a pin sized hole would create a beam of water that could cut a person in half. Can you tell me fast the water is actually moving? I appreciate there are a huge number of factors to consider (cabin pressure, water pressure, cabin size, size of breach and probably a host of others) but roughly how fast would the water be moving? In a (hypothetical) much larger vehicle with much greater internal space, would people potentially be able to witness and even avoid such an event (closing a hatch that can resist the pressure, for example)?81.129.229.14 (talk) 12:44, 10 May 2009 (UTC)[reply]


Well, submarine says that "When submerged, the water pressure on submarine's hull can reach 4 MPa (580 psi) for steel submarines and up to 10 MPa (1,500 psi) for titanium submarines like Komsomolets," - so we can be pretty safe in saying that 1500psi is the most pressure we have to consider here. I looked on the website of FlowCorp who make machines that cut anything from cardboard to titanium using high pressure water streams. According to their site, a machine that can cut through a few inches of cardboard requires a stream that's about one hundredth of an inch in diameter with a pressure of 20,000 psi - and to cut through steel, they use 40,000 to 60,000 psi. So it seems unlikely that a 1500 psi stream from a pinhole (which is around a hundredth of an inch in diameter) would cut a person in half if it has less than a tenth of the pressure needed to cut cardboard. Flowcorp's machines produce a Mach 2 flow at 40,000 psi and a Mach 3 flow at 60,000 psi - so at 1500 psi, the speed would probably not be all that fast.
Actually, we can work it out - the Hagen–Poiseuille equation should pretty much apply here:
Where:
is the volumetric flow rate
is the internal radius of the tube (meters)...0.000125 meters
is the pressure difference between the two ends (pascals)...10,000,000 pascals
is the dynamic fluid viscosity (pascal-second (Pa·s)), 1.3 × 10−3
is the total length of the tube (meters) - say 5cm??
So the flow rate is .000014 m3s-1 - which (through a hole with a cross-sectional area of our pinhole) is 285 meters per second - which is about 640mph. That's quite a bit more than I'd suspect from the FlowCorp numbers...but it depends sensitively on the diameter of that "pinhole" - if the hole is half that diameter then the flow rate is 1/16th as much and the speed of the jet is 1/4th as much. That sensitivity on the size of the hole means that this story can be as true or as false as you want it to be!
SteveBaker (talk) 14:56, 10 May 2009 (UTC)[reply]
The water would presumably start to spread out and slow down as soon as it came through the hole, so it would be going slower once it reached the person that was going to be cut in half by it. I'm not sure how much slower, so it may or may not be significant, but I do observe that the video clip on that website shows the high pressure stream only travelling a few millimetres before hitting the target. What we are discussing would, presumably, be orders of magnitude further (unless the person was leaning about the side of the sub at the time, but even that seems unlikely - they are double-hulled, aren't they?). --Tango (talk) 19:01, 10 May 2009 (UTC)[reply]
The Russian submarine Kursk suffered a torpedo explosion that ripped a 2 square metre hole in its hull at a depth of 100m. Nevertheless, 23 members of the crew at the other end of the submarine survived long enough for an officer to take a roll call and write a list of their names. Sadly, they all died before a rescue could be attempted.
In an incident with a happier outcome, a seawater hose failed on the Australian submarine HMAS Dechaineux (SSG 76) when it was at its maximum diving depth. The engine room was flooded, but the crew managed to control the flooding just in time to allow the submarine to surface. Gandalf61 (talk) 15:56, 10 May 2009 (UTC)[reply]
Check out Cecil Adams' answer to a similar question here. --Sean 13:12, 11 May 2009 (UTC)[reply]

The volume of ice at the north and south ice caps and Greenland?

I seem to recall that the rise in sea level is suppossed to be 50 metres if both ice caps, the ice over Greenland, and all other ice melted. But I am sceptical. The world is a big place. The oceans have an area of about 360 million square kilometres. If they rose by 50 metres, then the extra volume would be 18000 million million cubic metres. What is the total above-sea-level volume of ice at the north and south ice caps, Greenland, and other glaciers and ice? 84.13.171.69 (talk) 14:39, 10 May 2009 (UTC)[reply]

Well, your skepticism is easily debunked given some numbers:
Firstly: be careful - not all of the sea level increase predicted from Global warming comes about from melting ice. Remember, as objects heat up, they expand - and that will happen to our oceans if we don't stop the temperatures from increasing. However, the rate at which the warmth from the atmosphere heats the deep oceans is hard to estimate (I got told off about that the last time this question came up!) - so let's ignore that for the moment - bearing in mind though that it's certainly not an insignificant part of this process.
The ice over the antarctic is way more than a kilometer thick in places - remember that a cubic kilometer is a thousand million cubic meters - so 18000 million million cubic meters sounds impressive - but that's "only" 18 million cubic kilometers.
According to antarctica the antarctic ice sheet covers 13.7 million square kilometers and averages 1.6 kilometers thick...which is 22.4 million cubic kilometers - significantly more than the 18 million you need to make your math come out right. Which means that a 50meter sea level rise can easily be accounted for by the melting of antarctica alone...that's forgetting all of the ice and snow on mountains and glaciers in the rest of the world and the effect of the expansion of the volume of water due to the overall warming trend. There are even worse consequences possible here...as the ice melts from the continent of antarctica, the quadrillions of tons of pressure of all of those cubic klicks of ice goes away and the continent will rise upwards as the pressure is relaxed. This will allow the continental shelf in the region to rise up - displacing yet more water to the rest of the planet. To add insult to injury - the increasing water depths pressing down on the continental shelves (and further inland as flooding begins) will press downwards on the other continents making them sink a little - making it seem like the water is rising even faster (although in reality, it would be the land sinking - that's little comfort to someone living on the coast!) Some estimates for the worst case sea level rise go as high as 70 meters...and given these numbers, you can see why! SteveBaker (talk) 15:26, 10 May 2009 (UTC)[reply]
Of course to melt so much ice, it would take about 6x1024 joules to overcome the enthalpy of fusion of ice. Given that the relevant solar constant in Antarctica is well below the 300 watts per square meter average for the planet, it would take a long time to melt. (My back-of-the-envelope calculation suggests that if we could convert 100% of the solar radiation in Antarctica into 100% efficient ice-melting, we could melt that much ice in just 50 years, though!) Of course, this calculation is not a realistic model of climate-change - I haven't accounted for albedo, let alone convection and weather! - but it does put some perspective on the size of the numbers involved. Nimur (talk) 15:54, 10 May 2009 (UTC)[reply]
There are also factors that go in the other direction, though, including geothermal heating and heat transfer between the tropical and polar regions, as well as increased ice flow rates as the ice thins and its temperature rises. Looie496 (talk) 16:36, 10 May 2009 (UTC)[reply]
A couple of things I missed:
Melting ice at the north polar ice cap doesn't directly affect the sea level because it's already floating - which means that it's already displacing the same amount of water it'll create when it melts. That's good news - but along with it comes bad news. As Nimur points out, it'll take a long time to melt all of the antarctic ice...however, it doesn't actually have to melt - if it merely slides into the ocean and floats - that's enough to produce a problem. It has been noted that there are mechanisms that accelerate the destruction of glaciers when the meltwater from the top of the glacier finds it's way through crevasses to the underlying ground and produces a lubricating layer between the ice and the ground beneath. This allows the ice to slide much more rapidly down to the ocean where it can first form "ice shelves" and then break up into icebergs (which displace water even before they melt).
Greenland contains only about 2 million cubic kilometers of ice - less than a tenth the amount of antarctica - but it's melting much more rapidly - so in the short term, it's a bigger problem.
SteveBaker (talk) 17:08, 10 May 2009 (UTC)[reply]
Your first point directly above is not quite correct (as was pointed out to me the last time I answered a similar question - isn't it great that we can learn from new material as we come across it?). Don't forget that the water is salty, but the ice is fresh, so the displacement is not nearly as exact as we might hope. Antarctica is a huge multiple whammy due to the reasons mentioned above, but the arctic will also be a significant source of sea level rise. Matt Deres (talk)
Well - it's true - seawater is about 2.5% denser than distilled water - so there would be a tiny difference due to the arctic ice cap melting - but it's not enough to make much of a difference to global sea level. The arctic ice is only 4 to 5 meters thick on average - so even if 100% contributed to sea level rise, it would be negligable compared to the antarctic glaciers at 1600 meters thick...but you're right, technically that 2.5% density difference makes a tiny difference to the ocean depths. SteveBaker (talk) 12:24, 12 May 2009 (UTC)[reply]
Half right. The salinity makes a huge difference to the current flows and speed of mixing. Which in turn drives the rate of rise. I see we do not have an article on Double diffusive convection yet.--BozMo talk 12:33, 12 May 2009 (UTC)[reply]

Time Travel

Wouldn't time travel be undesirable? Besides the grandfather-paradox and everything else about going back in time (which is really impossible considering you need a 'receiver' which wasn't built years ago), consider this scenario. Some time in the future the world is going to face a calamity, e.g. the sun expanding into a super giant and enveloping the earth. All those people in the future (who are still on this planet) would want to find the fastest way to get out of the situation. Assuming they all choose to go back in time. The population of the planet at the time they choose to go back to would increase immensely. But the calamity in the future is still going to happen. Assuming all those people living there at the time of that calamity also decide to go back in time, and this keeps happening, sooner or later we'd have an infinite number of refugees on the planet. Maybe we should make some rules and regulations on what times in 'history' (which will be our future anyway, until the receiver is built) you are allowed to send billions of people back to, to prevent ridiculous overcrowding.--KageTora (영호 (影虎)) (talk) 15:22, 10 May 2009 (UTC)[reply]

As long as you go back far enough that you won't live to see the calamity a second time, and you don't have any children, there isn't a loop to worry about. As long as you spread people out over a millennia or two, there shouldn't be able real problem. (Assuming such a thing is possible, which it probably isn't. Also, if you can invent a time machine you can probably use similar methods to invent a spacecraft that travels faster than light, in which case you could evacuate to another solar system instead.) Incidentally, the sun will turn into a red giant, not a supergiant. --Tango (talk) 15:27, 10 May 2009 (UTC)[reply]
If you accept the repugnant conclusion, this would likely be a very, very good thing. — DanielLC 15:31, 10 May 2009 (UTC)[reply]


Yeah - there have been countless sci-fi stories about the undesirability of it...and few (if any!) that make it sound in any way desirable. However, time-travel is impossible - so it's not really something we have to be concerned about. However if you'd like food for thought:
  • Let's dump all of our CO2 emissions and other garbage back into the Triassic era.
  • Temporal tourism results in huge crowds of time travellers with fancy holographic cameras gathering just before momentous events in history. It becomes virtually impossible for any historic event to go "as planned".
  • Stock markets collapse, banks fold, casino's can do no business as future travellers do the "deposit a penny in 1800 and collect the compounded interest in 2800", sending back in time copies of almanacs and horse-racing results.
  • Temporal anomalies (if your version of time travel allows them) make life hard to lead.
  • Business collapses as each one attempts to patent all of the products of their competitors before that product was even invented.
  • Businesses realise that they can manufacture goods in one time and send them back or forwards to a time when they were either vastly more expensive or insanely desirable.
  • The movie business collapses as people travel forwards in time until their copyrights have expired and watch new movies (and the sequel that hasn't even been made yet) for free.
Pretty soon, you realise that it all becomes very silly! SteveBaker (talk) 15:35, 10 May 2009 (UTC)[reply]
(EC)If they didn't have any children, they would all die out in, say, a hundred years or so (assuming life-span hasn't increased dramatically), but that would leave a block of about a hundred years which would be uninhabitable, because of the huge population. This would keep happening until all the 100-year blocks would be gone. Also, presumably it'd be the same people coming back every time, if their 'history' (our future) is not changed by all these people coming back (which would actually be the grandfather paradox).--KageTora (영호 (影虎)) (talk) 15:49, 10 May 2009 (UTC)[reply]
True, Steve. Nice answer. I could imagine the Battle Of Agincourt, with all the French and English soldiers suddenly surrounded by millions of people, and they'd be thinking, 'where did all these people come from? Shall we postpone the battle?', which would probably end up just not happening. Nice one.--KageTora (영호 (影虎)) (talk) 15:55, 10 May 2009 (UTC)[reply]
As long as your time machine doesn't allow for travel back to before it was invented (which, to my knowledge, all serious ideas for time travel don't - unless you can find a naturally occurring time machine that has existed for a while), a lot of those problems disappear (or can, at least, be solved). Time limited copyright and patents would obviously be pointless, you would to change those laws so they make more sense. Just like we have the Berne convention and similar agreements to handle copyrights in different countries, we would need agreements between different time periods (I'm not sure what they would say, though... the idea of limited monopolies to encourage creativity without stifling progress is a good one, limiting in terms of time is an excellent way to do it but we would need some other limitation if time travel become commonplace). Economics would change drastically once the time value of money becomes zero. The underlying principle of trade would still function perfectly well. Intertemporal trade would be very similar to international trade - things like Comparative advantage would still apply, just within a very different framework. Google suggests several people have given the matter some thought. Seeing as I'm supposed to be revising, I'll go and read some of their conclusions! --Tango (talk) 16:50, 10 May 2009 (UTC)[reply]
Sadly, not my original idea. There is at least one SciFi book that describes this idea. JFK's limo sets out through the streets of Dallas - but before it can pass the school-book repository, the whole of Dealy Plaza is suddenly crammed full of bizarrely dressed people clutching their tiny, white iTimeMachines. The presidential Limo can't even make it along the street and is forced to turn back and find a different route. As Oswald emerges, confused, from the repository, hundreds of happy tourists surround him demanding autographs and asking him to pose for their holo-snaps. One inexperienced temporal tourist offers Mr Zapruder $100 for his camera. Sadly, when they return to their own time, nobody has any clue what they are talking about and nobody has even heard of Oswald. SteveBaker (talk) 16:54, 10 May 2009 (UTC)[reply]
I generally assume the Novikov self-consistency principle, so such problems don't exist. --Tango (talk) 17:52, 10 May 2009 (UTC)[reply]
Exactly what I am talking about, Steve. Consider the combined populations of both any single point in the future and all possible points in the future. It would be possible even to go back to the same point in time you've visited before multiple times and see many versions of yourself. It's a ridiculous thing. Plus, you could even go back in time, live your life there, and so long as you didn't alter the path of your parents meeting (Cf. Back To The Future), you could be safe in the knowledge that even when you die, you'll be born again, anyway.--KageTora (영호 (影虎)) (talk) 18:00, 10 May 2009 (UTC)[reply]
I don't see any problems with any of that. It's rather different to how we generally consider existence to work, but it is all perfectly consistent with itself and the assumption that time travel is possible. --Tango (talk) 18:22, 10 May 2009 (UTC)[reply]
Seems to me though the Novikov principle might result in incredibly constrained existences for those people who travelled to the past. You wouldn't be able to buy that particular loaf of bread, because it has the folate needed for the neural development of the fetal ancestor who then is smart enough to go to the school where he meets the woman with whom he parents a child whose descendant eventually introduces your grandpa and grandma. You wouldn't be able to chop down the tree whose descendant eventually falls across a creek and allows your mother to escape a lava flow just in time. That would be some strange existence. Franamax (talk) 20:46, 10 May 2009 (UTC)[reply]
Yes, but you probably wouldn't notice anything strange. You probably wouldn't even try and chop down that tree. You just have to get rid of this purely religious notion of "free will" - you are part of the universe, your decisions are part of the universe. If you universe is "conspiring" to prevent paradoxes, it will probably do so simply by you not thinking of causing them. The only time you would observe anything strange would be if you consciously tried to cause a paradox. Accidental paradoxes would just not happen with no particular reason required for them not happening. --Tango (talk) 20:53, 10 May 2009 (UTC)[reply]


Apart from being undesirable it would be a complete disaster, im thinking about the Butterfly effect...:) —Preceding unsigned comment added by 84.64.16.41 (talk) 20:46, 10 May 2009 (UTC)[reply]

It's interesting that a descendant of H.G.Wells, who first put down the idea of a time machine in one of his books, called, funnily enough, The Time Machine, was on Discovery a few weeks ago discussing the grandfather paradox. He extrapolated that if he went back to kill H.G.Wells before he came up with the idea of the book, the universe (which tries its best to eliminate paradoxes - except in the quantum realm) would make the gun not go off. He would then be asked why he tried to kill H.G.Wells during his police interrogation, and he would say he was testing the grandfather paradox, which, in turn, according to his theory, might actually cause H.G.Wells to write the book.--KageTora (영호 (影虎)) (talk) 21:43, 10 May 2009 (UTC)[reply]

It's a fun thought. I'm not sure I'd call it "interesting", though. It is too highly contrived to be interesting. --Tango (talk) 23:36, 10 May 2009 (UTC)[reply]
Time Travel is one of those technologies that everyone wants to have, but nobody wants anyone else to have. See also Invisibility Devices, Mind Reading, Flying Cars, etc.
I think this is an essential quality of a good science fiction technology. APL (talk) 03:54, 11 May 2009 (UTC)[reply]

The original poster's idea has been done I think rather definitively, albeit in the opposite direction, in The Restaurant at the End of the Universe. --Trovatore (talk) 04:04, 11 May 2009 (UTC)[reply]

I wouldn't say any of steve baker's..er... examples are problems which we would face if there is time travel... Rather, since none of these has already happened, we can say these are pieces of evidence. Which, of course, leads us to one of two conclusions : a) As has been pointed by many, time travel is impossible. b) Even if time travel is invented in the future, we, for whatever reason, won't be able to do these things. Maybe because there is some superstrict board which monitors us and forbids us to do anything of that sort... maybe it has already been invented but the scientist is just too scared that people will misuse it in the above mentioned ways that he has just kept it mum... Maybe we didn't understand properly... a different interpretation which satisfies all these problems... Who knows... My point is that don't count your chickens before they hatch.. Anything is possible in Science and Religion... Rkr1991 (talk) 11:43, 11 May 2009 (UTC)[reply]

What does religion have to do with it? --Tango (talk) 11:46, 11 May 2009 (UTC)[reply]

Hey that was just a general comment... like all's fair in love and war... Like the joker says, why so serious ? Rkr1991 (talk) 11:54, 11 May 2009 (UTC)[reply]

"All's fair in love and war" is a well known idiom with a clear metaphorical meaning. "Anything is possible in Science and Religion" isn't. --Tango (talk) 16:12, 11 May 2009 (UTC)[reply]
Isn't the OP's question the exact plot of the Star Trek episode All Our Yesterdays? Astronaut (talk) 18:25, 11 May 2009 (UTC)[reply]
I hadn't seen that episode, but after reading the article, it would appear that it is, basically, yes.--KageTora (영호 (影虎)) (talk) 04:56, 12 May 2009 (UTC)[reply]
There's probably a book about this, I just don't know what it is. It occurs to me that you cannot travel through time to change the past in any pre-meditated way because after you'd made the change it would have always been, removing the reason for you to go back in time in the first place and therefore creating a paradox. Has anyone seen this in a book before? I'd like to have something interesting to read. -Pete5x5 (talk) 05:16, 14 May 2009 (UTC)[reply]

Psychokinesis

I would like to make a BIG POINT in Psychokinesis subject.

here is your link http://en.wikipedia.org/wiki/Psychokinesis

I have red the hole thing and i find you have forgotten a BIG BIG DEAL....

You talk about Psychokinesis and telekinesis ect... most of your subjects are american peoples, how and who ever made Psychokinesis happen and possible and some articles and facts of government and scientists opinions....

But you HAVEN'T EAVEN ONES SPOKEN ABOUT " the Chinese techniques in martial art" Or what they compare Psychokinesis to is :::::::"CHI ENERGY":::::::: the power of CHI or QI or KEE or other various names, as The Yin and The Yang .... I mean you have your links for all of this but (((( MY PROBLEM IS THAT YOU DON'T LINK THE TWO SUBJECTS TOGETHER!!!!!!! ))))


Psychokinesis is a word to replace the believe of the The Yin and The Yang..... Moving objects by the Chi Power, Deep Meditation and Chi exercises, You can develop your "Psychokinesis" Powers by doing QI GONG Exercises every day!

This is a big point , if you read your own articales you will realise that both subjects talk about the same effect but both are not linked together....


'I would STRONGLY recommend to make more researches and add an IMPORTANT article that connects the two subjects together !!!!''''Bold text

Chi Power or as some call Psychokinesis IS a NATURAL Effect of the Human being....


If Article added, It would be cool to add my Name as the person who reminded this fact to you. —Preceding unsigned comment added by 70.53.148.72 (talk) 15:35, 10 May 2009 (UTC)[reply]


Sheesh! Where do we start here?
  1. What is your actual question?
  2. Please don't post your message twice - please don't include your email address (I've deleted it).
  3. Psychokinesis has NEVER not once, ever been demonstrated under proper scientific conditions - it's bullshit. If you can do it - or know someone who can - talk to the Committee for Skeptical Inquiry people or the James Randi Educational Foundation and they'll give you a million dollars (literally!) if you can prove it under rigorous conditions. The fact that nobody has yet done that - despite this lucrative offer - speaks volumes.
  4. This is not the place to soapbox your favorite ideas.
  5. This is not the place to discuss changes to Wikipedia articles - use the article's own Talk: page.
  6. Be aware that telekinesis is considered "pseudoscience" and Wikipedia has some increasingly strict rules about how we portray such things. If it's bullshit (and this is), we're going to say so.
  7. Nobody's name is "attached" to an article - take a look at any of them - no accreditation is ever given.
  8. Wikipedia is "the encyclopedia you can edit" - which means that if you find a mistake or see something that needs fixing, you have to do it yourself. However, I'll go out on a limb here and say that every edit you make along the lines you describe will be deleted again within a lot less than 24 hours.
SteveBaker (talk) 15:44, 10 May 2009 (UTC)[reply]
Steve, can you package that response into a suitable template format, for quick re-use? I've been on pseudoscience patrol often enough to need it... Nimur (talk) 16:09, 10 May 2009 (UTC)[reply]
I have it on a hot-key. :-) SteveBaker (talk) 16:37, 10 May 2009 (UTC)[reply]

And (9) Please don't SHOUT. It will only serve to destroy what little credibility you might gain.--86.25.193.89 (talk) 16:52, 10 May 2009 (UTC)[reply]

"UN-altered REPRODUCTION and DISSEMINATION of this IMPORTANT Information is ENCOURAGED, ESPECIALLY to COMPUTER BULLETIN BOARDS." —Tamfang (talk) 20:52, 10 May 2009 (UTC)[reply]

I think he's complaining that the Psychokinesis article doesn't mention the Chinese concept of Qi. Astronaut (talk) 18:09, 11 May 2009 (UTC)[reply]

I tried placing it on a computer bulletin board with telekinesis, not touching the keyboard, sadly without any results. What am I doing wrong? Should I have used a Mac instead of a PC? Edison (talk) 14:02, 12 May 2009 (UTC)[reply]
I think Linux is the only OS that supports telekinesis, and also FreeBSD if I recall correctly --BiT (talk) 15:27, 12 May 2009 (UTC)[reply]

Why do males have nipples?

- Most male mammals have nipples like female mammals. Why do the males need nipples? Is it a kind of rain check, to be used later if the females become unable to suckle for some calamitous reason? ~~ —Preceding unsigned comment added by 59.93.8.216 (talk) 15:41, 10 May 2009 (UTC)[reply]

ava looke at Nipple#Nipples_on_male_mammals a little bit maybe

ncal43 —Preceding unsigned comment added by Ncal43 (talkcontribs) 15:44, 10 May 2009 (UTC)[reply]

The male nipple actually carries a very nice lesson about how evolution works. The only genetic difference between male and female mammals (well, for most mammals) is the Y chromosome, which marks an animal as male. Because there is only one copy of the Y chromosome in a cell, it cannot evolve in the same way as other chromosomes, which have two copies each. The consequence is that the Y chromosome is very small -- nature keeps as few genes as possible on a chromosome that can't evolve in the usual way. This means that the genetic difference between males and females is tiny -- basically just a few genes that control the expression of other genes. Since the genomes are so similar, every structural difference between males and females requires complicated gene-programs, so it pays for nature to keep the differences as small as possible. It's easier to reduce an organ to a small, nonfunctional form, like the male nipple, than to eliminate it completely. Looie496 (talk) 16:52, 10 May 2009 (UTC)[reply]
I would maybe reword that last sentence to say that it is easier for an organism to use the same body-development plan for as long as possible before coming to rely on genes on the sex-linked chromosomes. Looking at the link Ncal43 supplied, it seems that rather than the "organ" being "reduced", it is actually the further development of the mammary gland which is suppressed. And the Y chromosome is actually full of a startling amount of junk. Franamax (talk) 20:09, 10 May 2009 (UTC)[reply]
It only takes a tiny bit of female hormone to make the male breast start producing milk. The normal male breast is indistinguishable from the immature female breast. See Gynecomastia and Male lactation. Edison (talk) 23:00, 10 May 2009 (UTC)[reply]
There is only really one relevant gene on the Y chromosome - SRY. It determines the production of male hormones, everything else follows from that with exactly the same genes as females have. --Tango (talk) 23:32, 10 May 2009 (UTC)[reply]
I should clarify - there are other sex-linked genes, but they don't determine sex. They happen to be on the part of the Y chromosome that doesn't cross-over with the X chromosome, but that is just coincidence. --Tango (talk) 23:34, 10 May 2009 (UTC)[reply]
Further to the above, I would like to question the implicit assumption invariably accompanying this question, that nipples only have one function. Many organs in many organisms have dual or multiple functions, and the nipples are no exception. Apart from their primary role as efficient interfaces between maternal breast and infant mouth (which is not indispensable, as Monotremes lack nipples), they are also, at least in H sapiens, erogenous zones, being erectile and sensitive during sexual arousal. This applies to male as well as female nipples. Any mature and sexually active male unaware of this must have a very unimaginative partner. 87.81.230.195 (talk) 03:53, 12 May 2009 (UTC)[reply]

Physical error in Star Trek?

In the new Star Trek film, Kirk and Sulu are falling in a free fall toward the surface of Vulcan, when they are beamed up in mid-fall to the Enterprise, and they are shown falling on the transported pad with an audible thud, and then merely brush it off. Now, when falling on the ground from the sky, isn't what kills you that the deceleration happens too fast, causing all the kinetic energy in your body being transferred in one go? In this case, it doesn't matter what Kirk and Sulu are falling on, or from what height. If they aren't gently slowed down in advance, they'll die anyway when they suddenly stop falling. In fact, they don't even need to hit anything - merely stopping in mid-air would be fatal if it happened too fast. Have I got this right? JIP | Talk 17:06, 10 May 2009 (UTC)[reply]

It's fiction. Try not to worry about it. Transporters don't really exist so issues of whether they are depicted realistically are quite utterly unanswerable. (When Time Magazine asked "How does the Heisenberg compensator work?", Star Trek technical adviser Michael Okuda responded, "It works very well, thank you.") SteveBaker (talk) 17:12, 10 May 2009 (UTC)[reply]
I was expecting this kind of answer. Merely replying this way is, in fact, equal to saying Kirk could have simply waved a magic wand and caused Nero's evil rogue ship to disappear and make everyone happy, when after all, it's fiction. What I would have wanted, is an answer to whether I have understood the physics thing in falling down correctly. Just because something didn't really happen is no excuse to throw all pretense of realism out of the window. JIP | Talk 17:16, 10 May 2009 (UTC)[reply]
Actually, that's why the transporter needs a Flux_capacitor. It adjusts the speed of incoming bodies to match the inertial frame of the receiving station. In the old series, Scotty is doing this manually with those levers he operates. If the relative speed is high, he may not compensate the movement completely - and rather than risking to have them fly up and then down again, he errs the other way. Alternatively, look at the concept of a Bergenholm in the Lensman series, which operates by temporarily suppressing the inertia of a body, allowing it to accelerate without picking up kinetic energy (and to revert to the old velocity once it's disabled again). --Stephan Schulz (talk) 17:33, 10 May 2009 (UTC)[reply]
Yes, you are essentially correct, if they impacted the transporter at the same speed they were going to impact the ground at, it would have the same effect. We'll have to assume the transporter changed their speed when it changed their position (I see no reason why it couldn't do that - if you can rematerialise matter you should be able to choose its momentum pretty freely) - let's assume the small drop is because whoever was transporting them (I haven't seen it yet) was afraid he wouldn't get the position exactly right (perhaps due to them falling in an unpredictable way) and didn't want them to materialise halfway through the pad. As for what would happen if you just stopped in mid-air, it would depend on what stopped you. The reason rapid deceleration is bad is because different parts of your body decelerate at different rates (the bit that impacts first stops almost immeadiately and the rest of your continues moving, so you get crushed). If whatever stops you stops you in a similar fashion, it will be just as bad, if it stops all of you at the same rate (using artificial gravity, or something), then it would be harmless. --Tango (talk) 17:43, 10 May 2009 (UTC)[reply]
I'm sorry - but you can't expect a scientific answer for something that IS magic. Transporters do not - nor cannot exist...period. What they do or do not do with residual momentum when the source and destination are moving at different speeds is quite utterly unknowable. I could say - "Well, the StarTrek transporter perfectly removes the difference is speed and direction between source and destination - and all you saw in the movie was due to the transporter placing the center of gravity of the humans at an appropriate distance above the deck. The problem was that they were lying flat at the time and thus fell two feet to the ground on arrival"...but it's fiction...it doesn't exist...it doesn't matter. When you watch a StarTrek movie, you turn off your brain and enjoy the ride. SteveBaker (talk) 17:47, 10 May 2009 (UTC)[reply]
Nonsense, it's still entirely possible to apply a modicum of scientific thought to the process. After all, science is a way of approaching problems. So long as we accept that A) These are not real-world facts we're dealing with. and B) If we go too deep we'll eventually hit either an unknowable, or a contradiction. Then why couldn't this question be approached in a completely logical, scientific way?
We have observed transporters moving individuals from planet surfaces up to a ship in orbit. (In fact, that's their primary purpose.) The speed difference between the ship and the planet surface can be massive. (They usually show the Enterprise entering orbit against the planet's rotation, for whatever reason.) Since they can complete these planet-to-ship transports without splattering anyone against the side of the transporter room, we have to assume that the transporter is adjusting the away team's speeds to match the ship's speed. So the physics of this dramatic beam-out in the new movie is nothing special, the transporter must do that for every single transport. (Luckily, obscure transporter physics is not what makes the scene dramatic.)
So why did they crunch against the floor? Easy. They beamed in on their sides instead of the usual boots-down orientation. Even if they were perfectly lined up with the floor of the transporter you'd still fall a little. Imagine the difference between beaming in with both boots flat on the deck, and beaming in with one elbow just barely touching the deck at a weird angle.
We've seen this sort of thing before, where someone beams up in a weird position and then falls when they beam in. Not so much in the original series, but in TNG, Voyager, etc. Another fun thing is when they beam someone up who's running, but they beam in stationary and have to 'catch themselves'. I'm sure we've seen that once or twice. APL (talk) 03:47, 11 May 2009 (UTC)[reply]

Tango's reply answered my original question about physics. As for SteveBaker's comment, you are essentially right, but your original reply came out as a simple handwave, rather than the detailed explanation you now gave. Stephan Schulz provided a good reply while keeping in mind that Star Trek is fictional: "Yes, if they weren't slowed down, they'd die. That's why Scotty used a flux capacitor to slow them down. How does a flux capacitor work? By magic." I wasn't asking about the internal workings of flux capacitors or transporters, but about simple everyday physics that happen outside the transportation. JIP | Talk 18:04, 10 May 2009 (UTC)[reply]

(EC - twice)Transporters could not exist and would be a dangerous thing anyway even if they did. We all know about the 'got transported inside a physical object by accident and died' thing. Well, that would happen in any place that has an atmosphere. You'd be transported into that air, and oxygen atoms are physical objects. You'd end up meshed up with the air.--KageTora (영호 (影虎)) (talk) 18:23, 10 May 2009 (UTC)[reply]
That problem could perhaps be solved by having the transporter transport the air out of the way first, by having the transported object and anything at its destination simply exchange places? How would it do that, then? The same way it would already work: by magic. We aren't interested in how transporters work, we are interested in what effects they have. JIP | Talk 18:27, 10 May 2009 (UTC)[reply]
Even that seems tedious and dangerous, not speaking about the heisenberg compensator magical plot device. The only possibility that I can think of for a transporter to work would be the existence of a forth spacial dimension which we learned to access somehow. Anyone who knows about Flatland knows how it could be used for teleportation. --131.188.3.20 (talk) 23:32, 12 May 2009 (UTC)[reply]
Well, just as a side note, for a transporter to work, it would have to dismantle all the atoms in the person's body, then reconstruct the body at destination. This would mean that the actual scene where they suddenly appear in mid air would be silly, considering they never did that before in any other scenes. As for the rapid deceleration thing, yes. There is an advert on British TV warning against bad driving where two cars have a head on collision. And it warns (very graphically) that even if you have air-bags you can still be killed because your internal organs keep moving even when you and the car have actually stopped suddenly, and they can all be ruptured.--KageTora (영호 (影虎)) (talk) 18:35, 10 May 2009 (UTC)[reply]
Which, I think answers your question. It would be an instant dismantling of every atom in the body, therefore causing no movement of internal organs caused by any rapid deceleration, and then reconstructing the person at destination.--KageTora (영호 (影虎)) (talk) 18:41, 10 May 2009 (UTC)[reply]
So, when they reappeared above the transporter pad, they would have zero velocity, and then they would just drop down the half a metre or so onto the transporter pad, leaving them essentially unscathed? That makes sense. How the transporter is going to change the velocity to zero without going through deceleration is a question of quantum physics, not mechanics, and therefore outside the scope of this topic. JIP | Talk 18:47, 10 May 2009 (UTC)[reply]
Teleportation is generally described in terms of transferring information - you examine the object at its source, destroy it, transmit the information about it to the destination, and rebuild it. There is no reason why you should have to rebuild it with the same momentum it started with (although to momentum of parts of it relative to the others would need to stay the same - you don't want your heart to stop mid-beat, or anything, who knows what would happen then?), since it isn't the same object. It is a new object that is created to be identical to the first. --Tango (talk) 18:52, 10 May 2009 (UTC)[reply]
"There is no reason why you should have to rebuild it with the same momentum it started with" No reason? Except that simple, point-by-point subtraction of the free-fall momentum is not going to work! Every atom is so confounded by thermal noise; macroscopic displacements of organs are going to throw the tissues out of place; molecular vibrations; statistical trends of gas-flow in the lung and fluid in the blood.... I don't think you could just "subtract" a scalar speed from every atom in a body and have everything "be okay." (Imagine what making an enormous change to the net velocity of every single atom will do to the instantaneous temperature and pressure of the material?) SteveBaker is correct to hand-wave this off as magic. If we had a scientific explanation for these "small details", we would have already figured out transporter technology. We do not have any explanation for these details (probably because it is fundamentally physically impossible; at the very least, we have at least got a technological limitation). Therefore, every time you make a "science-like" statement, as a "flux capacitor that resets the momentum", you are just moving the bar up a notch in terms of where you stopped caring about the magic. You can't explain the flux capacitor scientifically, so it's just technobabble for a "magic-wand". I don't see why you should even bother pretending that this has solved the physical incongruity. Nimur (talk) 19:06, 10 May 2009 (UTC)[reply]
Well in TNG, they replaced the flux capacitors with pattern buffers, so they must have been working on the science. :) Possibly helped by the Stargate SG-1 team. I'd like to know about a few more of the small details though, such as why a phaser vanishes the person, their clothing and anything they're holding - but not one scrap of ground or the building they're holding onto. Also why no-one ever thought of installing seatbelts when they upgraded the Enterprise. The TV/film industry is truly a strange land. At least the old-time SF writers had to try to explain the miracles. Franamax (talk) 19:47, 10 May 2009 (UTC)[reply]
That isn't a problem with removing the momentum, that's a problem with teleportation itself. We have to suspend disbelief at some point. In my opinion, good science fiction requires us to only suspend disbelief for a very small number of things (they can be extremely unbelievable, though) and then everything else follows logically from those assumptions. If you assume that teleportation is possible and just ignore all the problems with it, then it requires no further suspension of disbelief in order to accept that you can remove the momentum from a teleported object. Star Trek is reasonable sci-fi by that definition, but they do tend to resort to technobabble a little more than they ought. X-Files is terrible sci-fi (well, "speculative fiction" is probably a better term, it isn't strictly sci-fi, but the same point applies) - it requires you to suspend disbelief separately for each episode (which is particularly bad considering it is meant to be about solving mysteries and there is no way you can possibly solve the mystery before it is revealed [and trying to do so is part of the fun of watching such shows] since you don't know what unbelievable thing is going to turn out to be true this week). Stargate is good, once you accept the premise of the show there is very little that is unbelievable in each new episode (nothing at all in many episodes). --Tango (talk) 20:24, 10 May 2009 (UTC)[reply]
Yes, also, laser weapons would be viewed as a constant stream of light, not a bolt that flies along much slower than the speed of light, considering they are made of light.--KageTora (영호 (影虎)) (talk) 19:54, 10 May 2009 (UTC)[reply]
Funnily enough, transporters DO exist, at least in some form. I saw something on Discovery about one that has been built, but it can only transport a single atom at this point in time. I have no links for this. Anyway, there was some question about it being the same atom or a replication of the one the machine just destroyed. The scientists who created it, however, said it was the exact same one, but didn't provide any proof. To transport a human, however, would require (obviously) a bigger machine and much more power to store all the information during the 'transportation' process. But this is a problem, because we have no way of knowing whether it is the same person that has been 'transported' or an exact replica of one that has been utterly destroyed, which is, in fact, terrifying if you are the one going through the process. The reason we have no way of knowing is because, being an exact replica, they will say they are the same person.--KageTora (영호 (影虎)) (talk) 19:51, 10 May 2009 (UTC)[reply]
Is this] what you're talking about? They're not claiming to have moved the physical atom though. Franamax (talk) 19:54, 10 May 2009 (UTC)[reply]
... and even if one atom could be "transported" (or re-assembled as an identical copy), there are bandwidth limits that mean it will never be possible to "transport" anything nearly as complex as human by any technology imaginable at present. It's fun to pretend, though. Dbfirs 20:27, 10 May 2009 (UTC)[reply]
Quantum teleportation. It's quantum information (qubits) that gets moved, so it's not really teleportation. It's the same as this message making its way from my computer to yours—there are no electrons or other particles that make the trip. I'm not teleporting electrons even though there happen to be electrons on both ends. Furthermore you can move qubits around by using a quantum channel, you don't need the quantum teleportation protocol. So really it's not a big deal, and I wish these articles wouldn't be so sensationalistic about it. -- BenRG (talk) 22:14, 10 May 2009 (UTC)[reply]
A moderate fall to the floor is a moral lesson by the transporter. Falling at high speed can be dangerous when there doesn't happen to be a transporter with a helpful operator around. The transporter doesn't compensate completely for the speed difference in order to give the fallers a scare so they will try harder not to fall another time. That's my theory anyway. Seems at least as good as technobabble. Some parents use a similar principle when warning/punishing their children for doing dangerous things. "Son, you could have drowned by swimming less than an hour after eating so I'm pulling out the waterboard to show you how it feels". What, your parents didn't do that? PrimeHunter (talk) 20:55, 10 May 2009 (UTC)[reply]
But it would be exceedingly unusual for the velocity of an orbiting transporter room to exactly match the velocity of the place they're beaming from/to. There are at most two locations moving with a given velocity on the surface of a rotating sphere. Only dramatic license can explain the observed behavior of Star Trek transporters. -- BenRG (talk) 22:14, 10 May 2009 (UTC)[reply]
Back to the workings of the flux capacitor, which JIP asked about, I'm thikning that what might have happened (and I'll grant you I haven't seen the movie) is that the atoms were not just snatched fromt he air like, say, a baseball player catching a ball. Instead, couldn't they have been slowed down at a rate allowing them to suffer minimal damage, then "beamed in" when they were slow enough to avoid the harmful effects?
In other words, if velocity was lethal limit, all the atoms were slowed down while continuing to go ythrough the air, with a "transporter lock" on them. They were slowed gradually, then, even moved back up a ways if they had to be. Then, when they were going slow enough, they could then be safely brought to a place where they could land.
This may just be explaining the "magic" a little more, but at least it sounds more plausible than some things. After that, I'll just use the line fromt he parody I wrote in college once:
Scotty: I can't defy the laws o' physics, Cap'n.
Kirk: Sure you can, Scotty, this is Star Trek, we do it all the time. :-)Somebody or his brother (talk) 01:12, 11 May 2009 (UTC)[reply]
That wouldn't fit with how transporter beams are usually described, it would need to be a tractor beam. There is no reason why they couldn't write a story in which a tractor beam and a transporter beam are used together for such a purpose, but I see no reason to assume that's how it worked unless they say so. As I've said above, there is no reason to think momentum has to be preserved during teleportation. (And, as BenRG points out, there is going to be relative velocity between the source and destination anyway whenever the ship is in orbit.) --Tango (talk) 10:16, 11 May 2009 (UTC)[reply]
Actually - the things that bother me most about that movie in particular are the ways that the computer interacts with people. Like when the computer politely informs the captain that the autopilot has been damaged...surely the autopilot is a piece of software that runs in the computer? If it has enough working processors to perform speech recognition and to drive all of those fancy displays - why the heck can't it run the 7 lines of code you need to implement an autopilot?
 while ( the autopilot is engaged )
 {
   if ( you're too far to the left ) steer to the right a bit ; else steer to the left a bit ;
   if ( you're pointing too far up ) steer down a bit ; else steer up a bit ;
   if ( you're going too fast ) slow down a bit ; else speed up a bit ;
   sleep ( 1 millisecond ) ;
 }
You could argue that the computers sensors were damaged - yet they appear to be giving the captain realtime on-screen data of time to collision - you could argue that the computer's control over the engines and steering are broken - but then we'd have to believe that there is a hard-wired connection between the joystick on the captain's chair and the actual engine controls, which seems really unlikely.
If he hadn't kicked the engineering team off of the ship he could have had them write those 7 lines of code (it took me less than 30 seconds) then had the last of them beam off the ship. Anyway - what's wrong with "Computer: Transfer control to the shuttlecraft"? Or just aim the ship in the right direction and say "Computer: Don't touch the engine controls" - then run to the transporter bay and beam yourself to the shuttlecraft?
Then, when Scotty is trying to "get a transporter lock" on the falling crewmembers, he's looking at a display with dots on it showing the position of the two crewmembers and frantically trying to line up a couple of cursors on top of them. Aside from the fact that he only seems to have to do this in two dimensions(?!?) - the fact that computer knows their positions accurately enough to draw the graphics on the display means that it would be TRIVIAL to have it 'snap' the aim cursor to the nearest dot...making it a simple matter for any unskilled transporter officer to snatch people out of the air like that.
Conclusion: The Federation's computer programmers are utterly useless!
Anyway - we should listen to the wise words of Arthur C. Clarke: "Clarke's Third Law: Any sufficiently advanced technology is indistinguishable from magic." - which is what we have here. Magic - with all of it's capabilities and arbitary limitations. SteveBaker (talk) 12:30, 11 May 2009 (UTC)[reply]


There are some minor problems with the code given for the autopilot. In reality it's not that simple. What you just described was the most simple proportional controller, which will oscillate horribly even in the most ideal conditions. For control systems at least a PID controller is required, and if not all of the system's parameters are known, or there can be measurement errors (which they always are), than even that's not enough and you have to combine them with predictors and parameter estimators, or use more advanced and complicated controllers. However, I still agree that if the voice recognition works, the autopilot should also work, except for some required hardware accelerators burning out (you know one of the most important starship design rules: no fuses in the cockpit), but even then there should be backup software solutions available. But I am sure it's a lot more than just 7 lines of code.
Science fiction means, that it can contain devices or events which are not possible by our current understanding and/or technological level. You can invent your own rules, but you have to stick to them. So if you have a flux capacitor and you use it by your own rules and without contradicting yourself, it's fine. You can use any crazy science you want, if the connections in it are logical, the consequences are well described, and all the gadgets are explainable under the rules you use. Look at Stanislaw Lem for example: in the novel Fiasco he uses giant walking robots, cryogenics, some kind of pseudo-FTL space travel, gravity-altering devices and weapons, nanofungus, but it still remains scientific: the miraculous gadgets are not just plot devices, their functionality as well as their shortcomings are presented in detail and, logically, seem plausible. Or, a more modern example, Fine Structure [4] has every crazy thing even thought about by science fiction authors: alternate dimensions, teleportation, FTL, time travel, superpowers, universal constructors, telepathy, etc., but is still amazingly scientific and realistic.
Star Trek is, by the way, not science fiction in the strict sense. Science is always just a background or a plot device. The problem of these kind of "flux capacitors" is, that they usually do not follow even their own rules. If it really worked, would it make sense? - that's the question we should ask. In the Back to the Future, for example: Why would anyone have a newspaper article about a photo of an empty place in the graveyard? How do Marty's parents not freak out when they realize their son looks exactly like the mother's previous lover? How does doing just nearly similar things lead to the nearly same future? How can Marty possibly take the place of a completely different child (as he altered his parent's marriage quite a lot)? - These are the questions that make it unrealistic, and not the "how does the flux capacitor work?". --131.188.3.20 (talk) 14:23, 12 May 2009 (UTC)[reply]
Plot devices aside, there is actually an awful lot of considerations embedded in the auto-pilot's concern "Where should the ship be and how fast should it be moving?". It would start with knowing what course was entered, knowing current position and velocity relative to something (what reference frame does one use on a starship?), and being able to adjust for potential obstacles, etc. I don't see a huge problem with a ship being able to predict time till collision and yet not having enough guidance for a full autopilot to know where it should be going, etc. Your point about simply having the ship maintain constant velocity in a predetermined direction is a good one though, I don't see why that should be a problem. Dragons flight (talk) 16:29, 12 May 2009 (UTC)[reply]

I know I'm coming late to the party, having only just seen the movie, but I have to but in with the obvious answer. <assume star-trek meta-physics>The transporters always have to compensate for a large velocity difference, since the ship is in orbit, thus has a significant velocity in the frame of the "stationary" person being transported from the planet surface (this has been mentioned above). As for their horizontal orientation upon arrival, the same point applies: being in orbit, the ship is almost never in the same "upward" orientation of the person being transported, so the transporter must compensate for this always. So, if you are talking about consistancy with previous and subsequent events (which I can only assume represent the physics of the Star Trek universe), their velocity upon arrival is not an error, but their orientation is.</assume star-trek meta-physics> Ew I feel dirty, let's not talk about hypothetical future physics anymore :-D -RunningOnBrains 16:55, 12 May 2009 (UTC)[reply]

Also, it seemed to me that near the end of the movie when they beam Spock off the little future ship, he's sitting down when he gets transported but arrives standing up. I guess the plot wouldn't have benefited from slapstick "chair pulled out beneath him" humour at that part of the script, but the nebulously defined transporter serves its role as a charmingly flexible plot device yet again. TastyCakes (talk) 17:14, 12 May 2009 (UTC)[reply]
Star Trek transporters (and the equivalent technology in other si-fi) have always been able to straighten people's legs, there are plenty of examples of it. I don't see anything that function would contradict, so we just need to suspend disbelief for it. --Tango (talk) 17:57, 12 May 2009 (UTC)[reply]
My point is that the transporter apparently couldn't change the orientation of Kirk and Sulu when they were falling, but it could change Spock's orientation when his ship blew up. TastyCakes (talk) 18:50, 12 May 2009 (UTC)[reply]
Perhaps the sudden change in gravity would be harmful? As it is, they went from about 1g (presumably they were at terminal velocity, I haven't seen it) to 0g (they would have been in free fall once they appeared above the pad). Perhaps changing from 1g in one direction to 1g in another would be too much (it would be a change of about 1.4g in magnitude). It's not a particularly good argument, but it will do if you would rather that than just calling it creative license. --Tango (talk) 17:57, 12 May 2009 (UTC)[reply]
If transporters worked by creating some kind of wormhole, then momentum, velocity, gravity and all these would have a great effect. However, by the use of Heisenberg compensators, I thing it's very clear they work by deconstructing the target and reconstructing an identical copy. So they could reconstruct the copy when and where and facing whatever direction as they wanted. The only question that remains is why are they not using it for "saving" people and reconstructing them in case of death, or just healing injuries by replacing damaged body parts. Maybe a protocol forbids it? --131.188.3.20 (talk) 23:20, 12 May 2009 (UTC)[reply]
Why not? BECAUSE IT'S FICTION. It doesn't suit the storyline for there to be no danger. They continually walk the fine line between making things ridiculously easy for everyone - and yet keeping the technology seeming fresh and exciting. SteveBaker (talk) 01:37, 13 May 2009 (UTC)[reply]
I think you still do not understand what I was saying. I have no problem with creating crazy rules, so no need for shouting; the fact that they don't even stick to their own rules is what was bothering me. I was not saying "it's impossible for it to function", I'm rather saying "even if it functioned like described, there would be huge plot holes". And I'm not speaking just about Star Trek, but all kind of "light" science fiction. --131.188.3.21 (talk) 07:59, 13 May 2009 (UTC)[reply]
One of the things you need to be careful of when building a fictional world is to keep it self-consistent. Star Trek isn't particularly good at that, which tends to strain the suspension of disbelief at times. --Tango (talk) 11:00, 13 May 2009 (UTC)[reply]
Because "their patterns degrade in the buffer" over time. There are plenty of times when this is mentioned. The amount of information required to reconstruct a human body would be enormous, so presumably they have some kind of specialised storage medium that doesn't work for long periods of time. --Tango (talk) 11:00, 13 May 2009 (UTC)[reply]
There was an entire mini-series in the Star Trek: The Next Generation spin-off book series where Montgomery Scott is transported in an emergency situation, and then kept in the transporter buffer for seventy years, so when he reappears in Captain Picard's time, he's still as fresh as he was in Captain Kirk's time. JIP | Talk 19:22, 13 May 2009 (UTC)[reply]
Running, There's no error in their arrival orientation. It was the safest orientation for them to materialize in. If I wasn't in a strictly standing position I sure as heck wouldn't want to be beamed in upright. That would just give me farther to fall before I wanged my head against the deck! APL (talk) 04:29, 15 May 2009 (UTC)[reply]

On a side note, About 20 years ago, I read an interesting book by an author called Simak - can't remember the title (it's on my bathroom bookshelf, but I can't be bothered going and getting it) about a guy who was transported to some planet on the intergalactic transportation network, and through some error, two copies of him were made, one arriving at the destination intended and the other in some place that nobody knew existed. The one that arrived at the correct destination is subsequently killed in an accident, while the other somehow manages to get back to Earth (doesn't say how, because that's actually when the book starts - loophole alert!). The book goes on to discuss the legal implications of him being 'dead', yet still alive. Interesting book.--KageTora - (영호 (影虎)) (talk) 02:23, 16 May 2009 (UTC)[reply]

Ha! Wikipedia has an article on him, and saved me the trouble of going to the bathroom. The book is called 'The Goblin Reservation'. Wikipedia is wonderful ! --KageTora - (영호 (影虎)) (talk) 02:26, 16 May 2009 (UTC)[reply]

the sun's limb

There's an old joke:

"Your cough sounds better."
"I've been practicing."

Well, I have forty-some years' practice in making myself misunderstood. So this is a paraphrase of my question of April 28 (I didn't see the replies until today).

Given:

  • The sun, being gaseous, has no discrete surface. (I got clear agreement on that point, but that's not what I was looking for.)
  • It sure looks as if it had one.

What is it that looks like a surface? When astronomers speak of the limb of the sun, e.g. to define the four 'contacts' of a transit, what do they mean? —Tamfang (talk) 19:10, 10 May 2009 (UTC)[reply]

It is a gradient, it's just hard to see it very clearly. In some cases, especially eclipses, it shows up very nicely as a gradient. Have you looked at photosphere and chromosphere and solar corona? Depending on your astronomical need, and your area of research interest, any of these might be considered "the surface" of the sun. Specific definitions with specific radii will be suitable for particular cases, but as you already realize, there is not a clean radius at which there is total consensus to call the "edge". Nimur (talk) 19:32, 10 May 2009 (UTC)[reply]
Gradient. The central region has been blocked out to prevent saturation of the imager, because the contrast difference between the photosphere and the corona is pretty huge. Nimur (talk) 19:32, 10 May 2009 (UTC)[reply]
In the eclipse picture, I see a gradient in the outer corona, yes. I don't see a gradient in the part of the picture that is relevant to the question. —Tamfang (talk) 20:37, 10 May 2009 (UTC)[reply]
Would one definition be the point where an object behind the sun is no longer visible at a given (or any) wavelength? Franamax (talk) 19:51, 10 May 2009 (UTC)[reply]
Perhaps, but I don't think that would give you a firm boundary. Objects would just get harder and harder to see (as more and more of the photons from them get blocked). There would be no point where suddenly all the photons are blocked when none of them were before. --Tango (talk) 20:58, 10 May 2009 (UTC)[reply]
There is no precise boundary to the Sun, just a gradual fade to nothingness.
The reason it looks like it has a clear boundary when you look at it with the naked eye (or, more advisedly, through a solar filter!) is because the range over which it varies from dense enough to be easily seen to sparse enough to be impossible to see is quite small (from this far away, at least). If you look at it through a telescope you can see that it does fade out gradually, as seen in this picture. --Tango (talk) 20:04, 10 May 2009 (UTC)[reply]
Sun#Photosphere: The change in opacity is due to the decreasing amount of H- ions, which absorb visible light easily. That's a start, anyway. —Tamfang (talk) 20:37, 10 May 2009 (UTC)[reply]
I'm not sure of this, but it seems to me that clouds could be seen in a similar way. From a distance they can look opaque and sharply defined, yet the closer you get the less distinct their boundaries appear to be. I am not a meteorologist nor an expert on the sun - just wondering if it's an apt analogy. --Scray (talk) 01:48, 11 May 2009 (UTC)[reply]
Sounds like a good analogy to me. There are quite a lot of difference between small droplets of liquid suspended in a gas and a large ball of plasma held together by self-gravitation, but neither has a clear boundary. --Tango (talk) 10:18, 11 May 2009 (UTC)[reply]
As I recall, the apparent sharp "surface" is an optical illusion caused not so much by decreasing opacity as by the changing index of refraction. Supposedly in this situation there is a discontinuity, a well-defined limiting angle from which light rays can appear to come. I've never actually worked through this in detail. --Trovatore (talk) 09:08, 12 May 2009 (UTC)[reply]

Lightning Photographs

hi,

how do people take pictures of lightning? I mean its obviously not someone with super quick reactions, so how is it done? Constantly taking photos during a storm and hoping to get something? Computer detectors that can take a pic instantly?

thanks, --84.64.16.41 (talk) 20:35, 10 May 2009 (UTC)[reply]

Long exposure with a pinhole aperture? —Tamfang (talk) 20:40, 10 May 2009 (UTC)[reply]
Lots of photographs of empty sky will eventually yield a few photographs with good lightning. Especially in the era of digital cameras, this is becoming increasingly common. I know some people who attempt to do automated shutter triggering, but this is difficult. Nimur (talk) 20:55, 10 May 2009 (UTC)[reply]
All the pictures on lightning seem to have been taken at night, which supports the "long exposure" theory. (If you take a long exposure shot during the day it would just be overexposed all over unless you had a special lens with a minute aperture.) --Tango (talk) 21:08, 10 May 2009 (UTC)[reply]
Actually, I'm not sure even a really small aperture would work - it would probably have to be so small as to not pick up the lightning. The key things are the difference in brightness between the sky and the lightning and the length of time the lightning is there relative to the length of the exposure. The latter has to be very short if you don't want to have to take a crazy number of shots before you get a good one (if you have a 1/100 second exposure, you would probably have to take thousands of shots, if you have a 1s exposure you might be able to get away with a few dozen during a big storm - the exposure being 100 times longer essentially makes the lightning 100 times shorter). If the latter is very short, then the former needs to be very large to make up for it. --Tango (talk) 21:13, 10 May 2009 (UTC)[reply]
Yes that's right, all the really spectacular pictures are taken with long exposure, that is, fix the camera down and lock the shutter open. Not much good for anything other than to look pretty as often there is more than one strike in the shot (but makes for a better picutre). It is perfectly possible to capture lightning with a hand-held camera and a "normal" shutter speed, I've seen it done several times, but be prepared for a lot of failures. With high speed cameras it is possible to capture the ascender strike followed by a frame of the descender. This is more scientifically useful, but not so pretty. SpinningSpark 21:27, 10 May 2009 (UTC)[reply]
If you wanted to do that, though, wouldn't you use a high speed video camera? --Tango (talk) 23:30, 10 May 2009 (UTC)[reply]
Lightning strikes up?--KageTora (영호 (影虎)) (talk) 00:27, 11 May 2009 (UTC)[reply]
See Lightning#Leader formation and the return stroke. --Tango (talk) 00:52, 11 May 2009 (UTC)[reply]
You could "just" set up a digital video camera, aim it at the sky in night or daylight, and push "Record", then grab the one or two or three frames that recorded the strike. By digital video camera, I don't mean MiniDV or anything that records in PAL or NTSC, because the frame will look awful; but rather a camcorder that records to an MPEG stream of some sort. Tempshill (talk) 02:33, 11 May 2009 (UTC)[reply]
I've only shot lightning at night, as others have discussed above, but there are lightning triggers that will detect the lightning flash and release the camera's shutter. These can be used in daylight. Here's an example, no endorsement implied. I've also seen plans based on an Arduino. -- Coneslayer (talk) 12:08, 11 May 2009 (UTC)[reply]

May 11

Quanta

Reading A Brief History Of Time and hearing from other sources, it says that even observing a quantum particle changes it. What is all that about? Does it mean the light hitting it from an electron microscope changes it? Or is there some other mechanism involved? Also, how do they know this, if they didn't see what it was like before they observed it (logical to me)?--KageTora (영호 (影虎)) (talk) 00:32, 11 May 2009 (UTC)[reply]

Well, that might not be strictly true. If you set up an electron to have spin 1 on any axis and measure spin along the same axis, there should be no change in the state of the electron. It is of course a bit pointless to measure something that you already know, but it disproves the assertion nevertheless.
On the second question: Simply setup a repeatable experiment where you can make two measurements. Run the experiment 100 times and measure both things, then run it 100 times and measure only the second thing. If the results of the second measurement differ significantly between both batches, the first measurement had an effect. —Preceding unsigned comment added by 84.187.75.89 (talk) 01:50, 11 May 2009 (UTC)[reply]
I don't think that works. Even if you measure something you already know, your measurement still changes it. The spin after you measured it is now in an unknown state. Nimur (talk) 06:01, 11 May 2009 (UTC)[reply]
"Observing a quantum particle changes it" means that (some) measurements don't commute, in other words if you do measurement A and then measurement B you get a different outcome for B than if you'd done B first. Since it's a statistical effect you have to repeat the experiment many times (on identically prepared systems) in order to see it. So I think anon's description was exactly right. It's also true that measuring something you already know doesn't affect the system (at least, quantum mechanics doesn't say it has to). -- BenRG (talk) 12:29, 11 May 2009 (UTC)[reply]
(EC) The book is probabily talking about wave function collapse. Your interpretation of it as being the effect of the light on the particle being observed is a real phenomenon usually refered to as observer effect. These are two different things. The second one happens wheather you take quantum effects into consideration or not. It even happens in other situations that have nothing to do with physics. See for instance observer's paradox. The wave function collapse is a much more misterious phenomenon in which a particle which is in a state of superposition of several pure states suddenly jumps into one of those states when it is observed. To this day there is still considerable debate about what exactly happens when the wave function collapses. But there is no doubt whatsoever that it happens. As to the second part of your question, the particle being observed may have been carefully prepared so that they know beforehand what state it is in, and then when the experiment is performed it is verified that the particle indeed jumped into one of the possible states compatible with its original state and the experiment performed. Dauto (talk) 02:07, 11 May 2009 (UTC)[reply]
There's no doubt that the model with the collapsing wave function works, but a lot of doubt that particles suddenly jump into the measured state just as you measure them. The only thing you can determine experimentally is that the system was prepared to be in one state and later measured to be in another; what happened in between is a matter of interpretation. In path-integral quantum mechanics there isn't anything that looks like wave function collapse, and the spreading out looks symmetric in time—the maximum indeterminacy is midway between preparation and measurement. That matches the experimental reality better. If you have a particle emitter and a particle detector at opposite sides of a room, you can investigate what sort of changes to the room will affect the rate of particle detection (e.g. adding a barrier or a block of glass), and the answer (both in theory and in practice) is that the rate will only be affected if you put things where the path-integral formalism says the particle "is", not in the much larger area that the wave-function-collapse picture says it "is" just before detection.
The other reason to be suspicious of the picture with the spreading out and the collapse is that it looks like classical probability. There's always some uncertainty in the preparation of a system and that uncertainty increases with time (for example, an uncertainty in momentum leads to increasing uncertainty in position). But when you measure the position several times in quick succession you obviously should get roughly the same answer each time, no matter how much the position had previously "spread out" in your model. In order for the model to predict that correctly, you have to "collapse" the system's position in the model after the first measurement, and the mathematical process for doing that looks just like the mathematics of the quantum collapse (a Bayesian update). In the many-worlds formulation of quantum mechanics that's exactly what the collapse is, a Bayesian update with no objective effect on the world. The objective effect of measurement that people attribute to the collapse is actually caused by quantum decoherence.
In short, the situation is far too complicated and ambiguous to conclude that wave function collapse is physically real. Given how much it looks like a Bayesian update and how little it looks like other physical laws, I'd bet pretty strongly that it isn't. -- BenRG (talk) 12:29, 11 May 2009 (UTC)[reply]
Yes, I agree with that point of view. But that's still not a universally accepted interpretation of what happens at the wavefunction collapse. My statement intended to convey the fact that there is still dabate about that specific point. I think the term "Wave Function Collapse" is here to stay even if the decoherence interpretation eventually becomes universally accepted. Dauto (talk) 16:06, 11 May 2009 (UTC)[reply]
(Incidentally - an electron mictroscope uses electrons - not light - to image its target. Not that this changes the answers.) But basically, yes - there is no way to measure something without somehow affecting it. For large scale objects, the consequences are negligible - but for things like electrons, the consequences are significant - just one more weird thing that happens at the quantum level. SteveBaker (talk) 12:14, 11 May 2009 (UTC)[reply]
(Except that the rules do permit some measurements to leave the system unaffected—for example, a remeasurement of a property that you already know, but there are also less trivial examples.) -- BenRG (talk) 12:29, 11 May 2009 (UTC)[reply]
If you really want to enter into the "was the information really there before we measured it" rabbit hole, the most interesting places to start are EPR Paradox, Bell's theorem, and, somewhat differently, my favorite, Wheeler's delayed choice experiment. --140.247.10.164 (talk) 14:20, 11 May 2009 (UTC)[reply]
Clearly there is a high probability that Schrödinger's cat will eventually be mentioned somewhere in this thread. In fact, it was in a superposition of mentioned/not mentioned states - until I just mentioned it, which collapsed its wave function ... Gandalf61 (talk) 14:43, 11 May 2009 (UTC)[reply]

Hypocenters

I know that you can calculate the depth of a hypocenter by p waves, but is it possible to calculate the location of a hypocenter? —Preceding unsigned comment added by 174.6.144.211 (talk) 02:00, 11 May 2009 (UTC)[reply]

Yes. Dauto (talk) 02:23, 11 May 2009 (UTC)[reply]

how? —Preceding unsigned comment added by 174.6.144.211 (talk) 02:54, 11 May 2009 (UTC)[reply]

By comparing the time of arival of seismic waves at different seismographs around the world you can find out which seismographs were closer and which ones were further away and therefore reconstruct where and when the earthquake happened. Dauto (talk) 04:21, 11 May 2009 (UTC)[reply]
See hypocenter and particularly epicenter#Epicentral distance for an explanation. Astronaut (talk) 17:55, 11 May 2009 (UTC)[reply]

Chimera

In the Chimera dab page, somebody took out an entry for the wrong reason. My knowledge of genetics is pretty limited, so it's just possible that there is already an entry there that covers the same topic. The two items are:

  • Chimera (protein), a hybrid protein made by splicing two genes (still there)
  • Chimera (EST), a single cDNA sequence originating from two transcripts

Advice? Clarityfiend (talk) 03:26, 11 May 2009 (UTC)[reply]

I think you misunderstand the purpose of disambiguation pages. They are not intended to provide a comprehensive list of all of the meanings of a word - that's the job of a dictionary - and for that we have Wiktionary. The purpose is (as its name implies) to remove ambiguity when there are multiple articles with the same (or very similar) name. Since there is no article Chimera (EST), there is no ambiguity - so there does not need to be a DAB entry for it. Now, if you are arguing that there SHOULD be such an article - then go write one and add it to the DAB page. But until that happens, it's perfectly OK to remove the entry in the DAB. SteveBaker (talk) 12:06, 11 May 2009 (UTC)[reply]
A red main link is not a valid reason to delete a dab entry (see MOS:DABRL). Granted, the bluelinked article that went with it didn't refer to "chimera" as the guideline requires, but there are sufficient ghits both inside and outside of Wikipedia to make me think the topic is more than just a dictionary definition. If the two dab items refer to the same concept, there's no problem. However, if they are different, a related bluelinked article might be found or chimera might be mentioned in cDNA. Clarityfiend (talk) 21:22, 11 May 2009 (UTC)[reply]
I don't understand where you're coming from with that. MOS says: "A link to a non-existent article (a "red link") should only be included on a disambiguation page when an article (not just disambiguation pages) also includes that red link." - so is there an article out there (not a dab) that links to "Chimera (EST)"? If not, then the MOS clearly says that it shouldn't be in the DAB page. If I do a search on "Chimera (EST)" and click on "all pages that link to "Chimera (EST)"" in the search results - there is a talk page and two entries for the ref-desk. No articles. Hence the DAB entry most certainly should have been deleted per MOS:DABRL. What's to argue about? SteveBaker (talk) 12:10, 12 May 2009 (UTC)[reply]
What's the problem, Steve? I came here looking for an expert's opinion on whether this was something that should be salvaged. All I've gotten from you is a botched search and a flippant suggestion. Well, guess what? I've taken you up on that. Thanks for your "help". Clarityfiend (talk) 05:11, 13 May 2009 (UTC)[reply]
That was a rather rude reply. Steve was correct: there was nothing to link to, hence, no reason for it to be included on the DAB page. You've made an article now, so there's a good reason for it to be on the DAB page. Note also that Steve was talking about a search on Wikipedia itself, not a Google search, so your "botched search" comment is out of line. — The Hand That Feeds You:Bite 13:04, 13 May 2009 (UTC)[reply]
If you'll notice, my search was restricted to Wikipedia, plus I had already specifically stated that I found "sufficient ghits" in Wikipedia, so I stand by my characterization. Steve apparently missed the whole point of what I was after, and I found the tone of his responses to be rather unworthy of him.
As it turns out, I appear to have gotten a bum steer from Spinningspark as well. After reading the other article and based on what I've learned, I conclude that it is the same concept, necessitating the blanking of my work in favor of the more complete article. A real fiasco all around. Clarityfiend (talk) 20:58, 13 May 2009 (UTC)[reply]
I don't know much about genetics either, but they are clearly two different things; one is protein and the other is DNA. Also expressed sequence tag is probably a better target for the blue link as this is the subject that gives rise to Chimera (EST). But as Steve says, the red link has no incoming links from other articles and none of the potential blue links discuss it. Having an entry on the dab page only misleads readers into thinking Wikipedia has information when there is none. SpinningSpark 22:16, 12 May 2009 (UTC)[reply]

"God probably doesn't exist"

Buses in London carried for some time ads endorsing atheism. They said - "God probably doesn't exist". Does it make sense to say "probably" in this case? Isn't this a case where we cannot apply any meaningful statistics? PS: this question is not about the existence of God, so don't hijack the topic.--Mr.K. (talk) 15:54, 11 May 2009 (UTC)[reply]

You can't apply frequency probability, you can apply Bayesian probability. --Tango (talk) 15:57, 11 May 2009 (UTC)[reply]
(after edit conflict) I think you can get quite close to a frequentist probability. Culturally, there have been many gods (Zeus, Ra, etc). If you're willing to accept that Zeus doesn't exist (as most people will) then you can simply count the number of gods, and count the number that you're quite sure don't exist. This gives you a low estimate of the probability that a given god doesn't exist. I'm not saying this is valid reasoning, but it is a way to apply frequency probability.
You can also generalize this to all instances of a particular type of claim, which I think is closer to what people actually do and explains the connection with the informal use of the word "probably" here. As you go through life, you encounter various types of claims, and for many you eventually decide whether they're true or not. For instance, you will encounter many spectacular claims for which there is no strong evidence, ie. claims that require faith. And you may be undecided on some and decide to reject others. This gives you a probability distribution over all such claims.
This, I think, is more or less why many people will dismiss things like ESP, horoscopes etc. offhand, without careful scientific investigation. A thorough investigation would cost far too much energy, so they rely on their internal probability model (ie. experience) to reject these things.
It is in this sense that atheists might say that God "probably" doesn't exist. Their experience tells them that claims of that nature often turn out to be untrue (by their personal concept of truth), and they judge subsequent claims by that experience, in this case because there is no way to achieve a scientific level of certainty.
I should also point out that in this case the odd use of the word "probably", is exactly what lends the phrase its humor. Much like Carlsberg tag line: "Probably the best beer in the world". risk (talk) 16:24, 11 May 2009 (UTC)[reply]
I think the example with Zeus and others is not appropiate. They were deities, but they were not omnipotentt beings, creators of the universe, etc. All the so called "gods" from Greek and Roman mythology were just like some superhumans. The current concept of God is that of an omnipotent force or being, who or what designed the laws of physics. I think the question is not about weather he/it exists, but how we interpret it. These advertisments however don't seem to address questions about neither philosophy nor probability, they seem to be just a provocation, just like gay pride parades.--131.188.30.106 (talk) 08:59, 12 May 2009 (UTC)[reply]

The poster's a nice piece of provocation, and is a response to a similar campaign from the opposite camp. Note that according to I think therefore I am, the bus doesn't exist either. --Dweller (talk) 16:06, 11 May 2009 (UTC)[reply]

Since when has me existing implied buses don't? --Tango (talk) 16:15, 11 May 2009 (UTC)[reply]
I disagree; the logic of cogito ergo sum merely implies that while I definitely exist, the bus might not exist. It makes no more concrete statement as to the degree of certainty of the existence of the bus; except that it is less definitely extant than I am. Nimur (talk) 18:19, 11 May 2009 (UTC)[reply]
Furthermore, I think the cogito ergo sum reasoning proposes several "equally likely" scenarios; one possibility is that the entire universe also exists, and is as it seems; another possibility is that none of it exists, but it appears as such because some entity (God?) is deceiving me; or alternatively, that I am deceiving myself (and that I am the entire universe, including God). My personal belief is that the first case is the most interesting one. This case must be true. If it isn't, then none of you exist, so your opinions of what is true or interesting are irrelevant; and I can single-handedly decided what is true and correct anyway. Nimur (talk) 18:23, 11 May 2009 (UTC)[reply]

<-Chaps, it was a joke, based on the fact that buses can't think. Never mind. --Dweller (talk) 20:30, 11 May 2009 (UTC)[reply]

My English could be failing me but isn't "God probably doesn't exist" actually a categorical statement that "God does not exist in a probable manner" rather than a dubious statement on his (her) existence? Isn't that a truism, since whether or not God exists it could only be in a definite manner? I seem to remember getting sentences starting "Hopefully, xyz" marked through in red at school for this reason....--BozMo talk 16:17, 11 May 2009 (UTC)[reply]
I think that would be "God doesn't exist probably"(note the lack of a comma), or better yet: "God doesn't exist probabilistically". I suppose particles in quantum mechanics can be said to exist probabilistically, so the case could be made. risk (talk) 16:29, 11 May 2009 (UTC)[reply]
Acording to the linguist Steve Pinker there's nothing wrong with saying "Hopefully, xyz". I agree with him. Dauto (talk) 17:43, 11 May 2009 (UTC)[reply]
I think he is at odds with Fowler's_Modern_English_Usage which has to be the higher authority. As you say he is only a linguist not a grammarian, although his book the language instinct is in my top ten books of all time.--BozMo talk 18:19, 11 May 2009 (UTC)[reply]
Which article in Fowler? (For those who don't know, it's notoriously ill-organized.) — All I remember of The Language Instinct, a few years after reading it, is vague disappointment. —Tamfang (talk) 01:22, 13 May 2009 (UTC)[reply]
There's an article, needless to say. --Sean 20:12, 11 May 2009 (UTC)[reply]
However the article seems to be entirely about US usage whereas we were discussing a London bus I thought. Misuse being widespread in the US isn't really relevant or unusual. It is however very difficult to care about it. --BozMo talk 21:03, 11 May 2009 (UTC)[reply]

The use of 'probably' is not in the mathematical sense, it is in the everyday sense of meaning 'most likely'. They're not saying they've done some stats and the probability says X, rather they are saying it is 'most likely' that god doesn't exist. Their reasoning will be based on a huge multitude of factors, not least a lot of what modern science has uncovered about the beginnings of the universe. Anyhoo the point is - this is more likely a advertiser looking for a 'catchy' comment than a statement of fact/reasoning. ny156uk (talk) 16:39, 11 May 2009 (UTC)[reply]

The bus ad uses a rhetorical device that can be called "amplification of slight doubt" that has a venerable tradition in an English operetta:
CAPTAIN: I am never known to quail
At the fury of a gale,
And I'm never, never sick at sea!
CHORUS: What, never?
CAPTAIN: No, never!
CHORUS: What, never?
CAPTAIN:"Hardly ever! Cuddlyable3 (talk) 17:06, 11 May 2009 (UTC)[reply]

The inclusion of "probably" was, I believe, required to allow the advertisement to meet the requirements of the ASA that absolute claims made must be verifiable. To omit it would require proof that God does not exist. Bazza (talk) 13:18, 12 May 2009 (UTC)[reply]

Actually in a radio interview the instigator of the ads said the word "probably" was included to be inclusionist about the atheists supporting it. Many were not "fundamentalist atheists", who contend that it is utterly inconceivable that God exists, but were more moderate atheists who had concluded that the existence of God was unlikely. (If you think about the other unsupported claims that are allowed in adverts then the ASA theory isn't likely). DJ Clayworth (talk) 17:22, 12 May 2009 (UTC)[reply]
It is worth noting that a complaint was made to the ASA and rejected. I don't know if the presence of the word "probably" was significant in that decision, but I doubt it. --Tango (talk) 17:48, 12 May 2009 (UTC)[reply]
In the UK it is actually illegal to place in an advert a statement that you cannot prove. Hence, the 'probably' in the Carlsberg Beer adverts.--KageTora (영호 (影虎)) (talk) 01:09, 13 May 2009 (UTC)[reply]
There is an exception made for things that are obviously subjective or obviously not intended to be taken seriously. "Joe Bloggs is ugly" cannot be proven, but I could say it in an advert because no reasonable person would think it was supposed to be a statement of fact. I can also exaggerate wildly as long as it is clear to any reasonable person that I am exaggerating. I think the God statement is acceptable under the former exception. --Tango (talk) 11:03, 13 May 2009 (UTC)[reply]


In any case the Carlsberg thing is less than convincing as it doesn't seem to address the issue. If they're banned from stating outright that they have the best beer because they can't prove it, well, how exactly do they plan to prove that they have "probably" the best beer? --Trovatore (talk) 20:20, 13 May 2009 (UTC)[reply]

Does half the voltage mean half the power?

Does the same lightbulb in a country with a 110 V standard burn half as bright as in a country with 220 V? And does the same go for a battery charger? Will it take twice as long to charge batteries with 110 V instead of 220 V? DirkvdM (talk) 19:03, 11 May 2009 (UTC)[reply]

Actually, if you assume that the lightbulb's resitence is Ohmic The power drawn at 110V should be four times smaller from the formula . In reality the bulb's resistence dependes quite a bit on its temperature which influences the result above. If the battery charger is designed for both 110V and 220V which is quite common nowadays, there should be no change on the time taken to fully charge the battery. Dauto (talk) 19:13, 11 May 2009 (UTC)[reply]

That power formula is for direct current. I believe all countries use alternating current. The power is based on voltage and current. Without knowing the current, it is impossible to make a judgement about power. See AC power. -- kainaw 19:16, 11 May 2009 (UTC)[reply]
The formula I presented works for both direct current and alternating current, provided that the load is an ohmic resistor. As I pointed out, that's not the case here. Dauto (talk) 21:26, 11 May 2009 (UTC)[reply]
(edit conflict with both above) A simple resistive load will draw 1/4 the power when connected to 110 V, as compared to 220 V; see Electric power. But neither of the devices you mention are simple resistive loads. The resistance of a light bulb's filament depends on its temperature, which (circularly) depends on the power it's radiating. See Lamp rerating for some guidelines. A battery charger like this one is a complicated microprocessor-based device, which is in no way a pure resistive load. The one I linked to is designed to accept either input voltage, and its power supply likely uses approximately the same power in either case (drawing twice the current at 110 V, compared to 220 V). It would be designed to apply the same charging current to the batteries in either case, and hence take the same time to charge them. -- Coneslayer (talk) 19:17, 11 May 2009 (UTC)[reply]
A light bulb filament, whether tungsten or carbon, is not an ohmic resistance. When the temperature changes, the resistance changes, quite dramatically. The source [5] indicates that a tungsten filament at half the rated voltage passes just under 2/3 the current it passed at full voltage. The resistance from the chart would be about 2000 ohms at full voltage and about 1530 ohms at half voltage, for the particular bulb referenced. (Milage may vary). Edison (talk) 02:54, 12 May 2009 (UTC)[reply]

What are the benefits of a tree structure?

The Transhumanist    20:22, 11 May 2009 (UTC)[reply]

A tree diagram can show more levels of hierarchy than other types of chart. NeonMerlin 21:04, 11 May 2009 (UTC)[reply]
What are the benefits of showing more levels of hierarchy? The Transhumanist    01:35, 13 May 2009 (UTC)[reply]


In computer software, it seems that a vast number of types of data are best stored in a tree-like structure (or one of it's generalizations or specializations). Why this is, is hard to say - but it pops up all over the place. Take, for example, a phone book. A long list of names with associated numbers. Real phone books are essentially just linear arrays - but when you store a phone book on the computer, it turns out to be handy to use a tree structure. The 'root' of the tree might have 26 branches - one for each letter of the alphabet. Each one of those has 26 branches - and so on for perhaps 100 levels corresponding to the longest name + address in the phone book. At the leaves of the tree, we place the phone number that corresponds to the name you spell out as you 'walk' along the tree to get to that leaf. Obviously, the leaf that you get to by following XQWZXGKWRTQ doesn't exist - so we 'prune' the tree to keep only the branches that actually lead somewhere. You'd obviously want branches for numbers and space characters and stuff - but the idea is basically there. Now, when you ask the computer to look up a phone number, it just follows the branches letter by letter and in just a couple of dozen choices, it finds the phone number. Compare that to the number of names you have to look at to find an entry in a paper phone book! Furthermore, when someone new has to be added to the phone book, it's easy to create the extra branches and tack on another leaf. Doing that with a simple linear array would require that you shuffle a large amount of data down the array to make a space for the new name. Furthermore, while you're doing that, the computer can't look up any more phone numbers...but with a tree, one part of the program can be adding or deleting entries while another part is looking up phone numbers. Tree structures are VERY cool. SteveBaker (talk) 02:13, 12 May 2009 (UTC)[reply]
OK, so we have:
  1. Branching for efficient (faster) navigation
  2. Scalability for ease of expansion
  3. Modularity to enable multi-tasking
Humans using a phone book (a linear array) still have the benefit of "branching" because they can skip straight (or close enough) to the letter the name they are looking for starts with, etc.
If the phone directory is on rolodex cards (still a linear array), it's scalable, as the user can add cards anywhere in the collection.
Such a phone directory is modular, each index card being a separate module, allowing two people to use or work on the index at the same time. (E.g., one person can update the "M's" while you use the rest of the index).
What benefits are there to having a knowledge tree that is used directly by humans, say, on a word processor or on sheets of paper or in a published printed book? The example in the article on tree structure is an encyclopedia. Let's say that example was complete, with the whole encyclopedia rendered into a tree and published in a book or on a webpage. What would the benefits of that be?
The Transhumanist    01:34, 13 May 2009 (UTC)[reply]
Tree structures don't occur very often in the physical world - I'm having a hard time thinking of any, actually. A phone book with tabs for each of the letters A-Z is a kind of super-primitive one-level tree. The chapter list at the front of a book is somewhat like that. I guess in big technical documents, people sometimes do deep chapter/section/sub-section/sub-sub-section numbering - and then the index is a tree of sorts. But trees inside computers are typically dynamic, vibrant things - where data is added and removed easily and (often) frequently. I can't think of a physical analog of that. The lack of a physical analog of a tree structure means that these are not things that people are generally used to using. The file system on your PC is a tree structure - folders inside folders with files inside that - but aside from the initial structure of "My Documents/My Pictures", etc that is imposed upon you - most people have all of their files in gigantic, ugly flat lists. I don't think there necessarily IS a huge benefit for humans to deal directly with tree structures...in my mind they are the things of computers - and for that they are often ideal. Humans seem to prefer a structure like the Wiki or the World Wide Web where there is an arbitary mesh of connections going every which way. SteveBaker (talk) 02:22, 13 May 2009 (UTC)[reply]
"Tree structures don't occur very often in the physical world" - Steve, think again! Tree of Life, plain Tree, of course, nearly all river systems,... --Stephan Schulz (talk) 02:40, 13 May 2009 (UTC)[reply]
The tree of life is not a structure in the physical world - it's an artificial construct for storing data - and even that is pretty controversial because of the issue of genes crossing from one species to another. "The Tree of Life" should really be called "The Directed acyclic graph of Life"...but that doesn't trip off the tongue quite so well! Real trees are (of course) shaped like "tree structures" (although they are upside-down compared to what most computer geeks have in their heads) - but they don't organise information - and humans don't interact with them as tree-structures. We don't often start at the trunk of a real tree and follow the branching pattern to a specific leaf. I maintain that there is no physical analog of a computer's tree structure that humans interact with in that way. SteveBaker (talk) 14:11, 13 May 2009 (UTC)[reply]
Actually, there are more than one: Family trees, taxonomies, and organizational charts. The base structure of mind maps and concept maps are hierarchical. But the type of trees I'm most interested in are outlines. Such as Britannica's Outline of Knowledge (in its Propaedia, which includes over 700 outline sections). The Dewey Decimal Classification is a tree structure and de facto outline of knowledge, and so is the Library of Congress Classification. And of course Wikipedia's Outline of knowledge (which is likely to surpass the others in breadth and depth very soon), includes about 500 component outlines such as Outline of health, Outline of history, Outline of philosophy, Outline of robotics, Outline of Japan, Outline of Alabama, etc.). It will soon include hundreds more.
I've been having a hard time putting our outlines' benefits to readers over and above regular articles into words, but the input of editors such as yourself is helping a lot. Thank you.
The scalability and modularity benefits apply to articles in general, including outlines. But Wikipedia's outlines certainly take advantage of branching to a greater extent than regular articles, and therefore they are faster to use for navigation. They also show the structure of a subject, that is, they identify a subject's subtopics and show the relationships between those topics in a way that is easy and quick to view and comprehend. The relationships are metaphorically familial, that is, possessive (what belongs to what, like family ties). The relationships are shown through the arrangement of "parent", "offspring", and "sibling" topics. Articles do explain the relationships between their subtopics (sometimes), but an article's prose takes longer to read than an outline's hierarchy of topic names.
Is that description easy to understand? How can it be improved?
Any more observations about the benefits of tree structures, including outlines, would be most appreciated.
The Transhumanist    01:44, 14 May 2009 (UTC)[reply]
Why do these need to be separate articles though? For 'top level' topics like these, you can still have a conventional, readable article and put your 'sea of links' in a subsection. Take a look at two that I worked hard on: Computer has a large section at the bottom called "Computer#Further_topics" which tabulates practically every other Wikipedia page about computers into a handful of tables with links broken out into sections and subsections. I did the same thing for Automobile - and subsequently, the tables got moved out into separate template files what are shrunk down into one like "show" blocks in the "Automobile#See_also" section. These are tucked in to the bottom of the articles - which still read nicely - but which have shorter sections that summarize second tier articles that show up as links to more detailed articles: eg "Main article: Petrol engine". This means that when someone doesn't know that there is the concept of an "Outline" document (which 99% of readers won't - no matter how well you promote it) - they'll find the "main" article by typing in a really obvious search term like "Computer" - and arrive at a simple, easy-to-read article - which may be enough. If they find a section that interests them - then, right there, they have a link to a Main article that expands that short section into another full article. The "Further topics" section at the end contains the most links possible in the smallest space - but sorted into areas of interest. So if you're looking for articles about careers in computing-related fields, you'll find a table of "Computer-related professions" with a bunch of links that are split into hardware and software jobs. Another way to organize this vast amount of linkage is by packing them into a navigation box with expandable sections. I helped set one up that way for Video game industry - look at the box at the top-right of all video game industry-related articles and you'll find this standard box that cross links to every other article on the subject that we have. Other top-level articles such as Physics have taken a hybrid approach using both a top-right-corner "Nav Box" with expanding sections (just like Video game industry does) - AND a set of expandable sections at the bottom of the article (just like Automobile does). Notice that (for example) all of the 'top level' articles on the natural sciences have the same set of cross-links to other fields in that same expandable box format.
IMHO, any one of those three techniques is better than having a separate "Outline" article - which is harder to find and requires extra mouse clicks to navigate to. Better to pack that stuff into the bottom of every relevent article as an expandable section and cut out a bunch of unnecessary extra navigation. Also, packing the lists of bulleted links into a table allows for people to find the link they are looking for with much less scrolling up and down the page. If all you have is a list of links - then categorize them and pack them into a table in alphabetical order. SteveBaker (talk) 04:26, 14 May 2009 (UTC)[reply]
The benefits of a tree structure are twofold: First, it's easier for beginners. Most writing courses teach people to make an outline first, and it's a good way for them to start out. Second,, I tend to think and write that way myself--possibly due to some experience in intercollegiate debate. I always list things, mentally at least, before I write or speak about them. But there's two benefits of having separate the outline articles separate. First, so that different people can do it their own way. There's a corollary of NOT PAPER: we're a BIG encyclopedia. The second is that a pure tree structure with collapsable boxes hinders universal accessibility, or so I've been told, because reader programs cannot follow it. And I've learned this from my work as a librarian: however you organize things, someone will want them differently. They don't usually have the economic possibility of multiple methods--though even they try, because the subject headings are deliberately different from the classification on the shelves. DGG (talk) 15:18, 14 May 2009 (UTC)[reply]


Thank you for the feedback Steve, this is exactly what we need so that each design element is well considered.
Outlines are separate pages for a number of reasons. Outlines are integrated (connected) into a whole, which is harder to do (and confusing) if they are article subsections, to enable easy browsing of the tree of knowledge itself. Formatted tables are much more labor intensive (slower), both to build and maintain. Tree structures are simple and they are easily scalable.
To include lists in articles in the way you propose, would require rethinking and changing many of Wikipedia's fundamental guidelines.
"Further topics" is synonymous with "See also", and falls under the WP:SEEALSO guideline. Someone is bound to notice this and consolidate the two sections. See also sections are subject to moderation (i.e., being moderate), and they aren't intended to display redlinks. Links already included in the body of the article (in its text) are generally not repeated in "See also" (which means "related but not covered here"), making that section inappropriate for outlining the subject.
The division of Wikipedia into separate articles and stand-alone lists is a fundamental design feature of the encyclopedia. Embedded lists (including those at the end of an article) are intended to complement the prose of an article, not replace it. The purpose of articles is to explain. Therefore, lists of links (which are intended for browsing) should usually have their own entries. See WP:EMBED and WP:STAND.
Outlines tend to be long, and due to the size constraints of articles, "seas of links" (long lists) are not generally included in articles. See WP:SPLIT.
"Computer hardware" and "computer software" are major subtopics of "computer" and should therefore be covered in the Computer article. And because those subtopics are extensive, they are subject to Wikipedia:Summary style. Therefore, the hardware and software sections of the Computer article should be truncated versions of the articles of the same names.
I've done some experimentation with using navigation boxes in place of sublists in outlines. Those outlines aren't as convenient to use because you can't scroll down and "scan" all the contents. Clicking on show links to expand navigation templates is inconvenient and time-consuming, and disrupts scanning.
It takes one click to navigate to an outline from an article on the same subject. A subject that is chopped up into several collapsed navboxestakes several clicks to view.
Lack of awareness of outlines is a problem, but currently, it is a problem that is deliberate. To give the design team time and latitude in refining a core of the Outline of knowledge (and standards for its pages), we haven't heavily "promoted" the OOK nor integrated it extensively into the encyclopedia via links. To support the development of outlines, a guideline and explanation, and a projected model (showing future branches to be developed), are being drafted at this time. If we were to heavily promote or link to the pages of the OOK before the support pages were ready, we wouldn't be able to handle the volume of queries from confused editors. Chaos would reign.
To say that 99% of readers will never be aware of Wikipedia's outlines regardless of what we do, is sheer "can't be done" pessimism. They said the same thing about flying, and about sending a man to the moon. Sending readers to corresponding outline pages is much easier to accomplish. I'm confident our integration efforts will be successful.
Each of Wikipedia's navigation systems has strengths and weaknesses. But they also have a strength by coexisting: they're synergistic, playing an eternal game of leapfrog - in coverage and innovations. See WP:CLN for details.
The Transhumanist    19:51, 14 May 2009 (UTC)[reply]

Mutants leaving home to adapt

In what species, if any, have mutants been known to leave their home environments for places to which they were better adapted? NeonMerlin 21:03, 11 May 2009 (UTC)[reply]

Er, humans? David Vetter, for example. Rockpocket 21:31, 11 May 2009 (UTC)[reply]
Actually, Vetter was born in that environment, so perhaps Ted DeVita would be a better example. But there are lots of less dramatic examples of this: people with albinism moving to countries with less sunlight or people with arthritis moving to countries with a warmer climate. Rockpocket 21:41, 11 May 2009 (UTC)[reply]
Zoonoses must have done so at one point. Some species are rather polymorphic and thus may utilize different habitat niches. It's more likely that the "conquest" of a new habitat by a population subgroup with an adaptive trait takes place gradually; it would be hard to pass on your genes if you had no one to mate with in your new home. Also it may take some generations of selection for the organism to refine its adaptations. I'll try to find a specific non-human vertebrate example. In the meantime you may want to see Biological dispersal and rafting event. Sifaka talk 22:53, 11 May 2009 (UTC)[reply]
More a hybrid than a "mutant", but certainly different from your average polar bear: Grizzly–polar bear hybrids might have different habitat preferences than their parent species, but there is no information due to them being very rare. Sifaka talk 23:10, 11 May 2009 (UTC)[reply]
One of his descendants looks like a good bet [6]. Can't give you a species name, though. Since lots of scientists think that the first lifeforms were anaerobic the first bacteria taking steps toward becoming aerobic bacteria would probably also qualify. 71.236.24.129 (talk) 03:41, 12 May 2009 (UTC)[reply]
One of whose descendants? (more direct link) —Tamfang (talk) 01:41, 13 May 2009 (UTC)[reply]

May 12

cold virus and cold water

we all know that common cold is caused by a virus. Then why do people say that drinking chilled water that is purified earlier will cause cold?--harish (talk) 01:18, 12 May 2009 (UTC)[reply]

  1. We don't all know this.
  2. We don't always act rationally on what we know
Old wives' tales and folk remedies (or folk ailments) are unlikely to be spontaneously extinguished in a fit of rationality. — Lomn 01:35, 12 May 2009 (UTC)[reply]
There is a quality of free association that one finds with pre-bacteriological understandings of disease. "Hmm, mild sicknesses appear to increase in the winter -> must be because of the cold -> other things that are cold must cause illness." --98.217.14.211 (talk) 01:53, 12 May 2009 (UTC)[reply]
"List_of_common_misconceptions#Health". Axl ¤ [Talk] 16:38, 13 May 2009 (UTC)[reply]

using computer and eye problems

The computer monitor is just like any other object in the environment. If we reduce the monitor brightness and use an anti-glare screen, the monitor will have the same brightness as other objects. We don't get eye problems if we gaze at a table for long hours. But if we do the same for a computer, why do we get eye problems?--harish (talk) 01:22, 12 May 2009 (UTC)[reply]

Gazing at anything for long hours may inhibit blinking which may lead to dry eyes and all the common related problems. What specific "eye problems" are you referring to? -- kainaw 01:38, 12 May 2009 (UTC)[reply]
You can get eye strain reading a book for too many hours in a row too. Holding something in front of your face and staring at it intimately for hours and hours is generally not great on the eyes. The problem with computers is that the time flies by pretty dang fast. I don't think it is a problem with brightness per se. --98.217.14.211 (talk) 02:57, 12 May 2009 (UTC)[reply]

By eye problems, i mean myopia or something like that. People warn that using the computer for long hours will result in wearing spectacles--harish (talk) 05:00, 12 May 2009 (UTC)[reply]

People say all sorts of things. Perhaps you should ask them for the evidence.--86.25.193.179 (talk) 05:51, 12 May 2009 (UTC)[reply]
And yet many people who do not use the computer for long hours also end up having to get spectacles... while many people who use computers every day don't. I don't think it's as simple as "don't look at X too much". -- Captain Disdain (talk) 08:26, 12 May 2009 (UTC)[reply]
You can definitely get eye-strain from staring at a monitor for too long - but that's a temporary condition, the muscles are simply getting tired from holding the same (short) focal length for hours at a time. You rest - they get better. Taking some time out every 10 minutes or so to stare out the window - or somewhere else as far away as possible - definitely helps. But I've been using computer screens in my job and at home ever since there WERE computer screens - and although I'm a little short-sighted, I can still pass the eyesight test for my driver's license without glasses. I don't think we have solid scientific evidence on this, either way, but it's certainly not as black and white as some people make out. SteveBaker (talk) 12:02, 12 May 2009 (UTC)[reply]
(anecdotal evidence) I've been using computers since I can remember (2-3 years old) and for LONG hours a day. That includes long hours with those old monitors with very low refresh rates, high radiation and so. Also, I've been reading compulsively both in the computer screen and in paper with poor illumination. Needless to say, I've been repeatedly told that I would damage my eyesight. Now I'm 23 and my eyesight is perfect. Even glasses with very slight correction are incredibly uncomfortable for me. Only when I was very young I remember that sometimes my eyes itched after playing with my old MSX for too long, but, for some reason, it happened like 3-4 times and never happened again. --Taraborn (talk) 13:33, 12 May 2009 (UTC)[reply]

Temperature of the Exosphere

A line in the recent Scientific American (sorry, I know, I'm just bored, and the website is so flashy...) states <a href="http://www.scientificamerican.com/article.cfm?id=how-planets-lose-their-atmospheres&page=2">thus</a>:

The temperature at Earth’s exobase oscillates but is typically about 1,000 kelvins, implying that hydrogen atoms have an average speed of five kilometers per second. That is less than Earth’s escape velocity at that altitude, 10.8 kilometers per second, but the average conceals a wide range, so some hydrogen atoms still manage to break free of our planet’s gravity.

1,000K at the exobase? Isn't that a little bit high? Is this a typo or am I missing something? 219.102.221.127 (talk) 03:40, 12 May 2009 (UTC)[reply]

May be this previous discussion gives some hint about the temperature issue. manya (talk) 04:26, 12 May 2009 (UTC)[reply]
See thermosphere. Looie496 (talk) 04:29, 12 May 2009 (UTC)[reply]

Ah, I see thank you. Darn, I missed it by one sphere. Maybe it should be added to the exosphere article that the temperature in parts can be extremely high, or at least that the fast moving particles of the exosphere have very high energies. 219.102.221.127 (talk) 05:26, 12 May 2009 (UTC)[reply]

dogs and grass

Why do dogs eat grass? Mine seems especially hungry for it in the spring. --Halcatalyst (talk) 04:01, 12 May 2009 (UTC)[reply]

Let me google that for you. :) manya (talk) 04:19, 12 May 2009 (UTC)[reply]
Many dogs are idiots. Tempshill (talk) 04:38, 12 May 2009 (UTC)[reply]
Many animals eat stuff not naturally considered part of their diet. Parrots eat clay, chicken eat pebbles, cats eat grass, birds eat various herbs. Some of these things have been found to help digestion or serve medical purposes. 71.236.24.129 (talk) 08:21, 12 May 2009 (UTC)[reply]
Interestingly enough, I used to have a pet duck that used to eat it's own feces after it had dried.--KageTora (영호 (影虎)) (talk) 01:05, 13 May 2009 (UTC)[reply]
<pedantry>'they'. the singular is fex.</pedantry> —Tamfang (talk) 06:01, 14 May 2009 (UTC)[reply]

Dogs, if they were still running in packs and actively hunting like wolves, would normally get fiber and veg proteins etc. by eating the intestinal tract of thier prey (partially digested grass and other stuff). I would suspect they eat grass to supplement this now missing fiber/vitamin source.67.193.179.241 (talk) 12:16, 12 May 2009 (UTC) Rana sylvatica[reply]

My vet said some dogs eat grass if they have indigestion; or Grassiscon. Or they may just like the taste of it. Lanfear's Bane | t 12:44, 12 May 2009 (UTC)[reply]
Cats often eat grass just before they are sick (presumably for a problem with digestion). Is there some scientific explanation? Would it work for human indigestion? (I prefer mints) Dbfirs 08:21, 13 May 2009 (UTC)[reply]
Many pet stores sell little pots of wheat or oat grass for cats to eat. (Our cats love the stuff!) So, I doubt that the stores are selling it with the intention that the cats throw up on their owners carpets, counter-tops, etc. Dismas|(talk) 10:01, 13 May 2009 (UTC)[reply]
Trouble can ensue if they don't throw up their hairballs. —Tamfang (talk) 06:01, 14 May 2009 (UTC)[reply]

Is the speed of everyones brain similar?

In the same way that computer CPUs can differ in speed, our brains could be thought of as having a clock speed of some kind. Clearly there is some speed, as in practical experience thinking does take time, it is not instantaneous. My question is - is this speed constant among humans - among westerners for example? Or do some people have brain-speeds that are say a half or a tenth of the average? Supplementary question - are there medical conditions or recreational drugs that can significantly slow down brain-speed? 78.145.21.210 (talk) 12:59, 12 May 2009 (UTC)[reply]

from the article myelin "The main purpose of a myelin layer (or sheath) is an increase in the speed at which impulses propagate along the myelinated fiber." It seems that the quality of the myelin varies see: http://www.medicexchange.com/Neurology-Brain-CNS/brain-images-reveal-the-secret-to-higher-iq.html --Digrpat (talk) 13:44, 12 May 2009 (UTC)[reply]
That Myelin article is I guess the closest you'll get to a hardware answer. My own experience is that I seem to have two speeds - very slow for thinking where someone says something and I don't have an answer ready and they're already on to the next thing before I open my mouth, and very fast where I can easily do things like catch a piece of paper dropped between my fingers. Yep I envy those people with a ready wit and repartee. Dmcq (talk) 15:19, 12 May 2009 (UTC)[reply]
I'm not sure clock speed is a good analogy. Whether or not one's brain chemically works at the same speed as others doesn't necessarily translate into speed of thought (associations, combinations of ideas, memory, mathematical operations—all of these are likely quite different neurologically). There are obviously some individuals who are really on the end of the bell curve in terms of some of these functions (John Von Neumann was reportedly a complete mathematical savant). I'm not sure there's any great way to measure the "speed" of the brain in general though. Keep in mind that even if you do measure the speed of a thought, it is not the same thing as the speed of your comprehension of the thought (which is another question altogether, neurological studies have shown—your brain knows things before "you" do). --98.217.14.211 (talk) 15:20, 12 May 2009 (UTC)[reply]
No. I am very slow witted. Some people are remarkably quick-witted, such as Paul Merton.--86.25.193.179 (talk) 16:14, 12 May 2009 (UTC)[reply]
As long as the brain is at normal temperature, the speed of the cellular components is fixed and the same for all people (it only takes a little bit of hypothermia to slow things down, though). But there is a concept of "processing speed" that operates on a higher level than the cellular, and possibly relates to intelligence. It isn't all that well-defined a concept, but you might look at this review for more information. There is a substantial amount of literature on the topic. Looie496 (talk) 16:15, 12 May 2009 (UTC)[reply]
Actually, clock speed might be a very good analogy. People often make the mistake of assuming the speed of a CPU is determined entirely by its clock speed, which is not the case, and the same mistake is being made here. The time it takes to solve a problem depends both on how many things you can do in a second and how many things you need to do. In terms of CPUs, a CPU can do one instruction for every tick of the clock, but different CPUs can do different amounts with each instruction. Some will have a special instruction to solve a given problem in one step while another would have to use general functions and do it in several steps. The same could be true of the brain - one person may be able to solve a problem quickly because they have a good method to do so while another person would take longer because their method is slower. Another factor in both brains and CPUs is memory, both the amount of working memory available and the amount of data already stored. A small child when asked to calculate "ten minus four" may take several seconds because they have to actually calculated it whereas I would answer in a fraction of a second because I have the answer stored in my memory having used it so many times before. The same happens with computers - really fast chess solvers often work by having the solutions to certain positions calculated and stored in advance (particular for the endgame when there are fewer possible positions). --Tango (talk) 18:09, 12 May 2009 (UTC)[reply]
There are people who speak very slowly. Are they slow witted? To some extent, this is a cultural thing. It may also be due to individuals copying others while growing up. Rural people and people from the American South supposedly speak slowly, though I have not noticed it. Some women speak very rapidly. I have heard Italian women rattle off Italian at amazing speed. - GlowWorm. —Preceding unsigned comment added by 98.21.108.29 (talk) 23:18, 12 May 2009 (UTC)[reply]
Italian needs to be spoken quickly — its information density per syllable is lower than that of English. Luckily, it can be spoken quickly, and understood, because its phonetics is extremely well-marked and unambiguous (for example, there are only seven vowel sounds, compared to I think around fourteen in English.) --Trovatore (talk) 23:24, 12 May 2009 (UTC)[reply]
That's an interesting concept, the information density of a language. I haven't heard of it before. Perhaps the speed develops over centuries by people taking short cuts in their speaking. It would be considered slangy or slurring at first, but when enough people do it, it would become respectable. It would be done because the speaker realizes the listener is a bit bored with long-winded speaking. GlowWorm. —Preceding unsigned comment added by 98.21.108.29 (talk) 23:35, 12 May 2009 (UTC)[reply]
No, I think it's the reverse: When you speak quickly you must speak very precisely, or you won't be understood. The "slurring" leads to high-information-density languages, spoken slowly. This is just my personal take on the matter — I haven't looked into the research on the matter, which I would expect has been well studied, but about which I don't really know anything from an academic point of view. --Trovatore (talk) 23:46, 12 May 2009 (UTC)[reply]
I have heard that a certain amount of "noise" (unnecessary words) is desirable in printed text to make it more comprehensible. This is sometimes done deliberately in children's books. - GlowWorm. —Preceding unsigned comment added by 98.21.108.29 (talk) 23:56, 12 May 2009 (UTC)[reply]
Even secondary-school books have a low information density. That makes them boring to brighter students. - GlowWorm. —Preceding unsigned comment added by 98.21.108.29 (talk) 00:07, 13 May 2009 (UTC)[reply]
OK, Trovatore. I get it now. Italian is spoken fast because it is low density now, not generations ago. That also necessitates few vowel sounds. - GlowWorm. —Preceding unsigned comment added by 98.21.108.29 (talk) 00:24, 13 May 2009 (UTC)[reply]

units

please help me with this question: if the units of force, energy and velocity are 10N, 100J and 5 m/s, find the units of length, mass and time.

(i just don't understand this, the unit of force is Newton. so what's 10N doing here?) please give me hints on how to solve this question —Preceding unsigned comment added by 122.50.131.1 (talk) 13:41, 12 May 2009 (UTC)[reply]

Is it that the question is asking you to suppose that something with that force and energy is travelling at that velocity. On that basis, it's asking you to work out how far it's travelled, what its mass is and how long it's taken to travel there. I have my reservations about this, because my science knowledge is sparse (and a more knowledgable contributor should be along soon) but I think you'd need one of "length" and "time" to work out the other. So perhaps I've misunderstood "length". It can't mean the length of the object, because unless there's more to the question than you've told us, it could be any length from just under infinitely long to just over zero length. --Dweller (talk) 13:57, 12 May 2009 (UTC)[reply]
It is a very confusing question, but here is my guess as to what it means. In SI (as well as in other systems of measurement), units of force, energy and velocity are derived units, defined in terms of the base units of length, mass and time, which are the metre, the kilogram and the second. But you could reverse this logic and make force, energy and velocity your base units, and then define length, mass and time in terms of them - for example, unit of length = unit of energy / unit of force (not a very practical approach, but possible in theory). So I think the question is asking this : in a system of measurement in which the base units are force, energy and velocity, and have magnitudes of 10N, 100J and 5 m/s, what are the magnitudes of the derived units of length, mass and time ? Gandalf61 (talk) 14:16, 12 May 2009 (UTC)[reply]
It's a horribly written question, but that interpretation seems to work. Allow me to explain it slightly differently in case people didn't get that. We define new units, Foo, Bar and Baz. Foo is a unit of force an is equal to 10N, Bar is a unit of energy and is equal to 100J and Baz is a unit of speed and is equal to 5m/s. If order to have a consistent system of units (so all the formulae we know still apply, for example F=ma, without needing to add in constants) we need to define new units of length, mass and time with particular conversions to SI units so that everything works out nicely. What should those new units be? --Tango (talk) 14:29, 12 May 2009 (UTC)[reply]
To me it sounds like a perfectly fair question. It does not require Energy, Force and Velocity to be the base units. All it requires is that their values turn out to be the ones described in the problem. Length, Time and Mass may still be taken to be the base units. I like that problem. Dauto (talk) 15:44, 12 May 2009 (UTC)[reply]
Yeah, it's a perfectly good question, just badly written. It should have said something like "If the units of force, energy and velocity in a new system of units are equal to 10N, 100J and 5 m/s, what are the units of length, mass and time equal to in terms of the appropriate SI units?". (That's assuming we have correctly interpreted the question, which is far from certain.) --Tango (talk) 18:01, 12 May 2009 (UTC)[reply]

molecules and electrons

how can i find out the no. of electrons in 6.022 * 1022 molecules of methane?? —Preceding unsigned comment added by 122.50.131.1 (talk) 15:01, 12 May 2009 (UTC)[reply]

First find the number of electrons in one molecule of methane and than multiply that number by 6.022 * 1022 :) Dauto (talk) 15:51, 12 May 2009 (UTC)[reply]
Then check that you really mean 6.022 * 1022, and not 6.022 * 1023. Algebraist 16:49, 12 May 2009 (UTC)[reply]
And consider whether you feel comfortable reporting the answer to four significant figures. Nimur (talk) 04:44, 13 May 2009 (UTC)[reply]
Are there methods which could increase or decrease the number of electrons in the specified material? Edison (talk) 05:17, 13 May 2009 (UTC)[reply]
Even methane plasma is still electrically neutral... Nimur (talk) 20:07, 13 May 2009 (UTC)[reply]

What's That Bug, episode 271,828,183

This thing is about 1.5 inches long, lives happily underwater (swims well, etc.), seems pretty happy out of the water, doesn't bite (me). I found it in a small stream in central North Carolina, USA. Anyone know what it is? Thanks. --Sean 15:20, 12 May 2009 (UTC)[reply]

File:BagginsBug1.jpg
What am I?

This file may be deleted at any time.
File:BagginsBug2.jpg
The underneath

This file may be deleted at any time.
Looks like a dragonfly nymph, but no idea what species. Mikenorton (talk) 15:58, 12 May 2009 (UTC)[reply]
Yeah, that's what I was about to say. Looie496 (talk) 15:59, 12 May 2009 (UTC)[reply]
Wow, no kidding; thanks so much. I just got schooled by a 5-year-old. :) --Sean 18:24, 12 May 2009 (UTC)[reply]
It looks like the nymph of some kind of darner. This photo (no species specified), looks pretty close. There are a bunch of other pictures of darner adults and nymphs by the same photographer here, but the nymphs of the different species look rather similar. Deor (talk) 20:40, 12 May 2009 (UTC)[reply]
What I have to say it really doesn't look like is something I would hold in my bare hand to take its photo. The abdomen reminds me of a tarantula hawk, supposedly one of the top-ranked insects for the pain induced by its sting. Presumably it isn't, since we haven't heard you screaming across the Internet. --Trovatore (talk) 00:15, 13 May 2009 (UTC)[reply]
I was going to post that Sean was uncommonly brave but didn't want to look like a milquetoast on, you know, an anonymous Internet forum. Thanks for beating me to it, milquetoast! Tempshill (talk) 02:58, 13 May 2009 (UTC)[reply]
Thanks, Deor. That looks just like it. The little booger lives in my pond now, so hopefully s/he'll stick around and eat mosquitoes and other villains. --Sean 01:37, 13 May 2009 (UTC)[reply]

Be sure to add it to Wildlife of North Carolina! We need more benthic macroinvertebrates. Nimur (talk) 20:09, 13 May 2009 (UTC)[reply]

electrons and molecules

how can i find out the number of electrons in one molecule of methane? —Preceding unsigned comment added by 122.50.131.1 (talk) 16:11, 12 May 2009 (UTC)[reply]

You could start by looking at our article on methane. 65.121.141.34 (talk) 16:21, 12 May 2009 (UTC)[reply]
You'll get close if you start by writing down the names of the atoms, then next to each atom write the number of electrons that atom normally carries. Tempshill (talk) 18:00, 12 May 2009 (UTC)[reply]
Remember to then account for the overall charge on the molecule. (Methane is usually neutral, but if you wanted to do a similar analysis for the sulfate ion, net charge becomes important.) -- 128.104.112.117 (talk) 23:21, 12 May 2009 (UTC)[reply]
How could the number of electrons be affected by ionization? Edison (talk) 05:15, 13 May 2009 (UTC)[reply]
By adding or removing electrons, you create a non-neutral molecule. Quite a trick to draw a simple Lewis or valence-electron structure for, e.g., "CH4+", but such is life when you're doing mass spectrometry. DMacks (talk) 08:18, 13 May 2009 (UTC)[reply]

Removing COSTAR from Hubble

I don't really understand exactly how COSTAR can no longer be needed. So to begin with, I understand that Hubble had an aberration that required corrective optics in the form of COSTAR. Gradually old instruments have been replaced by ones that correct for the aberration themselves. How have the new instruments bypassed COSTAR? Is COSTAR something that can be turned on and off? Many thanks. RupertMillard (Talk) 18:02, 12 May 2009 (UTC)[reply]

To quote an official-looking page, "All the other instruments, installed since HST's initial deployment, were designed with their own corrective optics. When the FOC is replaced by the Advanced Camera for Surveys (ACS) during Servicing Mission 3B, COSTAR will no longer be needed." So appears that COSTAR has already been bypassed, and the last instrument needing correction is being replaced with one that does not need it.-RunningOnBrains 20:52, 12 May 2009 (UTC)[reply]
Yeah, but this is what I don't get — you stick something in a telescope to fix it. How can some instruments rely on it while others don't? RupertMillard (Talk) 21:09, 12 May 2009 (UTC)[reply]
They could have their own adaptive optics built in...just a guess though. -RunningOnBrains 02:00, 13 May 2009 (UTC)[reply]
I think the OP may be wondering whether the new instruments also peer through the same aperture as the old instruments. Tempshill (talk) 02:55, 13 May 2009 (UTC)[reply]
I think COSTAR's corrective optics can either be inserted into the optical path or stowed out of the way, but despite a good bit of searching yesterday, I was unable to find any sort of diagram showing this to be the case. -- Coneslayer (talk) 11:34, 13 May 2009 (UTC)[reply]
Yeah, that's exactly what I was driving at, Tempshill. I mean I had been trying to find out myself too. I guess I'll just have to settle for "that's just the way it is"! Thanks everyone for looking. RupertMillard (Talk) 20:18, 13 May 2009 (UTC)[reply]

egg yolks

Are there egg yolks from some bird species that are not normally yellow as per chicken eggs? Examples? 65.121.141.34 (talk) 20:17, 12 May 2009 (UTC)[reply]

This BBC article (http://www.open2.net/everwonderedfood/eggsecrets.html) seems to suggests that 'crimson' yolks are said to occur if a hen is fed coloured maggots. Not quite what you mean but mostly on-topic. ny156uk (talk) 21:02, 12 May 2009 (UTC)[reply]
Indeed. The colour of egg yolks is largely mediated by the diet of the mother, therefore you feed any bird a different diet and you get a different yolk colour. Apparently Philippine duck eggs are known for their orange colour, due to an excess of ketocarotenoids in their diet, and ducks from the lake areas in southern China (such as Honghu, Jianli, Xiantao, Tianmen and Hanchuan in the Hubei Province) have red yolks due to rhodoxanthin in the pondweeds they eat. Rockpocket 17:02, 13 May 2009 (UTC)[reply]

Common liquids

I realised recently (like...2003, but meh) that there are very few household liquids that I know of; nearly everything is a suspension or solution in water. I can only think of:

  • Water
  • Oil (including petrol/diesel)
  • Mercury

Are there many others that I'm simply ignorant of? 90.193.232.41 (talk) 20:50, 12 May 2009 (UTC)[reply]

That's an interesting observation! Another issue is that most things we use aren't "the pure chemical" but rather are dilute solutions or mixtures even if the material itself might be a liquid. DMacks (talk) 20:56, 12 May 2009 (UTC)[reply]
Mmhm. I noticed it when I realised milk was just water with suspended fat droplets; gravy was the same (water with something); so is pretty much everything I drink. What's blood? 90.193.232.41 (talk) 20:58, 12 May 2009 (UTC)[reply]
Blood is an aqueous solution with lots of cells and other things suspended in it. DMacks (talk) 20:59, 12 May 2009 (UTC)[reply]
At room temperature, there are e.g.:
  • Vinegar (Acetic acid)
  • Alcohol (Ethanol, Methanol, Propanol,...)
  • Carbon tetrachloride
--Stephan Schulz (talk) 21:01, 12 May 2009 (UTC)[reply]

(several ecs)One reason I can think of off the top of my head as to why this is true: if you look at the phase diagram for water, you will see that the green liquid area is the smallest region on the graph; most substances are liquid only for a certain, relatively small range of temperature and pressure. Also, you have forgotten some that are probably in your house right now:

Depending on what you mean by "household", there is also Hydrogen peroxide and Sulfuric acid. There are also many substances in your daily life which are kept as a liquid under pressure (even though they would be a gas if you released them from their container), such as 1,1,1,2-Tetrafluoroethane (probably cooling your refrigerator) and propane. -RunningOnBrains 21:22, 12 May 2009 (UTC)[reply]

Careful...the alcohols (especially drinking) in the home are almost certainly water solutions of these liquid chemicals. Likewise for peroxide and acetic acid. Pure sulfuric acid isn't even available at all. Cooking oil is a good household liquid (and important distinction vs the hydrocarbon oils and fuels). And from it we get biodiesel and similar liquids that are also different from hydrocarbon liquid fuels. OTOH, you'll want to read the glass#Physics of glass article to see what glass probably doesn't belong on a list of liquids. DMacks (talk) 21:34, 12 May 2009 (UTC)[reply]
I know glass isn't a liquid. I don't have "rubbing alcohol" (whatever that is) in my house. Aren't acids basically just a solution in water? =\ I guess I misunderstood chemistry in school...
Though they could be prepared as pure substances, both the acids and alcohols will almost certainly be solutions in water for any likely household use. Dragons flight (talk) 22:14, 12 May 2009 (UTC)[reply]
I usually have denatured alcohol at home (for cleaning). It's 95 or even 99% alcohol. I would not call this a "solution of alcohol in water", but rather "alcohol with impurities", or possibly "a solution of water in acohol". --Stephan Schulz (talk) 02:14, 13 May 2009 (UTC)[reply]
Perfumes and deodorants. Jay (talk) 09:59, 13 May 2009 (UTC)[reply]

A lighter may include a butane or naptha liquid. Dragons flight (talk) 22:20, 12 May 2009 (UTC)[reply]

Paint thinner / turpentine are other liquids in some households. Similarly, paints before application are usually liquid. Some paints are oil based, some are water based, and some are neither. Dragons flight (talk) 22:28, 12 May 2009 (UTC)[reply]

Courtesy of a slight Krazy Glue incident this evening, I got a chance to remind myself that the active ingredient in nail-polish remover is acetone (percent unstated on bottle). DMacks (talk) 08:14, 13 May 2009 (UTC)[reply]

Thank you very much for all the help, guys - this was exactly what I was after. ^_^ 90.193.232.41 (talk) 10:56, 13 May 2009 (UTC)[reply]

Most of the responses miss the OP's original point, which is a search for pure liquids one is likely to find around the house, not mixture or water-based solutions, but a substance that is a pure (or reasonably pure) liquid, and not a mixture of components. Almost everything listed above are clearly mixtures of multiple components. Nail polish remove is a water/acetone (or sometimes water/ethyl acetate) mixture. Paint thinner and terpentine are mixtures, as are most perfumes and cleaners. Even, strictly speaking, are cooking oils, which are not a single pure substance, but mixtures of a variety of triglycerides, composed of numerous different kinds of fatty acids. Water and mercury may sometimes be found around the house in pure forms, but beyond those two, you would likely be hard pressed to find any other liquids in your house which were not solutions or mixtures of some kind. --Jayron32.talk.contribs 15:20, 13 May 2009 (UTC)[reply]
Turpentine should be pure and 100% acetone nail polish remover is also available. Rubbing alcohol can be obtained for household use at 99% pure as well. The OP mentioned oil/petrol/diesel which are the same kind of mixtures as cooking oil and naptha so they should be acceptable also. Mineral oil is another such mixture that you may in your home. Taken to extremes, unless you stock fresh (and recently boiled) distilled water in your house that isn't pure either. If you still have carbon tet around your house, take it to your local hazardous waste handler. Rmhermen (talk) 17:45, 13 May 2009 (UTC)[reply]
For the record, cooking oil is NOTHING like petroleum oil. Petroleum is simple hydrocarbons, cooking oil is composed of substances like Triglycerides. But you are right to point out (as I had forgotten) that mineral oil is yet another different class.-RunningOnBrains 17:58, 13 May 2009 (UTC)[reply]
My reading of the OP was that he was looking for non-water-based liquids, rather than pure liquids. Dragons flight (talk) 17:50, 13 May 2009 (UTC)[reply]

I'm happy with mixtures, I just didn't want something which is basically a liquid purely because it's in some other liquid (something dissolved in water, for example). Alcohol is fine, even if not pure, as if you purified it it would be liquid. As would the components of petrol, if you separated them...I have assumed cooking oils would, too. 90.193.232.41 (talk) 23:20, 13 May 2009 (UTC)[reply]

magnetic rotation

Can one magnet using the repelling force of another magnet to create circular momentum of one of the magnet —Preceding unsigned comment added by 121.216.203.26 (talk) 21:30, 12 May 2009 (UTC)[reply]

It doesn't matter if it is repelling or attracting. Making one magnet move by use of another magnet is very common. The most visible example that comes to mind is dropping a magnetic cylinder in a liquid and placing it on a spinning magnet. The cylinder will spin also - mixing up the liquid. -- kainaw 22:35, 12 May 2009 (UTC)[reply]
The most common example of magnets attracting each other occurs in electric motors. —Preceding unsigned comment added by 98.21.108.29 (talk) 22:47, 12 May 2009 (UTC)[reply]
If our OP is asking whether there is some arrangement by which one magnet will keep another spinning against air resistance or friction - then the answer is a definitive "NO!" because that would be a perpetual motion machine. In the absence of air resistance and friction, a magnet can spin forever without the need to use another magnet to give it a continual 'push'. However, you could probably figure out some complex shape that would make one magnet start the other one spinning when you release them. But there is no such thing as a free lunch - the laws of thermodynamics guarantee that. SteveBaker (talk) 01:16, 13 May 2009 (UTC)[reply]

I need help understanding this task. I know that the charges on the periodic table go +1, +2, +3, +/-4, -3, -2, -1 (0), where 0 and +/-4 do not ionize (I think). But, what's the deal with the +e-? Honestly, I don't really know why some elements lose and gain electrons and some need that e-. Thank you. Sorry if this is a bit laborious. Thx! —Mr. E. Sánchez (that's me!)What I Do / What I Say 22:53, 12 May 2009 (UTC)[reply]

To a first approximation, atoms want their electron shells to look like noble gases (the octet rule), and they don't care which one. So they take the one that's easiest to get to. That is, the one where they have to gain/lose the least number of electrons, because each electron you gain/lose increases the overall charge, making it that much more energetically costly to gain/lose another. That's why the atoms on the left side of the periodic table tend to lose electrons, and the ones on the right gain electrons. It's the shortest, most energy-efficient path to be noble-gas like. (The whole analysis gets a little complicated in the transition metals, where there are electrons in the d-Orbitals which "don't count", but the general principle of being "noble gas like" still roughly holds.) -- 128.104.112.117 (talk) 23:17, 12 May 2009 (UTC)[reply]

Keeping strong alcohol in plastic bottles

I have a bottle of homemade Croatian moonshine slivovica, which I remember having a nice, slightly fruity smell and an okay aftertaste when it was new (once my throat stopped burning :) It still wasn't the most obvious tipple though, so about 3/4 of it ended up forgotten in a cupboard for the past year. I've just got it out and tried some, but now it has a much sharper smell and taste than I remembered. The bottle is plastic and was meant to hold mineral water, so I wondered if that was the cause. Could the spirit (which I guess is about 50% alcohol though I really have no idea) have leached any chemicals out of the plastic? 81.132.219.125 (talk) 23:38, 12 May 2009 (UTC)[reply]

Ethanol (alcohol) turns into ethanoic acid (vinegar) over time. Could it have been that? 90.193.232.41 (talk) 23:41, 12 May 2009 (UTC)[reply]
'Course, no one actually says ethanoic acid. It's acetic acid. Don't be pulled in by the useless silly IUPAC names. They'll never catch on, and this is a good thing. --Trovatore (talk) 07:44, 14 May 2009 (UTC)[reply]
It could have done I suppose, but wouldn't there still be a fruit smell? And wouldn't commercially produced spirits turn to vinegar in the same way, once the bottles had been opened? Thanks for answering btw :) 81.132.219.125 (talk) 23:48, 12 May 2009 (UTC)[reply]
Actually, now my glassful has been sitting out for a while, it does still have a fruit smell. The aftertaste is completely different though; it tastes like the smell of burnt plastic. If that makes any sense? 81.132.219.125 (talk) 23:51, 12 May 2009 (UTC)[reply]
I think there are a lot of possibilities, but I advise not drinking it -- alcohol is very powerful at dissolving stuff, and could conceivably be dissolving something from the bottle that you wouldn't want to be consuming. Looie496 (talk) 00:14, 13 May 2009 (UTC)[reply]
Use a glass bottle next time. Dauto (talk) 00:33, 13 May 2009 (UTC)[reply]

I know nothing about alcohol personally, but if our chemistry lessons were accurate in the real world, then yes - commercial spirits will also turn sour over time. 90.193.232.41 (talk) 10:53, 13 May 2009 (UTC)[reply]

Even water is known to leach residual chemicals from platic bottles. If you've ever left a water bottle in a hot car for a while, the water will take on a distinctive plastic taste. The presence of these chemicals are specific and quantifiable. See this study reported by the CBC. You can find other such studies around the web if you search, and it won't take long. I would imagine that alcohol, which is likely to be better at dissolving organic substances like plastics, would be expected to be better than even pure water at this, and thus it would easily be expected that alcohol stored in plastic over a long time could develop funny tastes. It may not be all plastics, but some will certainly impart unwanted flavors into your spirits. --Jayron32.talk.contribs 14:09, 13 May 2009 (UTC)[reply]
Cool, thanks a lot for the info. I wouldn't have drunk any more than the few sips I did, not when it now tastes so bad, but it's nice to know there's some sense in my uneducated guess :) 81.132.219.125 (talk) 22:38, 13 May 2009 (UTC)[reply]

May 13

Blood in near vacuum

The article blood says that blood in a vacuum looses its oxygen and turns blue. How long would this take? Would it happen before or after drying, as a matter of fact, would blood dry at all? Thank you. 190.17.201.142 (talk) 01:54, 13 May 2009 (UTC)[reply]

The lack of a source for that statement in the article makes tracking down related information. The Vacuum article has some useful information and lots of references. I also found this article that has some more in-depth information along the same lines. One of the key points is that the body contains the blood, so it does not boil and you do not freeze (at least not right away). With no air in the lungs, the blood becomes oxygen depleted fairly quickly, causing loss of consciousness in about 15 seconds, but you still may have a couple of minutes where recovery is still possible. -- Tcncv (talk) 02:41, 13 May 2009 (UTC)[reply]
I think the pertinent article is probably oxygen dissociation curve. In a vacuum, the air pressure is 0, making the partial pressure of oxygen also 0, so when the vacuum occurs, you drop to the left end of the curve, oxygen dissociates from hemoglobin, dissolved oxygen in the blood proper will also be lost. All of this would occur before any drying would. Blood would turn "blue" only in the sense that venous blood is said to be blue: it's the color of hemoglobin without an associated oxygen molecule. - Nunh-huh 02:50, 13 May 2009 (UTC)[reply]
Our vein and cyanosis articles explain that oxygenated blood is red, and deoxygenated blood is darker-red. The blue color comes from Rayleigh scattering by tissues. Notice that the sentence from the blood article that you referenced has a "citation needed" after it. Ginogrz (talk) 04:06, 13 May 2009 (UTC)[reply]
"Citation needed" is there because I just added it. -- Tcncv (talk) 04:55, 13 May 2009 (UTC)[reply]
Really interesting. So what would happen if I shed blood on a surface in an atmosphere at, say, 10 percent pressure that of the earth surface, for example in protected conditions at 15km altitude ? 190.17.201.142 (talk) 05:23, 13 May 2009 (UTC)[reply]

Runway threshold markings

This is not really a science question, but I figure the aviation folks are more likely to see this here. While touring the taking a tour via Google Earth, I happened to notice differences in the threshold markings on runways at various major airports. Most have twelve bars (six on either side of the centerline, while other very similar runways have eight (four on either side). For example, of Atlanta's four major runways, only 8L/26R has the twelve bar markings. However, this diagram shows that the runway 9L/27R is the same width and longer, but it only has eight bars. Another intersting case is Seattle-Tacoma International Airport here, where both runways have eight bars on the south end, and twelve on the north. Can anyone clarify this? -- Tcncv (talk) 01:56, 13 May 2009 (UTC)[reply]

There are standards - but there are SEVERAL standards - and there doesn't always seem to be a lot of logic as to which standard each airport uses. Runway#Runway_markings explains this to some degree. SteveBaker (talk) 02:42, 13 May 2009 (UTC)[reply]
Thanks. I saw that section, but it does not go into enough detail. I am convinced there is a standard out there somewhere that precisely states when to use eight stripes and when to use twelve. Aviation is two heavily regulated for airports such as Atlanta and Seattle to have mixed markings without a well defined reason. -- Tcncv (talk) 02:56, 13 May 2009 (UTC)[reply]
Apparently there are two configurations for runway threshold markings mentioned by the FAA (See section 2F here). They consist of either 8 parallel bars, or various numbers of parallel bars to indicate runway width (e.g. 8 = 100 ft, 12 = 150 ft). Unfortunately, my quick read didn't reveal why there are two configurations, or when each can be used. And as for why the two systems are mixed at SeaTac, I have no clue. -- Flyguy649 talk 05:04, 13 May 2009 (UTC)[reply]
A bit more. According to this, the 8-bars-no-matter-what configuration was to become invalid as of Jan 1, 2008. So it's possible that the google images are from a time when the airports hadn't yet finished changing over the markings. Just a thought. -- Flyguy649 talk 05:09, 13 May 2009 (UTC)[reply]
Thanks for the excellent info. The transition from configuration A to configuration B would certainly explain things. -- Tcncv (talk) 05:47, 13 May 2009 (UTC)[reply]
As further evidence, looking at the third runway construction in Seattle, it is apparent that the image of the south end of the airport is older that the image at the north end. And in Atlanta, the twelve stripe thresholds appear to be newer pain jobs than the eight stripe thresholds, judging from the tire marks. -- Tcncv (talk) 06:14, 13 May 2009 (UTC)[reply]
It may be that there is finally some effort to reconcile the multiple standards into a single one - but repainting those stripes is not a trivial matter. They look small from the air - but each one of those 'piano keys' is as big as a medium-sized parking lot - burning off and repainting that much paint isn't easy. Closing the runway in order to do the work is also difficult. Other aspects of the differing standards are even tougher to fix - lighting configurations, for example, can involve lights positioned out off the end of the runway on land that the airport authority may not even own...and a typical airport has many hundreds of lights. Some of the complications arise from updated requirements that 'grandfather in' the old standards for these kinds of reasons - others come about from military airfields being repurposed or shared for civilian needs - resulting in conflicting military versus civilian/FAA requirements, etc. It really is something of a mess. SteveBaker (talk) 14:00, 13 May 2009 (UTC)[reply]

Potential energy

We're given three charges, Q1, Q2, and Q3, which are arranged in an equilateral triangle and we're asked to find the total potential energy. Now normally I would have said that U = U1 + U2 + U3 = (k/d)(Q1Q2 + Q1Q3 + Q2Q1 + Q2Q3 + Q3Q1 + Q3Q2). But the answer says that U = (k/d)(Q1Q2 + Q1Q3 + Q2Q3). Why is this? —Preceding unsigned comment added by 65.92.6.148 (talk) 04:00, 13 May 2009 (UTC)[reply]

It appears that you are double-counting some energy. (Note that you're off by a factor of two). This is because the energy of two particles Q1 and Q2 is only counted once; if you also calculate it for Q2 and Q1, you are considering the same system twice. Nimur (talk) 04:34, 13 May 2009 (UTC)[reply]
(ec - partial duplication of prior answer) I'm no expert, but one way to check for consistency between the three change potential energy equation and the two charge potential energy equation is to set one of the charges, say Q3 to zero. When compared with the equation in Potential energy#Electrostatic potential energy, it appears that the textbook answer is correct. In short, don't double count Q1Q2 along with Q2Q1. That effectively is already built into the constant k. -- Tcncv (talk) 04:43, 13 May 2009 (UTC)[reply]
Put simply, the standard potential formula already counts the potential for both of the charges involved; you do not need to calculate and and and then add those three. (Those three quantities are not nonsensical, but they apply only if you consider establishing two of the charges and then bringing in the third. They can't all three be that third charge!) --Tardis (talk) 04:51, 13 May 2009 (UTC)[reply]
Why is that so? Is there some sort of derevation for this? Because the formula kq1q2/r comes from assuming one charge stays put, so why would it also account for the work that's done on it as well? —Preceding unsigned comment added by 65.92.6.148 (talk) 00:35, 14 May 2009 (UTC)[reply]
I managed to derive that for a constant force (such as when we assume gravity to be constant), the equation for U doesn't change whether we assume one of the bodies (eg the earth) says at rest or not. I just said that ΔU=-W_T=-(W_1,2 + W_2,1) = -(F•s_1 + F•s_2)=FΔh_1 - FΔh_2=F(Δh_1-Δh_2)=mgΔd, where W_1,2 is the work done on body 1 by body 2, and • is the dot product. Can someone help me with the derivation for a non-constant force? —Preceding unsigned comment added by 65.92.6.148 (talk) 04:00, 13 May 2009 (UTC)[reply]
It's really just a matter of definition. We could go around calculating the potential energies of each charge separately, but the contribution to A's potential from B is always identical to B's from A, so we might as well call their sum "the potential of A and B" and be done with it. Clearly we cannot then also consider "the potential of B and A" because we've already counted both those interactions. As for deriving it, it should be sufficient to apply Newton's Third Law (and its corollary that the forces are radial) and note that (where we define the scalar F as positive when repulsive). --Tardis (talk) 18:03, 14 May 2009 (UTC)[reply]

To answer this question, we need to look into the fundamentals... what is potential energy of a system ? It is the energy required to assemble the system. Or the energy you would get if you blew up the system. This is basically the "potential" energy that the system has. So first coming for a one charge system. What is its potential energy ? Assuming there is nothing else in the surroundings, it is zero. You didn't have to do any work to put it there. Note that we don't take into account the energy to create a charge... charges are simply given to us and we just put them around. This is wise policy, as the energy of a point charge, is in fact, infinite. So for a system of two charges, we first bring in the first charge. This wouldn't take any work. Now, for the second charge. We bring it in from far away. How far away ? You could say from your house, or from a known landmark like the Eiffel tower or the sun, but the standard practice is to bring it in from infinity. But it is repelled (or attracted) by the first charge.So we integrate to get the work done, which is how we get your famous formula. Now for say three charges, we put the first charge in. Again no work done here. Next we put the second charge in. You know the formula. Now for the third charge, it is repelled (or attracted) by both the first and the second charges. So we plug in the formula for both of them, and add them all up, which is how you get your textbook formula. So once you understand why you get a certain formula, it is easy to build skyscrapers on you strong basement... Rkr1991 (talk) 08:00, 14 May 2009 (UTC)[reply]

Makes sense. But out of curiosity, how would you re-derive the equation for potential energy while taking into account both charges moving (I guess it's more of a math question than anything....). —Preceding unsigned comment added by 65.92.6.148 (talk) 08:40, 14 May 2009 (UTC)[reply]

Now that's interesting... The whole time we were talking about Electrostatics, where we assume everything to be in rest or in very slow motion. Now the question you are asking belongs to the domain of Electrodynamics. Now i need to talk a bit more in the basics, a point which i evidently forgot to mention. I had told that you integrate the differential work from infinity to the required position. The question is, along what path do i integrate ? That is, while getting from infinity to the point, what curve does to charge follow ? The answer is it doesn't mater. You can integrate along any curve want, it doesn't change the answer. This is because the electrostatic force, given by coulomb's law, is a term we call conservative [[7]]. Its line integral is independent of path, and we get the same answer always. Now in the realm of electrodynamics, coulomb's law doesn't hold good. Further, in addition to electric forces, there are also magnetic forces. Both of these are now non conservative. So, our answer would now depend on the path we choose. So, if we get different energies if the charges are all blown up in different directions, there isn't really much meaning in the term potential energy anymore... So in electrodynamics, there is no such term Potential energy. However, there exists a four vector potential term... but we don't really care about that do we ? Also, don't forget to sign you posts, by placing four '~' marks at the end... Rkr1991 (talk) 15:09, 14 May 2009 (UTC)[reply]

MICROPROCESSOR

Deleted. Poster has posted the same homework question on RD/C repeatedly. Tempshill (talk) 15:51, 13 May 2009 (UTC)[reply]

Hypnotic state of mind while watching t.v.

(kids' eyes glass-over, people walk by unnoticed, because of the fast 'flickering' of the tv)

I've heard this several times, but searching for it online, I can't find anything on it.

67.236.121.225 (talk) 06:32, 13 May 2009 (UTC)[reply]

Photosensitive_epilepsy#Symptoms has a subsection about Television, traditionally the most common source of seizures.Cuddlyable3 (talk) 07:49, 13 May 2009 (UTC)[reply]
Immersion (virtual reality)?--Lenticel (talk) 08:03, 13 May 2009 (UTC)[reply]
Hyperfocus is marked as needing work, but the term may help with googling studies. 71.236.24.129 (talk) 08:13, 13 May 2009 (UTC)[reply]
You can get the same effect reading a book, writing or doing other tasks. Any time you "zone out," you're so focused on the task at hand that other stimuli are ignored. — The Hand That Feeds You:Bite 13:21, 13 May 2009 (UTC)[reply]
Also see tetris effect, highway hypnosis and automaticity. Lanfear's Bane | t 13:54, 13 May 2009 (UTC)[reply]

Sound Energy Converter

Is there and if there isn't, is it possible to create a device converting sound into kinetic, electrical, or electromagnetic energy?The Successor of Physics 12:56, 13 May 2009 (UTC)[reply]

See Microphone. -- Coneslayer (talk) 13:12, 13 May 2009 (UTC)[reply]
While we're at it, see geophone and hydrophone and piezoelectric crystal as well. Nimur (talk) 13:21, 13 May 2009 (UTC)[reply]
In the 1880's Thomas Edison invented a device which converted sound waves into vibration, and then into rotary motion, and it could power a drill. He noted that many a mother-in-law's voice had been thought to be capable of drilling holes. Edison (talk) 17:23, 13 May 2009 (UTC)[reply]
... really? That deserves a [citation needed] tag. Nimur (talk) 19:11, 13 May 2009 (UTC)[reply]
They had such a machine on exhibit at the reconstructed Edison Menlo Park lab at Greenfield Village when I last toured the place. I found a reference which (snippet view) refers to the rotary motion being used to run a sewing machine, voice powered. "STITCH! STITCH! KEEP STITCHING DAMMIT!" Treadle powered and later electric powered machines were More conducive to domestic tranquility. "Menlo Park Reminiscences," by Francis Jehl, (writen 1937, Dover reprint 1990) (who was present at Edison's lab in the 1870'2-80's) tells on pages 180-181 of the "phonomotor." On page 180 is an illustration of it from the Scientific American (apparently July 27, 1878). It was the Sci Am reporter who said "Mr. Edison says he will have no difficulty in making the machine bore a hole through a board, but we consider such an application of the machine of very little utility, as we are familiar with voices that can accomplish that feat without the mechanical appliance." The machine used a diaphragm connected via a rubber tube to a spring, which carried a pawl that acted on a ratchet on the flywheel. Certain voice sounds could cause the flywheel to rotate "with considerable velocity." See also [8], [9], [10], [11], [12], and [13]. The notability guideline says "If a topic has received significant coverage in reliable secondary sources that are independent of the subject, it is presumed to satisfy the inclusion criteria for a stand-alone article." Does it appear that this little "philosophical toy" or demonstration gadget qualifies for its own stand-alone article, since it caught the fancy of scientific journals and science writers in various countries? Edison (talk) 21:13, 13 May 2009 (UTC)[reply]
Sounds like a great idea for an article...and once you get it into a halfway reasonable shape - you should definitely send a quote from it to the "Did You Know?" section of the Wikipedia front page. SteveBaker (talk) 22:41, 13 May 2009 (UTC)[reply]
Certainly a microphone fits the bill. It produces alternating current electricity from sound. SteveBaker (talk) 22:41, 13 May 2009 (UTC)[reply]
Thanks, guys! I forgot even about simple stuff like microphones! I must have gone mad!The Successor of Physics 13:03, 14 May 2009 (UTC)[reply]

megawatt to kg, dm and minute

can anyone tell me the answer to these four questions: (i dont need hints)

1. convert a power of one megawatt on a system whose fundamental units are 10 kg, 1 decimeter and 1 minute. 2. find the value of 60 joule/min on a systemwhich has 100g.cm and 1 min as fundamental units. 3. write the dimensions of a/b in the relation v is velocity and t is time. 4. write the dimensions of a * b in the relation E= b - x2/at, where E is energy, x is distance and t is time. —Preceding unsigned comment added by 122.50.139.37 (talk) 13:17, 13 May 2009 (UTC)[reply]

If we do your homework for you (instead of helping you do it yourself), will we get your course credit? -- Coneslayer (talk) 13:23, 13 May 2009 (UTC)[reply]


i have already solved these questions. i just need to verify my answers (there's no one else left but you people, cos my friends are all out of town and i'm on vacation. i need to check my answers)

If you show us your work, we'll be happy to confirm the answers. This is a core tenet of our do your own homework policy. — Lomn 13:45, 13 May 2009 (UTC)[reply]
I think in that case, what might work better, would e for you to tell us the answers that you got, and then we could see if they are correct? 65.121.141.34 (talk) 13:46, 13 May 2009 (UTC)[reply]
(EC) You could send us your answers and ask us to check them for you! But we really do have strict rules about not answering homework questions - it's not just people being mean to you. SteveBaker (talk) 13:48, 13 May 2009 (UTC)[reply]
Please do your own homework.
Welcome to the Wikipedia Reference Desk. Your question appears to be a homework question. I apologize if this is a misinterpretation, but it is our aim here not to do people's homework for them, but to merely aid them in doing it themselves. Letting someone else do your homework does not help you learn nearly as much as doing it yourself. Please attempt to solve the problem or answer the question yourself first. If you need help with a specific part of your homework, feel free to tell us where you are stuck and ask for help. If you need help grasping the concept of a problem, by all means let us know.

So I just measured my penis size, why am I not happy with it?

I am almost 8" inches long erect, definitely well over 7", but only 6" exactly in girth. Why am I not satisfied with my penis size?--Pipelinefine (talk) 14:28, 13 May 2009 (UTC)[reply]

The reference desk will not address medical or psychological issues. If you have a medical concern, see a physician. If you have a psychological concern, see a psychiatrist. Nimur (talk) 14:32, 13 May 2009 (UTC)[reply]

the reason probably has to do with your self-esteem (google self steem and impotence to see extreme examples), which is probably made worse by the stigma and discrimination you face for being a troll. 79.122.61.98 (talk) 16:58, 13 May 2009 (UTC)[reply]

Over 7" is above average, I believe, so your lack of satisfaction is presumably about you, not your penis, and since we only have information about your penis we cannot help you. If it is causing you significant distress, then see a psychiatrist (your GP/family doctor can refer you to one), otherwise just try and get over it! --Tango (talk) 17:15, 13 May 2009 (UTC)[reply]
Take it easy, no body else is measuring but you. Length graph. Girth graph. Mac Davis (talk) 03:42, 14 May 2009 (UTC)[reply]
Is this the time to say "It isn't the size of the ship, its the motion of the ocean"? A penis is to sex as a hockey puck is to hockey; it may be a needed component, but your enjoyment and understanding of the game will become far greater when you take a more holistic approach to it. Just as real hockey fans learn to appreciate the off-puck action, you may find that the enjoyment of all games improves when the focus is off of a specific "tool" in the game, but rather on the entirety of the game itself. --Jayron32.talk.contribs 04:07, 14 May 2009 (UTC)[reply]
Well said Jayron32, I hope your ocean is long in motion. Richard Avery (talk) 13:43, 14 May 2009 (UTC)[reply]
If you want information on normal range of variation in sizes, we do have it here in Wikipedia. See the detailed article Penis size. We may not answer medical questions at the desk, but we do have information in the encyclopedia from which you can answer them yourself. 8 x 6 is in the highest few percent, as you can see from the graphs there.DGG (talk) 15:27, 14 May 2009 (UTC)[reply]

Why does sticking your thumb on a water hose make the water come out faster?

I'm not sure if this is the right place to ask this question, but our article on the List of common misconceptions states (depending on which edit you happen to look at):

It is not true that a nozzle (or a person's thumb) on the end of a garden hose makes the water squirt farther because the same amount of water gets forced through a smaller opening. The rate of flow of water through the hose is not a set constant; in fact, putting one's thumb over the end of the hose reduces the rate of flow. What is constant is the water pressure at the source. When water is flowing, the pressure decreases the farther from the source one gets due to friction between the water and the pipes it's flowing through. The faster the water moves through the pipe, the greater is the friction that cuts down pressure at the output end. A thumb over the end of the hose decreases the flow rate, causing the friction from the source to decrease, causing the remaining water to have more speed.[1]

A discussion is here[14].

Would some editors who know more about friction and pressure help settle this issue? A Quest For Knowledge (talk) 14:56, 13 May 2009 (UTC)[reply]

Take a look at the above section, WP:RD/S#Submarine hull breach. SteveBaker's math is pretty applicable to the hose problem. Note the important distinction between mass flow rate and flow velocity. Reducing the aperture size (with your thumb) will increase the flow velocity but decrease the mass flow rate. Nimur (talk) 15:00, 13 May 2009 (UTC)[reply]

To put the above in layman's terms, covering the end of the hole with your thumb both reduces the amount of water coming out and increases the speed at which it comes out. These are not necessarily contradictory ideas. --Jayron32.talk.contribs 15:09, 13 May 2009 (UTC)[reply]
Jayron32, thank you for putting it in layman's terms but is the increased speed caused by pressure or friction? This seems to be the heart of the issue. A Quest For Knowledge (talk) 15:21, 13 May 2009 (UTC)[reply]
Both change and both are variables that change the velocity. David D. (Talk) 15:55, 13 May 2009 (UTC)[reply]
Having said that if you turn the tap off nothing happens so pressure is the primary issue. David D. (Talk) 15:59, 13 May 2009 (UTC)[reply]
I don't believe that pressure increases. You have the same amount of pressure pushing from the tap, which is forcing a similar amount of volume per second to flow through the hose (the rate will decrease somewhat due to increased flow resistance, analogous to a resistor in an electric circuit, but that is subtracting from the spraying effect, not adding). Because you have the same (or similar) amount of volume going through a smaller area, the water must speed up in a linear, meters/second way (if you think about it, volume flow rate, q, is equal to A*V, decreasing A while keeping a similar q will invariably lead to a higher V). The pressure drop should equalise (after a few seconds of flowing through your thumb) to approximately the same, it's going from whatever it is at the end of the pipe to atmospheric. To get a more mathematical idea about this phenomena, you might find Bernoulli's principle an interesting article. TastyCakes (talk) 16:14, 13 May 2009 (UTC)[reply]
The pressure at the nozzle does change. I agree the pressure at the valve does not. Also bear in mind there is not a constant flow rate in this example. David D. (Talk) 16:33, 13 May 2009 (UTC)[reply]
q is not approximately constant. In fact, for a fixed pressure difference the volumetric flow rate is strongly dependent on the size of the outlet. (Consider a basin full of water, the rate of water flowing out clearly depends on the number and size of the holes provided.) Also, water in a real hose is subject to viscosity, which affects the pressure reaching your thumb and is a factor not considered under Bernoulli. Dragons flight (talk) 16:35, 13 May 2009 (UTC)[reply]
For the purpose of Bernoulli, is water considered inviscid? I have only seen Bernoulli used in reference to gasses such as air, which is about 50-100x less viscous then water. 65.121.141.34 (talk) 16:23, 13 May 2009 (UTC)[reply]
Ya, Bernoulli's principle is true for all incompressible fluids as I understand it. In fact I think it should be simpler with an incompressible fluid than you've probably seen with air. TastyCakes (talk) 16:26, 13 May 2009 (UTC)[reply]
In short, I think saying it's a misconception about the water flow is incorrect. While it is true that the volumetric flow rate is not constant because of changes in friction etc is completely accurate, the concept is true (in my opinion) in that the area of the end of the hose is decreased much more than the volumetric flow rate is decreased, and as such the q = V*A equation dictates an increase in velocity, even though the effect is somewhat lessened by q's own reduction. The pressure will increase at the end of the pipe because the flow rate has reduced the friction loss along the pipe. But I think in your average garden hose the difference will be quite small, since the change in volumetric flow rate would also probably be quite small. It is not the main cause of the effect, in my opinion; the area reduction is. TastyCakes (talk) 16:36, 13 May 2009 (UTC)[reply]
In the laminar, viscous limit, the volumetric flow rate is proportional to A2 and so changes more rapidly than A (see: Hagen–Poiseuille equation). Obviously your thumb is not conducive to laminar flow, but I think your assumption that the change in q is small is still incorrect. Dragons flight (talk) 16:51, 13 May 2009 (UTC)[reply]
mm isn't it P that's proportional to A*r2 rather than proportional to the area squared? And isn't that only for circular cross sections, which wouldn't be true for the thumb over the hose analogy? TastyCakes (talk) 17:00, 13 May 2009 (UTC)[reply]
So HP would be correct for the most part but your average velocity leaving the pipe still follows the simpler formula:
Perhaps you are right that Q is greatly reduced, but it seems quite clear from the equation that in the hose example it is reduced much much less than A, else the water wouldn't come out of the pipe faster. TastyCakes (talk) 17:11, 13 May 2009 (UTC)[reply]
I don't think you're factoring in the resistance along the hose. As the flow rate decreases the resistance along the hose decreases too. The effect of this is an increase of pressure at the nozzle. David D. (Talk) 20:44, 13 May 2009 (UTC)[reply]


That problem cannot be correctly solved by either the Hagen-Poiseuille's equation or the Bernoulli's equation alone. The first one assumes an uninterrupted pipe of circular cross-section which is not true here because the thumb interrupts the flow at the end. The second one neglects friction all together and cannot explain the phenomenon. But a combination of both can solve the problem. Lets stablish three points of interest, , , and . will be a point somewhere in the water reservoir being kept at a constant pressure , will be a point at the end of the hose right before the nuzzle (or thumb) whose pressure is yet to be determined, and will be a point outside the nuzzle kept at atmospheric pressure . Lets define the pressure drops along the hose before and after (that is without the thumb and with the thumb).
, and
.
We can use Hagen-Poiseuille to compute the water flow before and after.
, and
.
To find we use Bernoulli
Where we used the fact that the flow through points and must be the same,
.
and are the cross-section areas at point in the hose and point at the thumb.
Now we place the expression we got for into the expression obtained from Bernoulli's equation.
Where we introduced the quantity . Solving the quadratic equation for we get
, and
.
If we then get and the flow does not change very much. but if we then get which gives a flow proportional to the area and a constant exit velocity. Dauto (talk) 21:03, 13 May 2009 (UTC)[reply]

why do women love chocolate

or is it a myth —Preceding unsigned comment added by 79.122.61.98 (talk) 16:43, 13 May 2009 (UTC)[reply]

Probabily the same reason why men love chocolate. Dauto (talk) 17:10, 13 May 2009 (UTC)[reply]
I'm talking about the stereotype. Either its a myth or women love chocolate more and differently than men. 79.122.61.98 (talk) 17:24, 13 May 2009 (UTC)[reply]
A few studies seem to suggest women do have a higher preference for chocolate than men do, but the reasons for this are not known (PMID 1799282, PMID 12954417). Rockpocket 17:34, 13 May 2009 (UTC)[reply]
Yeah, I prefer chocolate to men any day!--80.3.133.116 (talk) 17:38, 13 May 2009 (UTC)[reply]
Touché, (I've edited to clarify my meaning). Rockpocket 17:54, 13 May 2009 (UTC)[reply]
What are your views on men buying you chocolate? --Tango (talk) 17:59, 13 May 2009 (UTC)[reply]
Our survey says... I specifically linked to the account at confectionerynews to highlight that this survey was commissioned by a chocolate manufacturer. 80.41.120.247 (talk) 19:31, 13 May 2009 (UTC)[reply]
I'd always thought it was just that women tend made a bigger deal about going against whatever fad diet they were currently on and 'being naughty' by eating the oh-so-fattening chocolate that <whoever writes those womens' mags> says is bad. Guys seem to be much less concerned about their weights and figures. --Kurt Shaped Box (talk) 17:49, 13 May 2009 (UTC)[reply]
Well, yeah, but I think there's a little more to it. Since we're dealing in stereotypes, I would think of a guy chowing down on a massive steak or similar "manly" meal rather than a box of assorted chocolate. Matt Deres (talk) 18:11, 13 May 2009 (UTC)[reply]
I seem to recall women reacting more strongly to the serotonin-related effects than men, but maybe I'm misremembering this. I did get the impression that I have read numerous scientific discussions of why women do seem to have a much stronger psychological reaction to chocolate than most men do. --140.247.252.198 (talk) 18:57, 13 May 2009 (UTC)[reply]
Yeah, I remembered reading the same thing (not sure where) but was too afraid to post for fear of being flamed. Google scholar seems to remember it too. SpinningSpark 19:35, 13 May 2009 (UTC)[reply]
Can you link to a specific study showing evidence that chocolate per se affects serotonin (not "carbohydrates" or whatever, as lots of other foods contain this)? The "Chocolate: Food or Drug?" (J Am Diet Assoc.99:1249-1256.(1999)) study is pretty unscientific when it comes to the neurobiology part - acknowledging that PEA doesn't reach the brain (and higher quantities are in cheese) but despite that still speculating that because "7 MDMA abusers [displayed] an intense craving for chocolate [...] it is worth noting that MDMA is structurally related to PEA and may produce similar effects". Also I only see higher cravings for chocolate during pre-menstrual stage, along with a mention of cravings for carbohydrate being affected by serotonin. --Mark PEA (talk) 10:17, 14 May 2009 (UTC)[reply]

what is this "I did get the impression that I have read numerous" sentence?? This phrasing sounds schizophrenic to say the least...79.122.61.98 (talk)

What, you say?The Hand That Feeds You:Bite 21:29, 13 May 2009 (UTC)[reply]
When I write sentences like that it usually means I changed by mind about what I was going to say half way through writing it and didn't go back and change everything so it was consistent. --Tango (talk) 21:44, 13 May 2009 (UTC) [reply]

I fucking hate chocolate, and ice cream, and anything like that

A distinction should be made here between milk chocolate and dark chocolate. Dark chocolate has less sugar than milk chocolate, and perhaps is less preferred by women. In fact, chocolate alone is quite bitter. The Wikepedia article on chocolate says, "Aztecs made it into a beverage known as xocolatl, a Nahuatl word meaning 'bitter water' ". The Wikipedia article is well written, information dense, and interesting, especially if you are interested in making and manufacturing things. It includes a lead to "mouthfeel" which has some good definitions. - GlowWorm.

Perspective of an electron

I was thinking the other day what it would be like to be a person on an electron while it orbits the nucleus of an atom. I tried to calculate it out but it got confusing. What I tried to do was make the proportion of a height of a person (about 2 meters) over the diameter of Earth (12,756,200 meters) equal to X (the height of a person on an electron) over the diameter of an electron (about 5.636x10^-15 M). I got ~8.836x10^-22 meters. After I got here though I couldn't figure out where to go, in other words how I would be able to use this information to help me relate my size relative to Earth to what it would like on an electron. I thought of trying to compare the size of the Sun in Earth's sky to the nucleus of a hydrogen atom in an electrons "sky". If this makes any sense to anyone, I would appreciate some help! Thanks!

And if this is still kinda confusing, I'll try to simplify what I'm trying to do:

As we stand here on earth, we can look out the Sun, the Moon, other planets, and other stars and galaxies. If you notice, electrons, although they don't orbit like Earth does around the Sun, still "orbit" around the nucleus of an atom. I'm curious to see what the "sky" would look like if an electron was earth and we were people on it. How far away would the Nucleus (Sun) be? How large would it look in the "sky"?

Hopefully you can get my drift, and help me out. Thanks again! —Preceding unsigned comment added by 129.21.109.153 (talk) 21:45, 13 May 2009 (UTC)[reply]

I don't know where you got the diameter number for your electron - but it's wrong. Electrons are 'point particles' - they have no size. The nearest thing to a 'size' you could use would be some function of the probability cloud that describes it's position...but that would give you a really odd view of what you're trying to understand. SteveBaker (talk) 22:33, 13 May 2009 (UTC)[reply]
The OP is referrring to the classical electron radius. Although Steve is right; the electron has no actual physical size in the way that macroscopic objects do. Someguy1221 (talk) 22:40, 13 May 2009 (UTC)[reply]
Yes, but the classical radius of the electron isn't a very useful conceplt. Compton wavelength of the electron might be a better choice? Dauto (talk) 23:39, 13 May 2009 (UTC)[reply]
How about, what would be its radius if it were a Kerr–Newman black hole of the same mass, charge, and angular momentum? That would seem to be a limiting value closer than which you couldn't really model it as a point. --Trovatore (talk) 08:11, 14 May 2009 (UTC)[reply]

Oh ok I see what you are saying. I was actually thinking that too, but I got a number for it so I figured it must have a diameter. I suppose this means that this couldn't really happen then, huh? —Preceding unsigned comment added by 129.21.109.153 (talk) 00:33, 14 May 2009 (UTC)[reply]

Some of our models of the atom treat an electron as a discrete little "planet" orbiting the nucleus of the atom. This can be a useful model for explaining some simple aspects of atomic structure, but is fantastically far from reality. Depending on what you are attempting to study about an electron, you can model it as either a) a discrete particle b) a "cloud" surrounding the nucleus of the atom c) a standing wave d) a quantum mechanical wave function. Like any models, these have varying degrees of proximity to reality, as well as varying degrees of "ease of use" and "usefulness" in particular situations. If, however, you stop trying to force an electron to behave like something you have experience with, like a little ball, and instead just accept it for what it is, it makes it easier to understand the more accurate models of electron behavior. --Jayron32.talk.contribs 04:02, 14 May 2009 (UTC)[reply]
Yep. There are some things (especially at the quantum level) that you just can't come to understand by analogy with macroscopic things. You have to resort to these rather unsatisfying ways of coming to terms with one property by itself - but never really getting a 'feel' for what the entire object is like. SteveBaker (talk) 04:52, 14 May 2009 (UTC)[reply]
This is especially true because you wanted to know what things would "look like." Your visual perspective is based on electromagnetic waves of a certain wavelength, visible light. Those wavelengths are about 300 nanometers to 700 nanometers. If you tried to "see" anything smaller than those length scales, you would not be able to form an image. At the very best, you could try to do some kind of interferometry or amplitude detection (LIDAR ?) And every time one photon of light arrived, it would blow your "planet" (electron) into a totally different quantum state (with new position and momentum). I don't think you could "see" anything. Nimur (talk) 13:22, 14 May 2009 (UTC)[reply]
Also worth noting is that while applying classical physics understanding to a quantum mechanical problem produces problems like you described, the reverse is not true. All macroscopic particles obey quantum mechanical equations perfectly, its just that within the realm of real measurement devices, the "classical approximation" is functionally identical to the quantum mechanical reality. In other words, all objects have a corresponding wave function; even YOU do, its just that your wave function is not terribly useful in getting information about your expected behavior. --Jayron32.talk.contribs 18:25, 14 May 2009 (UTC)[reply]

Grumbling Tummy

Childish question, this may seem, but why does your tummy grumble and gurgle? What's it doing? Eating itself? (Well, digesting, anyway).--KageTora (영호 (影虎)) (talk) 22:28, 13 May 2009 (UTC)[reply]

See Borborygmus. (I'll admit, I just learnt that word, and isn't it fantastic?!) --Tango (talk) 23:03, 13 May 2009 (UTC)[reply]
Excellent! Interesting to note that it even has a plural 'borborygmi'. Thanks Tango!--KageTora (영호 (影虎)) (talk) 23:12, 13 May 2009 (UTC)[reply]

May 14

Zoological Society of London - Presidents and Secretaries

I've been working on Zoological Society of London and I have two questions about officers of that society:

  • (1) Zoological Society of London#Presidents - there are seven links there to pages about titles, rather than people. Effectively, those seven links need to be disambiguated to point to the right person. Can anyone here help with working out which earls, lords, marquesses, dukes and viscounts are which?
  • (2) Zoological Society of London#Secretaries - there are two entries missing (the officers of this society serve for three year terms). These two missing entries are currently marked by [...], but if anyone can find out who those two missing entries are, I'd be very grateful.

Thanks. This may be more of a history question - will post a link from over there as well. Carcharoth (talk) 00:04, 14 May 2009 (UTC)[reply]

Why don't you raise this content question here ? Cuddlyable3 (talk) 14:45, 14 May 2009 (UTC)[reply]
I was kind of hoping people here could help. I've sometimes asked questions here about obscure stuff for articles and got answers and sources that have been channelled back into the articles, thus improving them. Sometimes you are more likely to get an answer here than on the article talk page, though you are right in that that should also be tried. See also Wikipedia:WikiProject Reference Desk Article Collaboration and Template:WPRDAC. Carcharoth (talk) 21:17, 14 May 2009 (UTC)[reply]

Nature of electron

Is an electron a particle or is it an em wave ? —Preceding unsigned comment added by 79.75.124.111 (talk) 00:22, 14 May 2009 (UTC)[reply]

See Electron which states that "Electrons have quantum mechanical properties of both a particle and a wave". somehow i thinck u new that! -hydnjo (talk) 00:42, 14 May 2009 (UTC)[reply]
An electron is definitely not an em (electromagnetic) wave. The particle of the electromagnetic field is the photon, not the electron, despite the name. The electron is a different kind of wave. -- BenRG (talk) 01:22, 14 May 2009 (UTC)[reply]
The electron is neither a wave nor is it a particle. It is an electron. When we attempt to explain its behavior using only classical mechanics it becomes necessary to sometimes treat it like a discrete particle, and other times to treat it like a wave, but this is only because classical physics is inadequate for dealing with the realities of electron behavior. Classical mechanics lacks the ability to create an adequate model for fully explaining electron behavior, and so we end up with the unsatisfying paradox of wave-particle duality. Thankfully, quantum mechanics exists, and it takes care of these apparent paradoxes just fine. See also Copenhagen interpretation for some more background on the philosophy behind the quantum mechanics/classical mechanics tension. --Jayron32.talk.contribs 03:54, 14 May 2009 (UTC)[reply]

Future remote DNA sampling

SciFi warning - What could be a future way of remotely sampling someone's DNA for analysis, say, from a meter or so away? Could a laser be used? Thank you.190.17.201.142 (talk) 00:27, 14 May 2009 (UTC)[reply]

Normally you need to extract DNA out of cells, with steps continuing after that. See DNA extraction. Mac Davis (talk) 03:38, 14 May 2009 (UTC)[reply]
There is nothing in our current technology that could be imagined to be used one day for a kind of quick DNA scanning of a cell? 190.17.201.142 (talk) 03:52, 14 May 2009 (UTC)[reply]
You could imagine an extra-large Glonckian scanner. This is a device that superimposes a copy of itself into the subject and collects a sample, then uses quantum teleportation to move the virtual sample (but not the copied Glonckian scanner) back into the cloud chamber of the original scanner. Of course, Glonks are usually mean to scan much smaller molecules, but one might eventually be built with a really large sampling hydrator to capture an entire DNA molecule. Tempshill (talk) 04:08, 14 May 2009 (UTC)[reply]
I suppose one might be able to sample dead skin cells by blowing air-flows over the person in question. Once you have a sample (even a small one) then extracting the DNA and running some kind of classification scan on it could probably be automated and brought down to a couple of minutes. But the trouble with sampling at a distance like that would be lie in being 100% sure that the material you get a hold of did actually come from that person. Cross-contamination is a serious problem for DNA magnification techniques - just one DNA molecule from someone else would be plenty to screw up the results. I suspect that we'd use some other identification technique in the future. Between photographs, super-low-dosage X-rays and other remote sensing techniques - I'm sure you could easily gather enough biometric data to figure out who someone is. SteveBaker (talk) 04:47, 14 May 2009 (UTC)[reply]
you could fire a very tiny projectile, that knocked off a few cells, or jab with a long thin needle very quickly. Perhaps a kind of smart dust or nanobot could accomplish the mission. Graeme Bartlett (talk) 10:30, 14 May 2009 (UTC)[reply]
Think about it. Cuddlyable3 (talk) 14:38, 14 May 2009 (UTC)[reply]

average life of furniture?

Today I was discussing the concept of semi-durable goods with a friend, things like furniture, book shelves, plastic bins you might keep on your desk or for your clothes etc. I have also seen lots of yard sales + things on the side of the road, (old chairs, furniture etc) and It got me thinking about what the average life of most of these goods is. How long does the average ikea table last? Bookshelf ? Chair? etc. Seems to me there is a big range, from 6 mo's for the worst of it up to 100's of years for the best. Anyone have any thoughts on where to get this data? Or ever see it discussed? —Preceding unsigned comment added by 72.230.5.95 (talk) 01:04, 14 May 2009 (UTC)[reply]

Depends on what you mean by lasting. Do repaired items count as lasting? Because then the lifespan of a piece of furnature is indefinate, see the Ship of Theseus paradox for more... --Jayron32.talk.contribs 03:48, 14 May 2009 (UTC)[reply]
Well, things like tables don't ever wear out - they get progressively more marked and chipped and so forth - but the "life" of the object is almost infinite. We have a 200 year old hall table and it's 100% functional. I don't see why it wouldn't be functional in another 200 years. Certainly some classes of furniture can get broken - but that's a random event that's not in any way related to the age of the furniture. So I don't think you can put a lifespan on that - if it breaks, it's a random event that's as likely to happen on the day you buy it as on a day 200 years later - and if it just gets unacceptably "scruffy" then it's down to your tolerance for scruffy furniture - it'll still be perfectly functional when it's so beat up that you decide it's at the end of it's life and replace it. Obviously, upholstered furniture has a more certain life - but you can always re-upholster anything like that as many times as you need to. We had a pair of sofa's that saw hard duty in our family room (they were really seriously abused) and they were terminally scruffy by 15 years old. However, a few hours with a sewing machine and a staple gun - and they looked brand new again. Things like bookshelves literally can last forever. The bookcase in my study was the one that came "free" with a complete set of Encyclopedia Britannica's in 1960. The books are long gone (who needs them when you have Wikipedia?) - but the bookcase lives on.
There are antique pieces of furniture in daily use in some people's home that was made many centuries ago - it's still 100% functional. Just about the only furniture we've even been forced to throw out was a pair of crappy MDF computer desks that both failed due to the poor choice of construction techniques - but my victorian writing desk gets daily use - and will undoubtedly outlive me!
SteveBaker (talk) 04:41, 14 May 2009 (UTC)[reply]
If you are looking for record breakers look at furniture from old tombs (e.g. [15]). This is mostly a question of durable materials. laminated particle board or Medium density fiberboard contains glues and binders that will dissipate or change over time. So it's unlikely our descendants will find many "ancient" examples of that. Despite it's reputation plastic is only durable under ideal storage conditions. Otherwise the plasticizers will get lost over time and the piece will become brittle. Whether a piece of furniture survives depends a lot on what value their owner assigns to it. The Germans have a funny nickname for their bulky trash pick-up days "furniture trading day". (OR One of my ancestors is remembered by reportedly using a Rosewood cabinet as firewood. They had just gotten brand new plywood furniture.) 71.236.24.129 (talk) 08:58, 14 May 2009 (UTC)[reply]
When we lived in Germany (Moenchengladbach) in the late '70s it was also known as the "monthly schrank," a schrank [sp?] being a large item of furniture something like a Welsh dresser. The custom was that anyone looking for 2nd-hand furniture or other items such as stereos would tour the streets and take whatever they wanted before the municipal lorries came round to collect the remainder. There was no social stigma attached, and a lot of the goods put out were good quality and in good condition. 87.81.230.195 (talk) 23:34, 14 May 2009 (UTC)[reply]
Here are hypotheses about the lifetime of semi-durable household goods. They may or may not be possible to quantify by collecting data from sources such as media advertisements, private ads, repair services and municipal trash dumps.
  • An item can have more than one average lifetime
    • The time it stays fashionable
    • The time it is saleable, new or used
    • The time it is useable, including justifiable repairs
    • The time it is collectable.
  • The above lifetimes are different for a high-consumption relatively wealthy society or an economically challenged society. Cuddlyable3 (talk) 14:23, 14 May 2009 (UTC)[reply]
And all bets are off for your wooden furniture if you get termites. 65.121.141.34 (talk) 19:56, 14 May 2009 (UTC)[reply]
I would never touch second-hand furniture for fear that it might have bedbugs (Cimex lectularius). They seem to be spreading rapidly. They are a real horror. I won't even sit on a chair any more in a waiting room or other public place. I no longer go to movies or theaters. When bedbugs become well established in a home, they are almost impossible to get rid of, like cockroaches. At least cockroaches do not suck your blood. Wikipedia has an article on bedbugs. Also see bedbugger.com. - GlowWorm.
If you aren't against bugs in general check out this guy for a natural countermeasure: Scutigera coleoptrata. 71.236.24.129 (talk) 10:21, 15 May 2009 (UTC)[reply]

'Bubbles' in the Sky

You know when you look up into a clear blue sky (could be anything, really, but this is an example), and you see little tiny round 'bubbles', almost like bacteria, but you can't quite look directly at them because they are on the surface of your eye (I'm not mad, everyone I know sees them)? What are they called and, more importantly, what are they? Actually bubbles in the liquid on your eyes? Do they even have a name?--KageTora - (영호 (影虎)) (talk) 02:57, 14 May 2009 (UTC)[reply]

floaters. Dauto (talk) 03:01, 14 May 2009 (UTC)[reply]
Incredible! This is why I love Wikipedia! Thanks!--KageTora - (영호 (影虎)) (talk) 04:30, 14 May 2009 (UTC)[reply]

Name that disease

There was a tropical disease I heard about in Guam that humans would contract when they rubbed their backs against the trunks of palm trees for itch relief. Tiny microorganisms would be left on your body and you would feel very bad and cause your skin to itch more. To my knowledge it did not particularly afflict Guamanians, I just heard about it in the Navy hospital. Can not remember if this was fatal. Can anybody help? I thought it was Chagas disease but after reading the article I am not sure this is the case. Mac Davis (talk) 03:30, 14 May 2009 (UTC)[reply]

Chiggers? --Jayron32.talk.contribs 03:44, 14 May 2009 (UTC)[reply]
Our article I link above mentions several diseases carried by Chigger larvae, some of which are endemic to the Pacific islands. --Jayron32.talk.contribs 03:46, 14 May 2009 (UTC)[reply]

Faffing about in space

I've been watching NASA TV to see the crew of STS-125 capture the Hubble Space Telescope and start servicing it. One thing puzzled me about the entire process: Why does it take so long? After Atlantis crept up on Hubble over several hours, it stopped for some 40 minutes about 50 feet away. The robotic arm then took an age to creep up to the grapple point on the Hubble. After the Hubble was captured by the arm, it was just held there for at least another 30 minutes (an orbital sunrise and an orbital "midday" came and went), until the Hubble was eventually drawn into the cargo bay and docked to the waiting servicing pallet. In all, most of the second day was spent getting to and capturing the Hubble. So what was going on with the long waits and slow pace of things? Astronaut (talk) 05:38, 14 May 2009 (UTC)[reply]

It's probably something to do with trying to synchronise your own speed with that of the object you are trying to capture. After all, they are both travelling at 16,000 mph. They're not just floating. And after all, they don't want to break Hubble, otherwise that would make the mission pointless, so they have to be careful and do a lot of preparation and checks inside the spacecraft before they start to do anything.--KageTora - (영호 (影虎)) (talk) 06:35, 14 May 2009 (UTC)[reply]
They are being exceptionally cautious - both the Shuttle and the Hubble Space Telescope are stupidly expensive pieces of machinery (in Hubble's case, nearly irreplaceable). So they pause after every step to make sure everything is still working right, that everything is as it should be, before proceeding to the next step. Also, keep in mind that the Hubble weighs eleven tons - that's not the kind of mass you want to have crashing into your spacecraft at high speed! Hence, all the manoeuvring is done very slowly, again, to avoid breaking anything (the same way as you walking more slowly when carrying a tray of glasses filled with juice). — QuantumEleven 11:06, 14 May 2009 (UTC)[reply]
Or to put it bluntly, when YOU have an eleven-billion dollar ride, you can drive it however fast you want. Nimur (talk) 13:35, 14 May 2009 (UTC)[reply]

global positioning system

what iz global positioning system —Preceding unsigned comment added by 117.198.240.128 (talk) 10:08, 14 May 2009 (UTC)[reply]

Please see our article on Global Positioning System and come back if you have any further questions. --antilivedT | C | G 10:10, 14 May 2009 (UTC)[reply]
GPS = you can haz your location... 79.122.112.53 (talk) 11:16, 14 May 2009 (UTC)[reply]
Wouldn't cats prefer to csn haz Cheeseburger Positionig System? -- Ferkelparade π 11:25, 14 May 2009 (UTC) [reply]
Cats can haz needz both CPS and GPS. It's useless to know only where cheezburger is - if you don't know your relative position to it, you can't haz it. Nimur (talk) 13:33, 14 May 2009 (UTC)[reply]
Need lotz and lotz of lolSats. SteveBaker (talk) 19:09, 14 May 2009 (UTC)[reply]
This article may help. Cuddlyable3 (talk) 13:35, 14 May 2009 (UTC)[reply]
Wow! Thanks, I was not aware of a Simple Wikipedia. Jay (talk) 08:45, 15 May 2009 (UTC)[reply]

SI to cgs

hi, i've got this question's numerical answer but i'm confused about the units.

the question is : convert 7350 N/m2 into cgs units

what i did: 7350 * 105 gcm-2s-2m-2

7350 * 105 gcm-2s-2(100 cm)-2

7350 * 105 gcm-2s-2 (10000 cm)-1

7350 * 10 gcm-2s-2cm-1

this is where i'm confused. the cm-1 cancels the cm and the answer becomes g/cm2, but i (and you) know that it should be gcm/s2, or should the cm-1 not come at all from the 4th step to the 5th step??

(i could have done it the easy way. N/m2 is pressure. so i could have written it as pascal and converted it into barye. but, unfortunately, barye hasn't been taught in our school. so, i can't possibly use that method) —Preceding unsigned comment added by 122.50.131.71 (talk) 14:16, 14 May 2009 (UTC)[reply]

Search google for unit conversion and you can get all kinds of cool unit converter calculators. 65.121.141.34 (talk) 14:25, 14 May 2009 (UTC)[reply]
How about using dynes, which are the CGS unit of force? Google can do that conversion. Nimur (talk) 14:34, 14 May 2009 (UTC)[reply]
Your mistake is in your first line, you did not correctly convert Newtons to correct units in CGS system. Verify your units. Even in CGS, a unit of force should still be a mass times an acceleration. It looks like you mixed up the conversion of Newtons and the conversion of meters to centimeters in to the same line, and did it incorrectly. There should be no cm^-2 on that line, it should just be cm. Nimur (talk) 14:40, 14 May 2009 (UTC)[reply]
Nimur caught one mistake. The other one is that you converted m-2 incorrectly. the correct convertion would be (100 cm)-2=(10000 cm2)-1. Dauto (talk) 22:52, 14 May 2009 (UTC)[reply]

meiosis

Hi I'm a bit confused by the events of Metaphase I compared to metaphase II. In metaphase I I believe the bivalents line up randomly at the metaphase plate. This is a source of variation in meiosis. But when it comes to metaphase 2 - do the chromosomes line up randomly again before they split? Is this is another source of variation? I have read the meiosis article but am still a little confused. Please help! —Preceding unsigned comment added by 139.222.240.42 (talk) 15:30, 14 May 2009 (UTC)[reply]

As described in our Meiosis article:
  • Anaphase I separates homologous chromosomes
  • Anaphase II separates sister chromatids
If there were no recombination and no mutation, then anaphase I would generate variation (by randomly segregating homologous but not identical chromosomes) whereas anaphase II would separate identical sister chromatids. Because of mutation and recombination, sister chromatids can differ, so differences arise at both steps. Does that help? --Scray (talk) 22:30, 14 May 2009 (UTC)[reply]
Meiosis is definitely complicated, and you're not alone in being confused. When the chromosomes are copied they form identical sister chromatids that join up with the other homologous pair. The task of metaphase I is to randomly segregate the pairs of sister chromatids from each other so that the resulting cells are basically a random combination of the chromosomes that were originally inherited from your father and mother. This is clearly a source of variation and it basically describes independent assortment of alleles that are carried on different chromosomes. The task of metaphase II is to separate the sister chromatids so that there's only 1 copy of each chromosome. There isn't quite as much randomization at this point since the sister chromatids that are being separated were initially identical to each other. However, due to random crossing over that happens during the first part of meiosis, at the time of metaphase II, the sister chromatids are themselves different patchworks of the maternal and paternal chromosomes and this leads to an additional level of randomization in the final gametes. Does that help any? --- Medical geneticist (talk) 23:03, 14 May 2009 (UTC)[reply]

enhanced oil recovery

Why do they inject nitrogen into oil fields instead of regular air? 65.121.141.34 (talk) 16:13, 14 May 2009 (UTC)[reply]

Because air contains oxygen, which may chemically react with components of the oil. Nitrogen is chemically inert and will not produce any undesirable reactions. --Jayron32.talk.contribs 18:20, 14 May 2009 (UTC)[reply]
Oxygen is also a key component of combustion. Nitrogen is safer. cheers, 10draftsdeep (talk) 18:28, 14 May 2009 (UTC)[reply]
Particularly under the high pressures gas is injected at under EOR. TastyCakes (talk) 18:47, 14 May 2009 (UTC)[reply]
I have been under the impression that unrefined crude oil is not very easy to ignite, though I suppose that a lot of safety measures are needed since it would be a very expensive and dangerous fire. 65.121.141.34 (talk) 18:49, 14 May 2009 (UTC)[reply]
Well yes and no... At high pressures and somewhat high temperatures, as experienced in most oil and gas reservoirs, crude is probably somewhat combustible, at least that's my understanding since fire flooding is used in places (no wikipedia article on it, but here is a patent on the subject). That coal seams can burn for as long as they do seems to support that. But the real danger is that "light ends" (short hydrocarbon chains) will come off of even the heaviest crude oil to some degree, and these are very combustible. Add to this that oil wells will frequently encounter gas and gas condensates (in varying quantities) and I think it is clear that there is enough flammable stuff in an oil well to justify avoiding the use of oxygen. TastyCakes (talk) 20:15, 14 May 2009 (UTC)[reply]
You'll find nitrogen used in other applications as well. For example, most beer kegs use Carbon dioxide as a propellant, since it is a natural product of the fermentation process, so adding extra CO2 to the system seems like a fine idea. However, Guinness and a few other high-end specialty beers will use nitrogen instead. CO2 will dissolve in water to form carbonic acid which will alter the flavor of the beer. For really malty beers like stout, the additional sour flavor from the acid can change the flavor profile in ways the brewer does not want. Nitrogen will not produce a detectable change in flavor, plus being less soluble in water, nitrogen tends to produce smaller bubbles than CO2 does, which produces the characteristic "waterfall" effect of a well-poured pint of stout. --Jayron32.talk.contribs 18:56, 14 May 2009 (UTC)[reply]
They also offer it as a substitute for air in tires, claiming it leaks out less (or more slowly) due to the nitrogen molecules' larger size. TastyCakes (talk) 20:51, 14 May 2009 (UTC)[reply]
Well, that doesn't make any sense... Air is Nitrogen, Oxygen, and a Carbon Dioxide (and negligibly small quantities of anything else) (on checking, Argon is actually more common that CO2... but I think we're already in the "negligible" domain). Nitrogen is the lightest molecule of those, with a higher drift-velocity. It would leak out faster than air. It could be argued that less inert nitrogen reacts less with the tire rubber/synthetic, but that's also flimsy logic compared to the rate of mechanical, frictional wear against the road surface. Sounds like a scam... Nimur (talk) 21:31, 14 May 2009 (UTC)[reply]
It may be lighter, but it has a larger radius than Oxygen (although I can't find an exact number right now). I don't think you could really use CO2 because of acidity issues, and any other "inert gas" is likely to be too expensive and no better than nitrogen, size-wise. I don't think pure Oxygen would really be an option either, due to corrosion and explosion dangers. They also claim that having pure nitrogen without any moisture has benefits, including reduced corrosion (not a big issue as far as I know). I tend to side with you, however, without a proper study showing otherwise I am skeptical as to the benefits of it. Here is an article on the subject. TastyCakes (talk) 21:47, 14 May 2009 (UTC)[reply]
I'm pretty sure drift velocity will affect diffusion (leaking) a lot more than atomic radius. The pore sizes are less of an important factor than the statistical likelihood that an individual gas molecule will get to a pore. Thus the lighter, faster gas diffuses faster. See Graham's law for more on differential rates of gas flow. N2 will diffuse about 7% faster than O2. If diffusion leaking were actually occurring on relevant timescales, a 7% worse performance would be noticeable. I conclude that "leak reduction" is not at all the motivation for Nitrogen in tires OR enhanced oil recovery. It's much more likely that chemical reactivity is the issue in both cases (although of dubious benefit for at least the tire case). Nimur (talk) 22:34, 14 May 2009 (UTC)[reply]
Surely it is more a matter of the average pore size/hole size of vulcanized rubber than anything else? I would also think the viscosity of the two gasses at tire temperature would play a bigger part than their drift velocity for long term leak purposes, although I don't know the values for nitrogen or air. In any case, you don't have to convince me, I'm not going to buy it any time soon ;) TastyCakes (talk) 22:41, 14 May 2009 (UTC)[reply]
(undent) Just to correct an inaccuracy above, if gas is leaking out of pores, the pores are likely millions of molecular radii across, and so the size of the molecule has NO bearing on the issue. All that matters is the molecular mass of the molecule, because at any given temperature, heavier molecules move slower, which means they "hit" the hole less often than do lighter gases. That is the only relevant property when considering this. Thus, regardless of the volume or linear size of the nitrogen molecule vis-a-vis the oxygen molecule, its only the mass that affects the leakage. See Graham's law for more info. The third paragraph of the "history" section explains it quite well. --Jayron32.talk.contribs 22:53, 14 May 2009 (UTC)[reply]
Hmm ok I guess that makes sense. Do you know how big the "holes" are in tire rubber? Are they really that much larger than nitrogen molecules? TastyCakes (talk) 14:10, 15 May 2009 (UTC)[reply]
Removing oxygen also avoids the development of bacteria (bugs) that can foul a well. Besides avoiding other chemical reactivity I think this is the major reason for oxygen and carbon dioxide removal. They even have to degas any surface water they inject.--OMCV (talk) 13:07, 15 May 2009 (UTC)[reply]

what is nitrostyrene?

wat is it made up of..ect. anybody know? its not on wikki —Preceding unsigned comment added by 66.237.50.35 (talk) 16:17, 14 May 2009 (UTC)[reply]

Assuming the nitro group is not directly bound to the benzene ring, then this is what it is: http://www.orgsyn.org/orgsyn/orgsyn/prepContent.asp?prep=cv1p0413 --Russoc4 (talk) 17:10, 14 May 2009 (UTC)[reply]
To answer the poster, a chemical name tells you exactly what it's made up of: it's a nitro attached to a styrene molecule. You can see that styrene has several different shapes/parts, so what we can't answer is exactly where the nitro is attached. Russoc mentions one possibility (that compound has some interesting chemical reactions possible!). Also, meta-nitrostyrene and para-nitrostyrene (with the nitro attached to the benzene ring in various places) have some uses, especially in their analogous nitropolystyrene polymer form. Apparently you can either polymerize the nitrostyrene directly or else polymerize styrene (or some other derivative) and then nitrate it. DMacks (talk) 20:08, 15 May 2009 (UTC)[reply]

KCN

What is the saturation point of KCN? Where would I find this information? 128.193.170.120 (talk) 17:50, 14 May 2009 (UTC)[reply]

Is there any kind of technology on the internet that lets you take a climate chart of a particular location, provided by the site, and do a search to come up with all the climate charts around the world that correspond most closely with the initial chart? This climate chart would include average monthly highs and lows and average monthly precipitation. —Preceding unsigned comment added by 189.4.53.90 (talk) 19:18, 14 May 2009 (UTC)[reply]

Rockets

We currently almost always launch rockets east when going to orbit (to benefit from the rotation of the earth). My question is if there is any consequence to always launching east, like possibly slowing the length of the day since you're pushing the earth back?

I'm sure the effect is tiny, but what if it's 500 years from now and we have billions of people launching megaton spaceships eastward to save on their fuel costs? Would the effect then become noticeable? TheFutureAwaits (talk) 19:31, 14 May 2009 (UTC)[reply]

Actually, some rockets are lunched north into a polar orbit, but you are right, most rockets are to the east. The earth weight is so high, and its spin rate so fast, that in order to slow the length of the day an amount that will be perceptible to people without clock accurate to 10 decimal places, rockets will have no effect until such point that we are launching in the trillions of tons per year range. At that point, there might be an effect, but it will be a very polluted planet with all the rocket exhaust. 65.121.141.34 (talk) 19:55, 14 May 2009 (UTC)[reply]
I think, in the US at least, the only reason rockets are launched east is so they travel over uninhabited water, lessening the adverse consequences of an aborted/failed launch. I don't know about launches outside of Cape Canaveral, though. I am not aware of any physical benefit of always launching east, other than impact speeds of space junk from previous launches would be lessened. As for affecting the rotation of the earth, the wind has a much greater effect than our various items in orbit, by several orders of magnitude. The tidal forces from the moon have an even greater effect. Unless we eventually have a space station a significant percentage of the mass of the moon, there should be no discernable effect. -RunningOnBrains 20:03, 14 May 2009 (UTC)[reply]
Actually ROB, if you launch east, you already are traveling at 700-1000 mph, so your rocket does not need quite so much fuel to get into a speed high enough for orbit. If you launched west, you would have the opposite effect and your rocket would start at -1000 mph instead of +1000 mph and would need to increase your speed 2000 more mph then an east launch. 65.121.141.34 (talk) 20:14, 14 May 2009 (UTC)[reply]
What goes up comes (usually) down. Launching a rocket to the east is not done by tilting the rocket on the launch pad. Cuddlyable3 (talk) 20:22, 14 May 2009 (UTC)[reply]
No, he's right...that was dumb of me. Launching east does give an angular boost.-RunningOnBrains 20:27, 14 May 2009 (UTC)[reply]
The angular momentum of the a closed system is conserved, so system consisting of the earth plus the artificial satellites would have a constant angular momentum, which mens the earth wourl slow down, IF the system were closed. However, the system is not closed. the eastward launch requires the expulsion of rocket exhaust in the westward direction.IF the exhaust exceeds escape velocity, then the eastward angular momentum of the system increases. Note that this effect and the transfer effect you originally asked about are trivial by comparson to the effect of tidal drag, which transfers angular momentum from earth rotation to lunar orbital distance. -Arch dude (talk) 02:07, 15 May 2009 (UTC)[reply]
If and when we reach the technological level of launching trillions of tons into space, i am sure we'll have methods to counter this effect. --131.188.3.21 (talk) 11:15, 15 May 2009 (UTC)[reply]

May 15

Can tame, captive-bred animals be used to breed offspring that can be returned to the wild?

I notice that my captive-bred pet sun conure is a member of a species that is endangered in the wild. As far as I know, there are loads of sun conures in captivity. There might even be more kept as companion parrots in people's homes than there are in the wild. So, would it be possible, if the need ever arose, to breed pet sun conures and release the young ones (that weren't familiar with humans) back into the wild? Or would they be unable to survive? Thanks. --90.241.160.124 (talk) 00:03, 15 May 2009 (UTC)[reply]

Does Captive breeding answer your questions? Tempshill (talk) 01:02, 15 May 2009 (UTC)[reply]
Not really. I'm interested in knowing if animals that behave exactly like they would in the wild can ever be bred in captivity from parents that are pets? We can talk about sun conures specifically, if that makes it any easier. Thanks. --90.241.160.124 (talk) 01:39, 15 May 2009 (UTC)[reply]
Do you mean to ask if is possible to intentionally create a feral population? Dauto (talk) 02:05, 15 May 2009 (UTC)[reply]
I suppose I am, yeah. But only in the areas where the bird is or was found naturally. --90.241.160.124 (talk) 02:45, 15 May 2009 (UTC)[reply]
Amazingly we don't have an article about the parrots of Telegraph Hill. If escaped parrots can adapt to such an unnatural environment, one would think they should be able to readapt to their native environment. Looie496 (talk) 03:11, 15 May 2009 (UTC)[reply]
Reintroduction may have some information for you. There is a difference between having the same skills as a member of a wild population, being able to integrate into and survive within a wild population and having sufficient skills to survive in the wild. Just to illustrate: A captive bred conure may not know the warning call a wild one would use for a panther. If it doesn't get eaten by one, it or it's descendants may be able to learn that. The captive conure may not have the proper social skills and knowledge about nesting to make it within a wild flock. If they are bred in sufficient numbers and with sufficient diversity to form their own flock they may develop skills they can pass on to their next generation, even if those are different from wild conure behavior. For example the parrots of Telegraph Hill might have a call for "hawk" that their rainforest relatives may not have or need. This form of reintroduction usually involves a high attrition rate when scientists have tried it. Successful reintroduction programs usually involve lots of steps, close monitoring and possibly several generations. An animals survival strategy also plays a role here. Some animals just breed in such large numbers that they can stand losing a lot of individuals. Animals with low reproductive rates usually invest in longer training periods and parental care to ensure their offspring make it. Hope this helps.71.236.24.129 (talk) 04:46, 15 May 2009 (UTC)[reply]
Domestic cats can product feral offspring. If a domestic stray cat has kittens, and they are not socialised (introduced to humans) within a certain time period, they will difficult or impossible to tame or introduce successfully into a domestic environment. I'm not sure we'll ever need to breed cats for release into the wild, though. --Kateshortforbob 11:22, 15 May 2009 (UTC)[reply]
Cats are a whole different ball game. I have successfully reintroduced several feral cats into domestic environments. It just takes an awful lot of effort and a certain type of home. (A townhouse apartment won't work. A barn would be ideal.) Since our shelters are overflowing with domesticated cats shopping for a home it's usually not worth the trouble. I even used to have a semi-tame wild cat that I could pet and feed (while the door to the outside remained open.) Most experts will tell you they're impossible to domesticate. Domestic cats do not need to be reintroduced into their natural habitat - which would be people's homes. Other Felinae like Felis silvestris, Prionailurus viverrinus, Felis margarita and Felis manul are in decline and captive breeding and reintroduction programs are under way. They face many different challenges like diseases, difficulty breeding, loss of prey and loss of habitat or human encroachment. The target here is not to domesticate them, to make them pets, and then reintroduce their progeny into the wild, but to keep breeding pairs in human controlled environments or zoos and then prepare their offspring for release into the wild. Those would still not be feral cats though. With zoo raised cats you have the same issues with reintroduction as described above, which is why they usually try to keep them in reserves or sanctuaries. With both cats an conures the question becomes where the "wild" that you want to reintroduce them into can still be found. 71.236.24.129 (talk) 14:35, 15 May 2009 (UTC)[reply]

Yes I believe that individual home reared Conures can successfully survive in what is left of thier native habitat. My largest concern would be with the diversity and size of the gene pool. Geneticists will tell you that without a "certain" amount of diversity and a certain number of indivivuals (there is a formula for such)a reintroduced population is doomed to failure. Wikipedia probably has articles on genetic bottlenecking, gene pools and other relevant topics but i don't know how to link them here. 67.193.179.241 (talk) 12:03, 15 May 2009 (UTC) Rana sylvatica[reply]

Huntington's disease in Scotland

I read in one source that Huntington's disease has an extraordinarily high prevalence in "some areas of Scotland". Which ones? 99.245.16.164 (talk) 00:20, 15 May 2009 (UTC)[reply]

The Moray Firth apparently. SpinningSpark 06:28, 15 May 2009 (UTC)[reply]

Making the Rage Virus

Is it possible for scientists to create a virus that could make humans rage with anger and cause them to kill other humans and animals? 174.114.236.41 (talk) 01:20, 15 May 2009 (UTC)[reply]

It already exists -- it's called intolerance. Wikiant (talk) 01:25, 15 May 2009 (UTC)[reply]
... and it's already mutated into a virulent strain, increasingly common in urbanized areas especially where bitumen or asphalt takes up a large proportion of the land surface area, and is known colloquially as Road rage. Fortunately, there are treatments available. BoundaryRider (talk) 02:01, 15 May 2009 (UTC)[reply]
Well how about an actual useful answer? A virus like this somewhat already exists. The rabies virus turns dogs and foxes into crazed killers - but fortunately, not so in humans (although it's nasty enough without that particular symptom) - but if such a virus is possible in other mammals, I could imagine something similar happening in humans. Our article explains that "In many cases the infected animal is exceptionally aggressive, may attack without provocation, and exhibits otherwise uncharacteristic behavior."...and..."The first stage is a one- to three-day period characterized by behavioral changes and is known as the prodromal stage. The second stage is the excitative stage, which lasts three to four days. It is this stage that is often known as furious rabies due to the tendency of the affected dog to be hyperreactive to external stimuli and bite at anything near."...and also..."a new symptom of rabies has been observed in foxes. Probably at the beginning of the prodromal stage, foxes, who are extremely cautious by nature, seem to lose this instinct."
That's a virus that's transmitted in saliva and has therefore evolved to cause its victims to go nuts and start attacking and biting.
SteveBaker (talk) 02:46, 15 May 2009 (UTC)[reply]
There are lots of infectious agents which do induce delirium and brain damage in their hosts. Depending on what parts of the brain are affected, it is not unreasonable to find an infectious disease which would cause the result described by the OP. --Jayron32.talk.contribs 04:21, 15 May 2009 (UTC)[reply]
The challenge is finding a virus that consistently targets brain areas in just the right way to induce a behavior like the "furious rabies" that Steve's (useful) answer cites. I don't think scientists currently have the knowledge or tools to design such an agent. --Scray (talk) 04:57, 15 May 2009 (UTC)[reply]
It would probably be quite easy to do this, one doesn't need to know the mechanism to select for things like this. After all somebody with this level of morals wouldn't mind human testing. It is totally unnecessary though, it seems altogether too easy to get whole nations to take up arms against others and bomb and torture and otherwise behave vilely. You want a way of doing this even less controllably? 08:07, 15 May 2009 (UTC)
In case anyone has missed the reference, the "Rage virus" is the premise of the film 28 Days Later. --Tango (talk) 13:59, 15 May 2009 (UTC)[reply]
It is definitely possible, though how hard it is I don't know. What you need is a virus that can get into the bloodstream to have DNA that will code for enzymes that will synthesize a psychotomimetic. The DNA/RNA for enzymes is easy to extract from plants/fungi (ergot, atropa belladonna), but I dont think these drugs (lysergamides, atropine, respectively) would cause people to start killing each other, although people under the influence of deliriants do sometimes accidently kill themselves (I can't find the ref right now, but some english guy flew to new york and took too many nytols to overcome jet lag. He was found in his hotel room bleeding from the head, he had smashed his head in with a lamp or something along those lines. Google isn't finding it unfortunately). --Mark PEA (talk) 17:06, 15 May 2009 (UTC)[reply]

Partial Pressure

Hello. There is an equal amount of nitrogen gas and hydrogen gas in a balloon that has a pinhole leak. There is no word of where the leak is exactly on the balloon. The temperature stays constant. Why will the partial pressure of nitrogen gas exceed that of hydrogen gas? Thanks in advance. --Mayfare (talk) 03:53, 15 May 2009 (UTC)[reply]

Sounds like homework, but anyway, you can read about Graham's law here just as well as in a science textbook. DMacks (talk) 04:12, 15 May 2009 (UTC)[reply]
Which molecules are moving faster? So which molecules will hit the pinhole more often? So which molecules will leave the balloon faster? So what happens to the ratio of the gases in the balloon? Answer these questions, or just read the Graham's law article cited above. --Jayron32.talk.contribs 04:20, 15 May 2009 (UTC)[reply]
Two! Two Graham's Law answers on WP:RD/S! Ah ah ah ah ah. Count von Count (talk) 04:49, 15 May 2009 (UTC)

Yet another perpetual motion machine

I know it wouldn't work. Tell me why. An electric transformer generates a voltage on one side proportional to the change in voltage on the other. If you put an exponentially increasing voltage one one side, that should generate an exponentially increasing voltage on the other, which would generate an exponentially increasing voltage on the first. This would make a self-sustaining increase in voltage. Another way to do the same basic thing would be by moving a charged particle along an exponential path. The Abraham–Lorentz force will create a force on it proportional to its jerk. — DanielLC 05:10, 15 May 2009 (UTC)[reply]

energy is lost as heat. —Preceding unsigned comment added by 82.44.54.169 (talk) 05:38, 15 May 2009 (UTC)[reply]
OP: memorize those five words, because it is (probably) the answer to most any perpetual motion machine you could think of (except the fancy stuff you could try to do with magnets, which might end up depeleting their own magnetic charges over time, and slow down for that reason). As to why, thermodynamics should give you the answer to that... —Preceding unsigned comment added by 79.122.38.222 (talk) 10:57, 15 May 2009 (UTC)[reply]
Daniel, by the Lenz's law the induced electromotive force back on the first side should opose the one you had to begin with, and not reinforce it. Read the article I linked. Dauto (talk) 12:56, 15 May 2009 (UTC)[reply]
1)The induced waveform would only be proportional to the input exponentially increasing waveform until the magnetic core material saturated. 2)There is a frequency response limitation in a transformer; it does not have infinite bandwidth, so the output waveform would not be able to reproduce an input waveform with too high of a rise time. The induced voltage from a cable thumper or lightning may not be able to travel through a power transformer, for instance, because the impulse is too brief. (Safety note: Don't experiment with this at home.).Edison (talk) 18:11, 15 May 2009 (UTC)[reply]

molarity

here's the question: what volume of 36M and 1M sulphuric acid must me mixed to get 1L of 6M sulphuric acid?

i need some explanation. don't just give me hints... explain those hints too, please... i'm new to this concept. —Preceding unsigned comment added by 122.50.136.97 (talk) 05:14, 15 May 2009 (UTC)[reply]

First you need to figure out the proportions needed to get a 6M concentration. Then you can figure out the amounts to give you the total volume you need. If you add X liters of 36M (36 moles/liter) solution to Y liters of 1M (1 moles/liter) solution, figure out the total number of moles that gives you and the total volume in terms of X and Y. Now the total number of moles divided by the total volume gives you the new molarity, which you want to be 6. That gives you enough information to form an equation that you can reduce down to a ratio between X and Y. From there, it should be a simple matter of calculating the amounts of each solution needed to give you the desired 1 liter result. -- Tcncv (talk) 05:41, 15 May 2009 (UTC)[reply]
Example: Suppose you had 1M and 5M solutions and needed 1 liter of 2M solution. Using the above approach, X liters of 1M solution added to Y liters of 5M solution gives you 1X + 5Y moles in X + Y liters. To get a 2M solution, set (1X + 5Y)/(X + Y) = 2. This can be rewritten as X + 5Y = 2(X + Y) or 3Y = X. So you need three parts 1M solution to 1 part 4M solution, this works out to 3/4 liters and 1/4 liters respectively to give you the desired amount. -- Tcncv (talk) 05:51, 15 May 2009 (UTC)[reply]

Gene Accession to Protein ID conversion

I am attempting to convert a large (1000s) list of gene accession numbers into their corresponding protein IDs (SwissProt/UniProt accession). The site [[16]] performed the task, but only returned protein accession numbers for ~80% of the supplied gene accession numbers, seemingly lacking the SwissProt IDs to the more recently characterized proteins. I looked at Pubmed, and found it to be more complete; but, as far as I know, the IDs can only be retrieved individually there rather than by a large list, which is obviously inefficient. I was wondering if there is a similar program to these with (ideally) the completeness of pubmed and the speed of idconverter. (NB: in addition to the gene accession numbers, I also have, or can acquire, the gene names and common names.) Any help will be greatly appreciated!Lashyn (talk) 06:03, 15 May 2009 (UTC)[reply]

Lashyn, I don't have an answer but I suggest you ask User:AndrewGNF who runs the Protein Box Bot. The bot seeds gene articles with the type of information you are trying to get. I'm not sure if Andrew has run across the same problem or not, but its worth asking. David D. (Talk) 06:56, 15 May 2009 (UTC)[reply]
I believe that SwissProt requires a pretty high level of evidence in order to get an accession number and there simply may be no SwissProt ID for every gene ID. The NCBI RefSeq genes are highly curated but other gene IDs may be predicted coding sequences, etc. The fact that you found SwissProt accession numbers for only 80% of the gene accession numbers is probably about right. Another way to look at it would be to take all SwissProt accession numbers and then find the corresponding Gene IDs. Then you'll have a complete list of the GeneIDs that CAN be linked to a SwissProt ID. --- Medical geneticist (talk) 12:24, 15 May 2009 (UTC)[reply]

Wet, Soap, Wash, Rinse, Dry

Since Swine Flu became the new thing to worry about i've seen a marked increase in how to wash your hands posters in toilets. Fair does, it's cheap to roll out and I suspect does make people more likely to wash their hands (reminders often do - even for seemingly insanely normal things). Anyhoo my question was this...I prefer to Soap, wet, wash, rinse, dry - does that (changed) ordering make any difference? Will my hands be less clean because I didn't wet before soaping? I guess it's nothing but what are Friday afternoons for if not trying to get answers to (largely) pointless questions? 194.221.133.226 (talk) 13:25, 15 May 2009 (UTC)[reply]

I'm sorry, but your order of doing things is wrong, and you now probably have Swine Flu. Please see a doctor. (Just kidding, of course. I don't think it matters a whole lot if you put the soap on before you've wetted your hands, as long as you take the time to wash and rinse.) --98.217.14.211 (talk) 13:42, 15 May 2009 (UTC)[reply]
I agree - your imminent death soon after you fall over into a muddy puddle and roll around snorting contentedly - is now virtually certain. The soap lowers the surface tension of the water and acts as a surfactant to encapsulate whatever comes off of the surface so it can rinse away more easily. You need the soap and the water to mix completely because neither soap or water alone can perform this magic trick. Whether you add soap to water or water to soap isn't so critical...although I guess if you first cover your hands with soap and then hold them under running water, you may send more of it down the sink than if you'd first wetted your hands, then shut off the water and then added soap. However, since most people use VASTLY more soap than they need - it's probably just as effective your way. The new advice for me was the amount of time to wash for. They are telling kids to sing the alphabet song while they wash - since that's timed to be about 15 seconds. On NPR the other day - they recommended that adults sing "Bohemian Rhapsody" but since that was (at the time it was released) the longest 'single' record ever made (5 minutes, 55 seconds) - I think that NPR may have been kidding! SteveBaker (talk) 14:59, 15 May 2009 (UTC)[reply]
Damn, I've been washing my hands while humming In-A-Gadda-Da-Vida. I don't have much skin left, but I laugh in the face of swine flu! Matt Deres (talk) 16:26, 15 May 2009 (UTC) [reply]
Hate to disabuse you, Steve, but the single of "Hey Jude" lasted 7:11, and that's not the only pre-1975 single longer than "Bohemian Rhapsody." Deor (talk) 21:49, 15 May 2009 (UTC)[reply]
More released singles longer than Bohemian Rhapsody, and also released prior to Bohemian Rhapsody include: 1971's American Pie at 8:33, the 1972 single version of Papa Was a Rollin' Stone at 6:54 (the album version was 11:54), 1972's Jessica at 7:30, the 1972 7:02 single verion of Layla (more successful as a single than the 1971 2:43 version). 5:55 is long, but when released it was certainly not even close to the longest single released to that point. Sorry Steve. For the record, I sing the full album cut of the Rare Earth version of Get Ready. With lava soap and a steel brush. My hands are a bit raw, but there ain't nothing on them after that. --Jayron32.talk.contribs 01:25, 16 May 2009 (UTC)[reply]
My sister is a medical student and she was telling me about their lessons in washing your hands (they have a very strict way to do it, washing each part in a very specific order, but I think that's just to make sure they don't forget a bit rather than because there is anything better about that order) and the interesting thing she said is that the key thing for getting rid of viruses is the mechanical action of scrubbing/rubbing/whatever your hands. The soap doesn't do all that much and, obviously, antibacterial agents don't help against viruses. So, I think the advice to wash your hands for a significant amount of time is probably good. --Tango (talk) 17:47, 15 May 2009 (UTC)[reply]
My wife used to be an operating department nurse - and that whole scrubbing-up ritual really does get burned into the soul. She hasn't worked at that job for 5 years - but you can still catch her standing there with her hands held up like a kangaroo when she's deep in thought because she spent so long being told to keep her hands firmly above waist height after scrubbing. SteveBaker (talk) 17:57, 15 May 2009 (UTC)[reply]

What is reality ?

I am a teenager, and i have read a bit about Quantum Mechanics, its interpretations, wave function collapses, and paradoxes. My question is, Where does this all leave us ?
Can we still think that the world is deterministic, that it is written on my forehead what i will do? Is there a place for free will ? Is there reality out there, or when i close my eyes does everything turn into imaginary wave functions with respect to me ? Is physics really complete, at low velocities, ie, is QM infallible in non-relativistic situations? I understand that there is a bit of a debate over these topics, but i do want to know where the scientific community stands at this point. In short, can someone please explain, what is reality ?Rkr1991 (talk) 13:41, 15 May 2009 (UTC)[reply]

Reality is anything that does not goes away after you are utterly sick of it. For example: being unemployed is a reality for many people. 122.107.207.98 (talk) 14:12, 15 May 2009 (UTC)[reply]

The uncertainties of the quantum world tend to average out at the macro scale that humans live on. So for most of the time, it can be ignored and the world treated as a more or less deterministic place. However, we humans are cunning and we can magnify quantum uncertainty to the point where it becomes visible and important (eg in the Schrodinger's Cat thought experiment). But Quantum theory isn't the only source of indeterminacy. There are many systems out there which are 'chaotic' in the mathematical sense that the outcome of some experiment on these systems depends on the initial conditions with extreme (and possibly, infinite) sensitivity. You've heard of "The Butterfly Effect" where (it is said) the flapping of a butterfly's wings can change the course of a hurricane on the opposite side of the planet a year later. Well, that may be an underestimate. It's possible that a quantum fluctuation of a single electron would be enough to change the weather patterns. So determinism exists - but only strictly within non-chaotic systems - or over shorter time/distance scales - and not at very small scales. However, we can still use basic victorian-style physics to build bridges that don't fall down and airplanes the size of large buildings that seem to defy gravity by exploiting classical airflow laws. We construct amazing things that work pretty much as we expect them to - and that's only possible because determinism is reliable in most cases.
I don't believe there is free will. What goes on inside the brain is a matter of cellular biology - which is basically chemistry - which is basically physics. There is no magical 'outside' will that changes what happens in the physics - and either with or without quantum or chaotic determinism, what is the thing that is making the choice for that chemistry to either go out for Chinese take-out or stay home and cook left-overs? Ultimately, chemistry is what drives that. But it doesn't matter because it seems to us like we have free will - so we might as well act as if we do.
Whether there is a "reality" out there or not is a non-falsifiable proposition - there is no conceivable experiment you can do that would show that it's not all a figment of your imagination. You've seen "The Matrix" - right? So science takes the view that you can assume that nothing really exists and that's OK - but it's a fairly pointless thing to do. It doesn't advance your knowledge, further your career or make you happier - so you might as well assume that reality exists and carry on playing with it.
Physics certainly isn't complete. We don't know about the Higgs Boson for example - we've built this gargantuan machine to test for it - and (sadly) the machine is currently broken. But even in the realms of the "normal" world - there are unknown things. We're still having trouble figuring out WTF happens when you take a "Wint-O-Green" Life Saver into a very dark place and crush it with a pair of pliers causing it to emit a brief spark of light.
Quantum mechanics is more than just a hypothesis. It's extremely well tested. If quantum mechanics were just a theoretical possibility, the computer you are sitting at wouldn't work because the flash memory inside relies on quantum effects to do what it does. If quantum theory didn't work, we'd never have figured out how to make blue LED's.
SteveBaker (talk) 14:46, 15 May 2009 (UTC)[reply]

An illuminating answer, i must say, but still more questions. You talked about the chaos theory. Even though the effects are very hard to predict, it remains that the situation is , much like the tossing of a coin,in principle, intrinsically deterministic. So no problem with there. Reality. My question remains unanswered. Will the world be reduced to complex wave functions when i blink ? Does there or doesn't there exist things that exist whether or not i see it ? Is the electron already there, or does it choose to appear there as i have forced it to make a choice by observing it ? As Einstein said, does the moon exist only if i look at it ? And you said no free will. Which pretty much means it is already decided what choices we will take. Which pretty much means determinism. But doesn't the uncertainty principle forbid this ? If something can be anywhere now, how can it be at some particular point in the future ? I'm sorry if i sound aggressive, buut these debates have been raging in my mind for quite some time now, and i had to ask an expert.Rkr1991 (talk) 15:02, 15 May 2009 (UTC)[reply]

I think you're confused by the often-quoted point of quantum mechanics where "observing" a system changes it. I don't like this word "observed" because it implies that sentient beings have some sort of power over the laws of physics: this is not the case. "Observation" simply implies interacting with another wave/particle/wavicle, so that its position and momentum at one point in time is known (whether or not a human happens to be watching to record the evidence of that interaction).
The answer to your question is yes, as far as we know, QM is infallible. But this is easy to say, because we have defined that QM only "applies" for slow and light (think non-heavy, not photonic) objects. Through predictions, experiments, and results, scientists have narrowed down the cases where QM makes good predictions, and determined that when certain conditions are present QM spews nonsense. It's easy to say that a theory is "infallible" when you only apply it to special cases.
As far as your latest post, you ask "If something can be anywhere now, how can it be at some particular point in the future?" It isn't fair to say that just because a particle's wavefunction is non-zero everywhere that it can be "anywhere". The probability of me tunneling (think quantum, not creepy) through the earth to appear next to your computer is, technically, non-zero, but that is not going to happen.
Take all of this with a grain of salt: I am, at best, an advanced beginner at quantum physics. I'm sure SteveBaker will correct me if I'm wrong :-D -RunningOnBrains 15:50, 15 May 2009 (UTC)[reply]
Regarding the original question, the Science Desk is really not very well suited to resolve these sorts of doubts. You have to go read some books. Looie496 (talk) 16:04, 15 May 2009 (UTC)[reply]

I don't think Rkr1991 should be so quick to brush away chaos theory. The important point is that chaos theory acts as an uncertainty multiplier. The teeny-tiny uncertainty of whether the butterfly flaps it's wings now...or...(wait for it)...NOW! can make the difference between that hurricane happening or not. The chaotic nature of (for example) the weather magnifies the degree of uncertainty from something that really doesn't matter at all - to something really major. What that means is that chaos theory can magnify the literally, fundamentally, non-deterministic things that happen in the quantum world up to the macro scale where humans are affected by the results. We cannot - even in principle, with infinite computing power and perfect knowledge - usefully predict the weather more than maybe a month into the future. Taken together, quantum theory and chaos theory mean that even at the large scales where we like to think that the 'real world' consequences of quantum uncertainty is negligable - it's not. SteveBaker (talk) 17:49, 15 May 2009 (UTC)[reply]

I would like to emphasize the fact here that i am not talking about what is going to happen in practice , merely the principle. Practicality notwithstanding, since the wavefunction is non zero everywhere, the electron can indeed in principle be found everywhere. Coming to practical terms, ok, my electrons can never be found in the sun, but that doesn't mean you know where it is now. The fact lies that there is a finite measurable indeterminacy in position of the electron, and now i can say that "If something can be anywhere now, how can it be at some particular point in the future?" But you are right, i am confused a bit about that observing part... the problem is that most books are too abstract or complex for me to plough through... I just want to get the picture, the principle, the idea, not everything in its mathematical beast-like form...Rkr1991 (talk) 16:34, 15 May 2009 (UTC)[reply]

And if the universe is infinite (or even very VERY large) - then somewhere on some far distant planet - a half ton rock has indeed just jumped four feet to the left and turned into a mauve grand piano without any provocation. In principle, that could happen to you, right now. (Please let me know if it does...that would be fun to know!) It is only that the probability is so amazingly low that we may ignore it for all practical purposes...and even theoretical ones for that matter! However, it is only the quantum uncertainty of that electron "tunnelling" that enables your computer's flash memory to work...so at the level of the very small, this effect dominates the way the universe works. SteveBaker (talk) 17:49, 15 May 2009 (UTC)[reply]

Ok, I'm just going to take a shot at this, but what I think the world would look like when you close you're eyes is just atoms reacting with each other. So things that you can see when you have you're eyes open, like light, are just the photons of light hitting electrons and reflecting off into space, to be reflected off into whatever. But when you're eyes are open, you're brain and interpret these waves into the color spectrum and creates shapes and sizes. But then I think even there is still kind of low resolution, if you consider string theory. I don't know a lot about it, and correct me if I'm wrong, but tiny little vibrating strings make up everything, and some are looped into themselves while others are just wiggly lines. So if you could see the true "reality," then you'd see all these little tiny strings are that are vibrating and interacting with each other. 129.21.109.153 (talk) 17:12, 15 May 2009 (UTC)[reply]

I think the whole "blinking" or "not looking" thing is a bad interpretation. If you don't look at the Sun, is it still there? Well the tan/sunburn that you will get implies that it is. When your eyes are closed, you are just blocking light waves from reaching your retina, but I know you aren't looking at things from this perspective. You are asking whether "reality" exists if you are in a coma, for example. I guess that depends on what you define as reality, and this is more of a philosophical question than a scientific one. As for free will, I personally have no problem with accepting it doesn't exist, in fact, I feel like I have no control over my life (I realise this may just be a symptom of depersonalization disorder, but I don't fit other criteria), but I always notice how my biology affects me. If I'm in the shower and it starts to feel cold, I turn the heater up. If I'm cold I go and put my jacket on, If I see a question on Wikipedia which I believe I'm capable of producing a decent answer to, I reply to it. Etc. In terms of philosophy, I would be classified as a physicalist determinist. --Mark PEA (talk) 17:29, 15 May 2009 (UTC)[reply]

There was some recent discussion about a related topic on this very page. Make sure you read my coments as well as BenRG's. To avoid repetition I will only coment about free will here. Can anybody explain me why people equate determinism with lack of free will? That makes no sense to me. I think it is exactly the oposite. In order to have free will, we need a deterministic world otherwise people would go around doing random things instead of doing things they want to do. Dauto (talk) 19:44, 15 May 2009 (UTC)[reply]

Assuming free will refers to a conscious ability to make decisions, in a deterministic universe, all our decisions would have been determined (or determinable) before we were born. There's no space for free will there. Zain Ebrahim (talk) 08:00, 16 May 2009 (UTC)[reply]
What you're saying doesn't make sense to me. Living organisms do what they 'want'. What an organism wants to do is a calculation based upon the information they are receiving from their senses, along with their genetic code and upbringing (and diet, drugs, ...). As I posted in the previous example (with the shower and the jacket), decisions are made to try and maintain homeostasis, although some genetic code is better at maintaining homeostasis than others (e.g. those with the short allele of the 5-HTTLPR are more susceptible to commit suicide (Caspi et al. (2003). Science. 301(5631):386-389.)). This is confirmed with the needs of shelter, water, nutrition, warmth... but doesn't really explain reproduction. The explanation for the urge to reproduce is presumably due to DNA that makes us 'want' to reproduce, because without it, we wouldn't be having this conversation. --Mark PEA (talk) 22:20, 15 May 2009 (UTC)[reply]

You're a teenager and you've hit a mine of interesting thoughts. As people have said, the way you're thinking about this isn't necessarily very scientifically useful. For one thing, you seem to be mixing 'reality' up with 'your perception of reality'. But it is fun and interesting, particularly when all these ideas are new to you. Get hold of some Philip K. Dick short stories, read the Principia Discordia (although it will probably just seem silly to you), enjoy playing with these ideas. But remember that for all practical purposes, reality exists independently of you and you exercise free will. All the rest is mental entertainment (or seeking after ultimate truth. One or the other). 80.41.104.220 (talk) 20:01, 15 May 2009 (UTC)[reply]


Quantum theory provides us with a striking illustration of the fact that we can fully understand a connection though we can only speak of it in images and parables.
However the development proceeds in detail, the path so far traced by the quantum theory indicates that an understanding of those still unclarified features of atomic physics can only be acquired by foregoing visualization and objectification to an extent greater than that customary hitherto.
We have to remember that what we observe is not nature herself, but nature exposed to our method of questioning.
The existing scientific concepts cover always only a very limited part of reality, and the other part that has not yet been understood is infinite.
Whenever we proceed from the known into the unknown we may hope to understand, but we may have to learn at the same time a new meaning of the word "understanding."
Any concepts or words which have been formed in the past through the interplay between the world and ourselves are not really sharply defined with respect to their meaning: that is to say, we do not know exactly how far they will help us in finding our way in the world. We often know that they can be applied to a wide range of inner or outer experience, but we practically never know precisely the limits of their applicability. This is true even of the simplest and most general concepts like "existence" and "space and time". Therefore, it will never be possible by pure reason to arrive at some absolute truth.

The above are selected quotes from Werner Heisenberg, one of the founders of quantum mechanics. Cuddlyable3 (talk) 20:36, 15 May 2009 (UTC)[reply]

If you've managed to wrap your head around the above, here's another bummer: Have you noticed just how much "nothing" is involved in solid matter? It still amazes me how it all works together so well that you can actually have solid objects and people instead of just random particle soup :-)71.236.24.129 (talk) 22:11, 15 May 2009 (UTC)[reply]
That is because physical science is learning more and more about what turns out to be less and less. Cuddlyable3 (talk) 10:26, 16 May 2009 (UTC)[reply]

Well, i am not a professional in this, so correct me if i am wrong. Dauto had asked why i equate determinism with lack of free will. From what i understand, Free will is taking decisions ourselves, when we have a choice, even if it is obvious, it is we who will choose. Sometimes they maybe be obvious, sometimes they may not, like coming to a fork in the road and not knowing which way to go. But the fact remains that we make the decision. Now if everything is already predetermined, then it is already known, perhaps to god, what decision we would take, say the left road in the fork (see : i made a decision here ). So if god already knew it, i was just destined to type lift road here and not anything else... Spooky : since i was destined to choose the left road, did i really have a choice ? Did i really exercise my free will ? Or in other words, in a deterministic world, does free will exist ? This is something like not punishing a criminal because he was destined to commit the crime... he couldn't have done otherwise... Note that this problem wouldn't arise in an indeterministic situation  : If even GOD didn't know what choice i would take, then it was entirely upto my mind, and me, of my own free will, chose left. I may have made a mistake somewhere, if so please excuse me... Rkr1991 (talk) 04:35, 16 May 2009 (UTC)[reply]

In terms of the punishing criminals thing: If the criminal is predetermined to commit crime, society is predetermined to lock them away. If someone punched me in the face and said "Sorry, I have no free will", then I will just punch them back and say "Me neither". What it boils down to in the end is a question of fairness. Is it fair that a child gets abused and then grows up to be a criminal, and then locked away, etc. when if they had grown up in a caring family with adequate wealth they would be going on to earn good money in a good job and live outside of prison? That is just nature I guess. --Mark PEA (talk) 09:36, 16 May 2009 (UTC)[reply]
Determinism contra free will is a contradiction that is impossible to resolve by logic. However the consequence of each choice is not experienced until after the choice is made. The resulting subjective development may be surprise, guilt, pleasure. learning or something else. Whether you regard that as predetermined or not, you cannot know it in advance. Wondering about whether something unknown might matter prompted Donald Rumsfeld's dictum Now what is the message there? The message is that there are known "knowns." There are things we know that we know. There are known unknowns. That is to say there are things that we now know we don't know. But there are also unknown unknowns. There are things we do not know we don't know. So when we do the best we can and we pull all this information together, and we then say well that's basically what we see as the situation, that is really only the known knowns and the known unknowns. And each year, we discover a few more of those unknown unknowns.Cuddlyable3 (talk) 10:22, 16 May 2009 (UTC)[reply]

Left/right and relativity

Does the left and right side undermine the relativity theory in a sense that there is an absolute benchmark - the possessor of these extremities? If the arms for example are considered left and right from the benchmark of one's own body, does it turn non-relativist, absolute? 91.135.250.34 (talk) 13:44, 15 May 2009 (UTC)[reply]

I don't think your understanding of relativity is quite correct. In general relativity, there is still a coordinate system, which means that relative positions can and do exist. The difference in relative position (how far apart your arms are) depends on who is observing and how fast they are moving and accelerating. But, under no circumstances would they "flip sides", because they are separated by a space-like interval. Maybe you heard some discussion about chirality and mixed up some terminology? Nimur (talk) 14:01, 15 May 2009 (UTC)[reply]

Identify this military equipment

Can anyone identify this thing? This image was posted on DefenseLink with the caption,

I'd like to put this image in Wikipedia on the appropriate article, but I'm not sure what it is. (Maybe a reflecting telescope?) Nimur (talk) 13:45, 15 May 2009 (UTC)[reply]

I think it's more likely a BGM-71 TOW launcher along with associated aiming mechanism. See this HMMWV-based launcher for comparison. — Lomn 14:04, 15 May 2009 (UTC)[reply]
There is a "71" on the tube, so I'd call that a solid ID. Further research indicates it's probably the M220 variant of the BGM-71. Nimur (talk) 14:18, 15 May 2009 (UTC)[reply]
Since you mention checking markings on the tube, the stenciling below the eyepiece suggests it's the TOW 2B top-down attack warhead, so the BGM-71F designation looks like the most specific for the missile itself. — Lomn 15:13, 15 May 2009 (UTC)[reply]

calculation

although the title suggests a mathematical problem, it's acutally chemistry

what is the molecular mass of a substance each molecule of which contains 9 atoms of carbon, 13 atoms of hydrogen and 2.33 * 10-23 grams of other component?

that's 108 grams of carbon and 13 grams of hydrogen. and adding them up, i get 121 grams. but how am i gonna add 2.33 * 10-23 grams?????!!! —Preceding unsigned comment added by 122.50.132.126 (talk) 13:51, 15 May 2009 (UTC)[reply]

Well, you have made a silly mistake... See, an atom of carbon doesn't weigh 1 gram. 1 mole, or 6.023X10^23 atoms weigh 12 grams. Now i think you'll do the problem...Rkr1991 (talk) 13:55, 15 May 2009 (UTC)[reply]

I expect you want the molecular mass in Daltons or "atomic mass units", not grammes. So its 108 Da from carbon and 13 Da from hydrogen. Grammes is related to Da through Avagadro's constant, so you don't need to know what element it is, nor do anything with moles in this case.YobMod 13:56, 15 May 2009 (UTC)[reply]

Biomass

Which mammal species has the largest mass (number of individuals x average weight) on earth? Is it people? 65.121.141.34 (talk) 14:11, 15 May 2009 (UTC)[reply]

A table at Biomass (ecology) says cattle outweigh humans, for one thing. Tempshill (talk) 15:38, 15 May 2009 (UTC)[reply]
Very helpful, thanks. 65.121.141.34 (talk) 16:08, 15 May 2009 (UTC)[reply]
At one extreme, you might think of the largest animals on earth - the Blue Whale. At 172,000kg each - and with only 12,000 of them left - they total about 2 billion kg. There are about 7 billion humans and at perhaps 80kg each, we reach about 560 billion kg. Even before we started on the mass extinction of the blue whales, there were only about 300,000 of them - so 53 billion kg is the best they ever managed. So perhaps we should be looking at things that are tiny but much more numerous than us. It is estimated that there are 1,000,000,000,000,000 ants in the world - but at 0.003 grams each - that only gets us up to 3 billion kg...more than the blue whales - but nowhere near as much as all the whale species taken together (and I did cheat and take all of the ant species together). So if anything is going to beat out humans, it's got to be something in the middle range - much more numerous than whales but much bigger than ants. And that's why cattle win. SteveBaker (talk) 17:23, 15 May 2009 (UTC)[reply]
Minor detail - last time I checked, ants didn't feed their young on milk... --Tango (talk) 17:41, 15 May 2009 (UTC)[reply]
I read in The Book of General Ignorance that if you take a field of cows and weigh all the worms in the field, the worms will outweigh the cows considerably. 90.193.232.41 (talk) 17:45, 15 May 2009 (UTC)[reply]
Our article Earthworm says "Darwin estimated that arable land contains up to 53,000 worms per acre (13/m²), but more recent research from Rothamsted Experimental Station has produced figures suggesting that even poor soil may support 250,000/acre (62/m²), whilst rich fertile farmland may have up to 1,750,000/acre (432/m²), meaning that the weight of earthworms beneath the farmer's soil could be greater than that of his livestock upon its surface." - but a lot of cattle are not kept on rich, fertile farmland. Here in the western world, they are mostly cooped up in gigantic factories - and out in the 3rd world, they are on barely-grazeable wasteland. In either case, the number of worms will be dramatically lower. Weighing in at perhaps a gram or two each, 1.7 million of them might constitute 2 or 3 metric tons of biomass. A cow weighs around 800kg - so if there are less than 3 or 4 cows to the acre then maybe this claim is true. Worldwide - there are about 1.7 billon cattle - each one weighs about 10 times more than a human - so even though there are around 7 billion of us - the cattle win the biomass contest by a factor of two. The worms though...well, if we go with the 250,000 to the acre for 'poor soil' and 4x1010 acres of 'arable' land in the world - then we have 1016 worms - which (at 1 to 2 grams each) is 1 to 2 x 1013 kg. Which beats out the weight of cattle at 1x1012...by a factor of 10.
So I don't know - worms or cows? Sadly, our OP asked about SPECIES - and while there is only really one species of domesticated cattle (they can all interbreed AFAIK) - there are dozens and dozens (or maybe hundreds) of species of earthworm. So while the total weight of all worms beats out the cattle - I think the cattle still win on a species-by-species contest.
SteveBaker (talk) 18:18, 15 May 2009 (UTC)[reply]
Well that and worms are not mammals. Otherwise some termite species would probably be a major possibility 65.121.141.34 (talk) 18:40, 15 May 2009 (UTC)[reply]

So small mammals perhaps? Are there any estimates of the world population of rats, or mice? 65.121.141.34 (talk) 18:07, 15 May 2009 (UTC)[reply]

Hmmm - good point. Well, rats weigh about half a kilo - so for every cow, you'd need to have more than 1600 rats...about three trillion of them world-wide...but again, to meet the OP's requirements, they have to be all of one species - and there are about 50 species of rat...so while I can't find a number for their population size - I'd be surprised if they won. http://www.snopes.com/critters/wild/rats.asp says that there are far fewer than one rat per person in the UK - and about one per 36 people in even the most rat-infested cities. Since we're looking for more like 500 rats per person...all of one species - it seems very unlikely. Mice, even less likely still because they are so much lighter. SteveBaker (talk) 18:29, 15 May 2009 (UTC)[reply]
If you want to be picky you'd have to sort out what cells are actually "human" or "cow" and what are independent organisms. I lost the figures in my files, but a significant portion of you walking around are actually bacteria, fungi and other separate entities. It may even out in the end, though, because you'd have to do that for all the qualifying mammals. 71.236.24.129 (talk) 22:31, 15 May 2009 (UTC)[reply]
That's a point worth considering. I remember a statistic that only 10% of the cells in a typical human body are human, although I'm not sure how reliable that is and I'm not it would be as drastic if you considered it by mass rather than by number of cells. If I had to guess, though, I'd say all mammals will have roughly the same proportion, so it shouldn't matter. --Tango (talk) 23:49, 15 May 2009 (UTC)[reply]

Rabbits breed like, well, rabbits - they're mammals and they're can be found pretty widely across the globe. Contender perhaps? ny156uk (talk) 23:15, 15 May 2009 (UTC)[reply]

Or Sheep? From the article it suggest the global sheep stock is 1.059.8bn. If we say 45kg (again from the article as low point for Ewes) that would be well about 45bn kg... oh well - don't you just hate it when you realise what you've gone and estimated ends up proving yourself 'wrong'? Seems that Sheep ain't nearly enough to compare with the human estimates noted above :-( ny156uk (talk) 23:21, 15 May 2009 (UTC)[reply]

http://faostat.fao.org/site/339/default.aspx might be worth a look around. Sorry should have done all this as one answer rather than the 3 half-assed answers! ny156uk (talk) 23:23, 15 May 2009 (UTC)[reply]

EARTH

Is Earth definitely symmetrical about the axis passing through through the Northern and southern most points??? Or Is it deformed ??? —Preceding unsigned comment added by 59.165.84.9 (talk) 15:21, 15 May 2009 (UTC)[reply]

Shape of the Earth. This article is a bit technical, but it's easy to answer your own question when you think about this question: how many Mount Everests are there?-RunningOnBrains 15:56, 15 May 2009 (UTC)[reply]
See also Land hemisphere. —Tamfang (talk) 05:32, 16 May 2009 (UTC)[reply]
It is very close to be symmetrical about that axis, but not perfectly so (due to oceans, continents, hills, valleys, etc.). Around other axes, there isn't rotational symmetry because of an equatorial bulge caused by the rotation. --Tango (talk) 17:38, 15 May 2009 (UTC)[reply]
I was surprised to read about the following asymmetry today: if and when the West Antarctic Ice Sheet melts into the ocean, it will have a more pronounced sea-level-rise effect on the northern hemisphere because it's so massive that water is bunched up near it due to its own gravitational pull! [17] --Sean 18:05, 15 May 2009 (UTC)[reply]
Actually, it's pretty darned close to spherical. There was a discussion last week or so (too lazy to dig it out of the archives myself) which brought up the fact that the variations in heights on the earth surface are smaller than the tolerances allowed for regulation billiard balls, and the "equatorial bulge" which makes the earth's equatorial diameter larger than its axial diameter is also smaller than allowable variations in regulation billiard balls. Basically, the earth is actually smoother and rounder than a billiard ball. The variations look huge when placed on human scales, but when looking at the earth as a whole, they are quite tiny. Take Everest for example. It is 8.848 kilometers tall. The earth is about 6,371 km in diameter. Thus, the biggest bump on the earth represents 0.139% variation from the mean diameter. I would call that pretty darned smooth. Also, the variation between the polar and equatorial diameters is 21.3 kilometers, or 0.334% variation of the mean diameter. Again, pretty darned close to spherical. --Jayron32.talk.contribs 01:09, 16 May 2009 (UTC)[reply]
That's Quite Interesting! And I was thinking it all went pear-shaped about 6 billion years ago.--80.3.133.3 (talk) 08:25, 16 May 2009 (UTC)[reply]

units

i'm very confused about this question, not only about the procedure but about the units also.

if the velocity of light is taken as the unit of velocity and an year is taken as the unit of time, what's the unit of length? what's it called?

should i do: length = velocity of light * 1 year

and then subsitute them with their values in m/s and days, or something else....

please help me

thanx —Preceding unsigned comment added by 122.50.132.126 (talk) 15:40, 15 May 2009 (UTC)[reply]

Take a look the article Light-year and come back if you have additional questions. --Zerozal (talk) 15:57, 15 May 2009 (UTC)[reply]
... and for extra credit, ask your teacher whether their "year" is a Julian year, a sidereal year or a tropical year (but only once you understand the difference between those terms yourself). Gandalf61 (talk) 16:05, 15 May 2009 (UTC)[reply]
Dimensional analysis and the related factor-label method are concepts which are important to be familiar with (at least for science-inclined types). -- 128.104.112.117 (talk) 16:09, 15 May 2009 (UTC)[reply]

May 16

Dryer sheets

So I have some uniforms for work. On the laundry label it says, in big bold letters, "DO NOT USE DRYER SHEETS." I'm just curious, what's the big deal? Can dryer sheets ruin the clothes somehow? They seem so harmless. TravisAF (talk) 03:28, 16 May 2009 (UTC)[reply]

What line of business are you in? Maybe there is some special reason? If (for example) you were in the medical field - perhaps some patients are allergic to things on dryer sheets...I have no idea whether that's true - but it's a thought at least. SteveBaker (talk) 03:31, 16 May 2009 (UTC)[reply]
Hmmm TravisAF's user page says he's in the airforce. Airforce uniforms and dryer sheets? SteveBaker (talk) 03:33, 16 May 2009 (UTC)[reply]
From what I've been told, dryer sheets affect the absorption of cloth like towels. I doubt that applies here though... Dismas|(talk) 03:43, 16 May 2009 (UTC)[reply]
Google says "..not using fabric softeners or dryer sheets since chemicals from these could clog the pores of the uniform's polyester material and adversely affect performance."[18] and "Evidently the PTU is a special moisture resistant fabric with evaporative properties that is ruined by both liquid fabric softener and dryer sheets."[19] in the specific case of US armed forces PT uniforms. Nanonic (talk) 04:01, 16 May 2009 (UTC)[reply]
Microfiber clothes and cleaning cloths sold in Germany bear a warning label not to use fabric softener to avoid ruining them. I've never seen that on any microfiber materials sold in the States. I was wondering whether the European fabric softener was different or whether they just didn't expect Americans to follow such labels anyway, so why bother? 71.236.24.129 (talk) 11:18, 16 May 2009 (UTC)[reply]

Digestion

Some people claim that mans digestive system is unsuitable for digestion of flesh of the other animals, but neither can we digest uncooked plant products!!Like we cook meat before consumption we cook even vegetables..We cook most of the natural products.Is meat which is cooked also hard to digest??? —Preceding unsigned comment added by 59.165.84.9 (talk) 05:29, 16 May 2009 (UTC)[reply]

The enzymes in your stomach primarily break down meat while other biomolecules are broken down in the rest of the gastrointestinal tract. Humans can indeed digest uncooked plant products, such as bananas. JameKelly (talk) 07:44, 16 May 2009 (UTC)[reply]
I'm also intensiely curious about this subject. Many vegetarians - the ones who take the whole thing really seriously - says that the human digestive system doesn't digest meat properly, and that it basically sits in your gut and ... rots. Is there any truth to any claim that eating meat is really that bad for you? The only real facts I know about it (which may or may not be right =p) are that you can't get vitamin B12 from any natural source other than meat, and that the unhealthiest natural fats are also found only in meat... 90.193.232.41 (talk) 08:41, 16 May 2009 (UTC)[reply]
Partly they're playing word games. Digestion and rotting both describe the Decomposition of orgainc material. When the Gut flora breaks down long chained fats some toxic substances can get produced. I have no data on whether that only happens if the digestive enzymes haven't broken down the chains sufficiently or whether that always happens. The argument loses a lot of momentum if you look at food chemistry. There are plenty of toxic chemicals in vegetarian foods (Cyanide came up quite often, glycoalkaloids are in potatoes and cinnamon contains coumarin, to name just a few.) Problems arise when petri dish results are transferred to complex real life systems. Evidence based medicine often fails to come to the same conclusions as theoretical models based on studying individual components. Dietary supplements are often not taken up as efficiently as the real nutrients. (google a study for calcium from cheese vs, pills for an example. Don't know of a B12 study.) I have severe doubts that a diet that has been proven to be deficient in at least one essential nutrient can be healthier than a good mixed diet. The thing is we're not eating that either. Some meat is consumed dried rather than cooked. See e.g. Pemmican Jerky (food) For raw meat see Steak tartare. Countless fruits, nuts and vegetables are consumed raw. (Think of all the things you could put in a salad bowl)71.236.24.129 (talk) 11:02, 16 May 2009 (UTC)[reply]
Cooking meat does assist digestion, but it's not essential. A much more important concern is infectious disease. The modern processing, storage, refrigeration and handling of meat may lead to harmful bacteria growing in the meat. If the meat isn't cooked properly, this can lead to food poisoning. Axl ¤ [Talk] 11:24, 16 May 2009 (UTC)[reply]
Although improper handling of vegetables has also caused a lot of sickness as well, both bacterial (E. Coli for example) and viruses (Hepatitis A, from recent memory). Improper handling of food is bad no matter what it is. -- JSBillings 13:03, 16 May 2009 (UTC)[reply]


I'll just come out and say it: it is scientifically incorrect to say we are not able to digest meat well. Actually, it's totally wrong and out of touch with reality. We are actually very well designed for meat consumption: our stomach is full of gastric acid specifically tuned to the right pH to catalyze protease and break down protein found in meat. In fact, we are more specifically tuned for digesting meat than we are for cellulose, which must pass out as roughage. Although we are omnivores, we have more structure and metabolism in common with carnivores than with herbivores. Take a look at human canine teeth and incisors. These are not designed for chewing leaves and roots. (We have molars for that, but unlike an herbivore, we do not replace our molars every year or so). Anybody who wants to make the claim that we are biologically "designed" for vegetarian diets should reconsider their facts. Humans are omnivores. In a modern industrial society, we are able to provide sufficient nutrition with a wide variety of alternative diets (including vegetarianism, which may have some health benefits), but we did not evolve "naturally" to be herbivores. Nimur (talk) 16:14, 16 May 2009 (UTC)[reply]

Bird identification

Garden Warbler?

I asked on the Commons help desk if there was somewhere I could check a bird identification (to avoid misclassifying a photo) and was referred here. If this is the right place to ask, could someone tell me if the identification is correct? If not, is there somewhere else I can ask? N p holmes (talk) 09:00, 16 May 2009 (UTC)[reply]

I agree with your ID as a Garden Warbler - I looked at it in the Collins Bird Guide. It's the grey patches at the side of the head that clinch the ID. Wood and Willow Warblers are yellower, as is the Chiffchaff. The only real way to absolutely clinch the ID is to hear it, though: the Garden Warbler has the most wonderful song of the summer. --TammyMoet (talk) 09:22, 16 May 2009 (UTC)[reply]
Thanks. I did hear it (a fast song with interspersed slightly longer notes): not a Willow Warbler or Chiffchaff, but I've never heard a Garden Warbler. N p holmes (talk) 11:01, 16 May 2009 (UTC)[reply]

Suicide Bridge

Isn't it possible to survive a jump into the water from a suicide bridge? Assuming you don't bellyflop.68.148.149.184 (talk) 09:14, 16 May 2009 (UTC)[reply]

If you look at Drag (physics) The force depends on the density of the medium. Compare 1.204 for air to 1000 for water from our article. A person Diving off a board that is 10 m (about 11 yd) high will hit the water at almost 50 km/h (31 mph). They will try to achieve a mostly aerodynamic form when they enter the water and try to minimize the area of impact. They will also most likely not choose their head as the first point of impact, but try to create flow around their hands and arms first (also see La Quebrada Cliff Divers). A person jumping off one of those bridges would hit the water at a higher speed (maybe not quite terminal velocity) and will likely present a lot of vulnerable surface area to the impact. Even if they don't break their neck, their skin might break on impact and they'd bleed to death because blood doesn't clot effectively in water. 71.236.24.129 (talk) 10:10, 16 May 2009 (UTC)[reply]
The world record for the highest dive stands at over 50m (see Dana Kunze) whereas the mid-span clearance below the Sydney Harbour Bridge, for exampe, is 49m - so, yes, such a jump into water is theoretically survivable, but only with a high level of fitness and training. Gandalf61 (talk) 10:59, 16 May 2009 (UTC)[reply]
Presumably that record breaking diver survived the dive with no, or insignificant, injuries. If you land feet first from a pretty high bridge you can easily break your legs but otherwise survive as long as someone drags you out quickly enough and you get medical attention. I'm not sure what the highest survivable dive would be, but it has to be more than 50m. --Tango (talk) 13:32, 16 May 2009 (UTC)[reply]
There are MANY people who have survived falls from airplanes without a parachute ([20] for example) - landing in bushes or whatever. I'm sure some of those ended up in water. (See also Free-fall#Surviving_falls) So it's definitely survivable - even at terminal velocity - but it's definitely not certain - or even likely - so it's not recommended. There are more interesting cases in Category:Fall survivors - and some useful data on fall survivability in Alcides Moreno. SteveBaker (talk) 16:03, 16 May 2009 (UTC)[reply]

Age and energy

Why do people above 16 seem to have less energy than those below 16? Thank you. Clover345 (talk) 10:24, 16 May 2009 (UTC)[reply]

After 16 we save our energy for things that under-16s in the old days weren't supposed to know about. Cuddlyable3 (talk) 10:37, 16 May 2009 (UTC)[reply]
Er no, I don't think that's a very good answer. —Preceding unsigned comment added by 82.44.54.169 (talk) 11:11, 16 May 2009 (UTC)[reply]
What do you mean, energy? Olympic athletes are usually not under sixteen. Similarly for Artic explorers who pull sledges to the poles. Perhaps you are thinking about under 16s being less inhibited with moving their bodies around than older people are, such as break-dancers. 78.146.190.197 (talk) 13:32, 16 May 2009 (UTC)[reply]
I think simple physics explains this. Assume your "under 16" kid is half your adult height - that means that they have something like one-eighth of your weight and their muscles have one quarter the cross-sectional area. The power produced by a muscle is proportional to it's cross-sectional area - and the amount of work it has to do to (say) have the kid run around yelling something annoying over and over again - is proportional to the kid's mass. So their power-to-weight ratio is twice that of an adult that is twice their size. So I'd conclude that while they might SEEM to have twice the "energy" you have - in reality, they are really able to produce twice the acceleration of one eighth the mass. Obviously as your size approaches that of an adult - this capability gets less and less pronounced. Once you are 90% of the size of an adult (perhaps) at age 16, you have pretty much reached that all-time-low. SteveBaker (talk) 15:54, 16 May 2009 (UTC)[reply]

Swans - can they break your arm?

I have often heard that a swan can break your arm. There isn't anything on the swan article about this - is this true or is it an urban myth. Also should there be something in the swan article about this? Catoutofthebag (talk) 11:09, 16 May 2009 (UTC) Additional question Also are there any recorded incidents of people having their arms broken by swans? Catoutofthebag (talk) 11:42, 16 May 2009 (UTC)[reply]

Whooper swan says they can be up to 15 kgs. That means their wings can produce enough lift (force) to move those 15 kgs. Think of hitting something with a 15 kg Mallet. People who handle swans learn how to disable their wings when grabbing them, from experienced swan handlers. (Only example I know is from Hamburg, Germany de:Eppendorfer Mühlenteich.) —Preceding unsigned comment added by 71.236.24.129 (talk) 11:38, 16 May 2009 (UTC)[reply]
The "mallet" would have a large wing attached to it, which would slow it down. And being kicked by a 200lb person is probably not going to break a limb either. It is unlikely that Swans move their wings fast enough to provide 15kg of lift when threatening people, as they would either take-off or fall over if the wings were flapped horizontally. 78.146.190.197 (talk) 14:37, 16 May 2009 (UTC)[reply]
It's a myth.--80.3.133.3 (talk) 12:56, 16 May 2009 (UTC)[reply]
Swans in the UK are in theory owned by the Queen. For hundreds of years they have been owned by royalty, and I believe were eaten by them. I understand that the sawns-can-break-your-leg myth was just put about to stop the peasants from eating them. 78.146.190.197 (talk) 13:26, 16 May 2009 (UTC)[reply]
On that site it says "From Swanuk.org: "Yes, but only in exceptional cases. If a wing in full span and velocity were to hit a weak-boned person (such as a child or an elderly person) then it is theoretically possible. In reality it is almost unheard of and is never used as a form of attack as swans are a defensive bird. The only time they become aggressive is when they are protecting their nesting ground or cygnets when they will chase off intruders, be they other swans, geese or humans who get too close." So is the answer a yes or a no? Or a theoretical yes. Catoutofthebag (talk) 13:42, 16 May 2009 (UTC)[reply]
Parsing the quoted text above indicates that the answer is "no" in almost all circumstances. The answer could only be "yes" in theory for a child or elderly person. 78.146.190.197 (talk) 13:48, 16 May 2009 (UTC)[reply]
Anecdote time. I used to share a house with a guy who slaughtered domestic geese for a living (several years before). He told me that a goose had once broken his wrist with a strike from its wing as he was trying to restrain it. He did have some sort of bone condition though... --Kurt Shaped Box (talk) 15:14, 16 May 2009 (UTC)[reply]
OR: My Thanksgiving turkey from last year was a fell beast, at least as strong as a swan, and it gave me some pretty good whacks, but nowhere near hard enough to break anything. --Sean 14:53, 16 May 2009 (UTC)[reply]

is there any evidence that the NSA uses Papal infallibility?

Is there any evidence (a leak, etc) of cooperation between the NSA and the vatican for using Papal infallibility to break encryption, for example, by factoring large composite numbers. Once the factors are produced, they are very easy to verify... --94.27.244.146 (talk) —Preceding undated comment added 11:25, 16 May 2009 (UTC).[reply]

I am very confused as to what you are asking. Are you saying that the Pope could theoretically use infallibility to have God factor the numbers for him? If so, this is really a question for the humanities desk. Although, if this is what you're asking, it is ridiculous; Papal infallibility only applies to matters of faith. The Pope can't decree who's going to win the Super Bowl, after all :-D -RunningOnBrains 11:41, 16 May 2009 (UTC)[reply]
Papal infallibility means the Pope can't be wrong, though you're right in that it applies to matters of faith. However, maybe there is a way to spin a large number with mysterious, unknown factors into a question of faith? If so, is there any evidence that the NSA has done this in cooperation with the Vatican? (The reason it's not "ridiculous" -- I mentioned this before -- is that the NSA doesn't have to believe in papal infallibility for it to work: the numbers are very easy to verify once the Pope produces them.) Any evidence of this ever being done? --94.27.244.146 (talk) 11:59, 16 May 2009 (UTC)[reply]
Believing that God would be swayed by any kind of spin is about as dumb as believing a good lawyer could get you out of Hell. SpinningSpark 12:04, 16 May 2009 (UTC)[reply]
(edit conflict) Again, you misunderstand the concept of Papal infallibility (I suggest you read the article). It does not suggest that the Pope can not be wrong, it is a provision where a Pope, if given a revelation by the Holy Spirit about church doctrine, can speak ex cathedra, and can not be accused of incorrect reasoning (since the "reasoning" came from God himself). -RunningOnBrains 12:06, 16 May 2009 (UTC)[reply]
I'm sorry, but there is no reason church doctrine cannot be massaged into including a factorization. The Vatican could very easily come up with questions of doctrine for the Pope the answering of which would be equivaelnt to factorizing a number. No reason it couldnt be done, and you have not given any. But I've just realized something: you haven't answered my actual question, about any evidence of cooperation between the NSA and the Vatican, at all! So if there is some evidence of that, then your whole answer could be considered a deterrent... --94.27.244.146 (talk) 15:41, 16 May 2009 (UTC)[reply]
There is a popular misconception that papal infallibility means that the pope is never wrong. That is not the case. It's not even the case that it means that the pope is never wrong when talking about religion/faith. What it does mean is that under certain conditions the pope can proclaim that he has been divinely inspired as to the truth of a certain matter. The Catholic philosophy is that God (the Holy Spirit) would not allow the pope to be wrong when making such ex cathedra proclamations. (Again, while ex cathedra means "from the chair", it doesn't actually mean that there is a special chair he sits on which makes him infallible. The chair is a metaphor referring to his official position as pope.) There are a number of rather strict conditions that must be met before papal infallibility is considered to hold - it is not used on a routine basis. Out article on papal infallibility in fact only lists seven instances in the past 2000(ish) years, the most recent one being back in 1950. Pope John Paul II did not make any ex cathedra proclamations, despite having a relatively long tenure of 27 years -- 76.201.155.7 (talk) 12:15, 16 May 2009 (UTC)[reply]
I understand there are very special conditions which must be met for such a procalamtion. Is there any evidence that shows the NSA and Vatican cooperate secretly to make, but not disseminate, such proclamations? Also, your implication that there are only seven instances in the last 2000 years is misleading...the article only lists seven that were recounted in a 1985 study, and the Wikipedia article you just linked continues "The Vatican itself has given no complete list of papal statements considered to be infallible.". As far as I can see the conditions, he would have to speak on a matter of faith that must apply to all followers of the church. I don't see why such a proclamation could not at once answer a question important to the NSA, and I also dont see why it should not be possible not to disseminate this proclamation (keep it secret). If there is no evidence of NSA cooperation with the vatican, why don't you, and the other poster above, just say this? —Preceding unsigned comment added by 94.27.244.146 (talk) 15:51, 16 May 2009 (UTC)[reply]
Is this question based upon some sort of conspiracy theory, as a matter of interest? --Kurt Shaped Box (talk) 15:59, 16 May 2009 (UTC)[reply]
Even if we accept Christianity and Papal Infallibility as fact, This isn't something the pope can do on demand. (Too bad. If it worked like that it'd be extremely testable. We could have a scientific proof of God in about ten minutes.) The only reason he's infallible is because he's directly relaying a message from God. If God sent you a message personally it'd be just as infallible, except no one would ever believe you. Presumably if God wanted to help the NSA crack some codes, He wouldn't even have to bother the Pope. He could send the decryption keys to the NSA directly. 16:27, 16 May 2009 (UTC)
Nobody on here has yet said categorically, "no, there is no such evidence" because nobody has ever researched such a topic. There is no reference we can cite that says "there is no evidence". But even so, I am going to go out on a limb here, and say, no, there is no reliable evidence that the NSA and the Pope have been conspiring together to use whatever misinterpretation of papal infallibility you are working under to break encryption techniques. (I estimate a very low probability that I will be proven wrong in this claim). The very notion is ludicrous, but since you won't give up until someone tells you "no", here it is. No. Maelin (Talk | Contribs) 16:36, 16 May 2009 (UTC)[reply]

How to identify flowering plants - both wild and garden?

There are several flowering plants I would like to identify, both wild flowers (here in the UK) and garden flowers. Is there any online database that helps in the identification of such flowers? And are there any usefull clues to narrowing the identification down, such as the number of petals? I have a copy of a flora - the New Flora Of The British Isles by Clive Stace - but you need to be a trained latin-speaking botanist to use it, and it has very few illustrations. 78.146.190.197 (talk) 13:16, 16 May 2009 (UTC)[reply]

Here's [21] somewhere to start. You'll need to submit info, but it also serves to familiarise you with some technical words which might assist you with your book. Richard Avery (talk) 14:21, 16 May 2009 (UTC)[reply]

Adaptation or death effect?

I was chatting with a friend and my thoughts sort of got off on a tangent about the human body. I know that, for example, if you eat too much of X you might end up actually needing to eat excess amounts of X because your body is "dependant" on the higher amount. The way people talk about it you'd think the body is aware of itself and makes conscious decisions. Obviously untrue. But is there a specific mechanism in the body that makes it absorb less of something if excessive amounts are present, as a failsafe? Or do the cells that absorb it naturally end up dying from the overdose, leaving only cells that absorb it inefficiently to survive and replicate themselves (a bit like how "supervirii" come about)? 97.104.210.67 (talk) 15:54, 16 May 2009 (UTC)[reply]

  1. ^ Epstein, L.C. Thinking Physics. San Francisco: Insight Press. ISBN 0-935218-06-8